You are on page 1of 91

VERBAL ABILITY

HAP_VA_BTECH_SEM_4_1218

READING COMPREHENSION – I
SESSION – 1

Directions for Q1 to Q10: Read the following information and answer the questions given below.

The Most Productive Day of the Week


It's possible that while you are at work, you may dream about a month of Sundays, but your boss wishes for a week of
Tuesdays. That's because s/he probably knows that productivity is one of the main factors bolstering a company's
growth. And a recent poll shows that workers are most productive on Tuesdays! Accountemps, an employment
agency, conducted a national survey of office managers, which shows that by the middle of the week, they see a
dramatic productivity decrease. While Monday is considered second in "productivity value," only nine percent of
office managers think Wednesday is the peak productivity day. Five percent believe it is Thursday. And Friday, well,
you can just imagine! However, forty-eight percent of the managers polled said that Tuesday is, by far, the most
productive day of the week.
A close analysis of workweek rhythms would turn up some obvious reasons for those survey results. First of all,
Monday is overloaded with meetings, designed to "get things moving," and everybody knows meetings aren't very
productive. Wednesday is "hump day" - get over it as painlessly as possible, a worker thinks, and the week is more
than halfway over. On Thursday, people are running out of steam; and Friday, everybody's thinking about the
weekend. There are reasons why the other days aren't productive, but what makes Tuesday special? Tuesdays,
employees hit peak performance because they are very focused on day-to-day activities. Also, it's usually the first day
of the week when they're focused on their own task. They're not in meetings that take them away from their primary
responsibilities. Actually, Tuesdays can be quite hectic. Workers are arriving at work fairly frantic. And so, in 10 hours,
they're doing 20 hours work. That's productive, but it's also tough.
This does not mean that nothing happens on the last three days of the workweek. Things do not get so lax that people
are sitting with their feet on desks, sipping coffee and talking on the phone all day, but there's a definite lack of focus.
The pace softens and the rhythm slows down. And this is not healthy: it produces fatigue and lowers productivity. To
prevent this midweek slowdown, some management consultants suggest that employers avoid jamming so many
meetings into Mondays. Work deadlines can be rescheduled to stretch out the workflow. Variations in productivity are
only natural, but both workers and bosses win when the peaks and valleys are less dramatic than they are now.
1. This reading does not explain
(a) the concern managers have about low productivity on different week-days and how it affects their companies
(b) the reasons for differing productivity levels on different week-days
(c) the results of a survey on low productivity

2. According to this survey which day of the week should be the most productive?
(a) Monday (b) Thursday (c) Wednesday

3. According to the survey workers are most stressed on


(a) Monday (b) Tuesday (c) Thursday
4. The writer is suggesting that
(a) workers would be more productive if they worked on other days as they do on Tuesday
(b) workers would be more productive if they attended fewer meetings
(c) workers would be more productive if the work deadlines were readjusted

5. In general this article


(a) reports the surprising results from a national survey of office managers
(b) gives the reasons why Tuesday is a particularly productive day in the work week
(c) reports on the productivity for different work days and makes suggestions for making work week
productivity more balanced

Page 1 of 91
VERBAL ABILITY

HAP_VA_BTECH_SEM_4_1218

Okinawans Know the Formula for Longevity


Okinawa is a small island in Japan. Some of the people who live there are the poorest people in the country. But
Okinawans enjoy riches of a different kind—they have the longest life-expectancy rate in the world. The average
Okinawan woman lives to the age of 86, while the average Okinawan man lives to be 78. The island also has the
highest ratio of centenarians in the world. At the beginning of 2002, 457 Okinawans were 100 years old or older.
That's 34.7 centenarians per 100,000 Okinawans. Compare that to the United States, which only has 10 centenarians
per 100,000 Americans.

Okinawans don't wait out the final years of their long lives in hospitals or convalescent care facilities. They remain
healthy through their final days. Dementia, senility, and coronary heart disease are almost nonexistent among these
older Okinawans. In fact, compared to the United States, Okinawa's death rate for coronary heart disease is 80%
lower. And Okinawans are more likely than Americans to survive heart attacks on the rare occasions that heart
attacks occur.

So what is the secret to the Okinawans' health and longevity? Is it a healthy diet? Exercise? Stress reduction? Attention
to spirituality? It isn't any one of these things, but a balanced combination of the four.

The Okinawan diet consists mostly of vegetables and whole grains. Okinawans also eat a lot of fruit, soy products, like
tofu, and fish rich in Omega-3 fatty acids, such as tuna, mackerel, and salmon. Most Okinawans drink at least six
glasses of water and six cups of tea every day. The green tea that many of them drink is lower in caffeine than other
teas and is rich in antioxidants, which are believed to help prevent cancer, heart disease, and stroke.

Okinawans continue to practice martial arts, ride bicycles, dance, garden, walk, and fish well into their old age, which
helps keep them looking and feeling healthy. They have relaxed schedules, which reduces their stress levels. And they
pay attention to their spirituality and their inner selves through prayer and meditation. Each of these behaviors is an
important piece of the Okinawan formula for health and longevity.

There is evidence that it is their behavior, not genetics, that allows Okinawans to live longer than most of us.
Okinawans living in Brazil and eating a typical Brazilian diet rich in red meat have a life expectancy 17 years lower
than that of their countrymen in Okinawa. This means that there is hope for the rest of us to learn the habits that
Okinawans have developed so we can live long, healthy lives too.

6. Okinawans _____ than anyone else in the world.


(a) are richer (b) live longer (c) farm and fish more

7. People in Okinawa eat a lot of _____.


(a) vegetables, fruit, soy, and whole grains
(b) vegetables, read meat, soy, and fish
(c) vegetables, soy, milk products, and whole grains

8. The four things that contribute to Okinawans' long lives are a healthy diet, exercise, _______
(a) longevity and spirituality
(b) low stress levels and spirituality
(c) martial arts and spirituality

9. At the beginning of 2002, there was an average of _____ centenarians per 100,000 people in Okinawa.
(a) 457 (b) 10 (c) 34.7

10. Okinawans have such long lives because of _____.


(a) genetics (b) their behavior (c) their diet

Page 2 of 91
VERBAL ABILITY

HAP_VA_BTECH_SEM_4_1218

READING COMPREHENSION - II
SESSION – 2

Directions for Q1 to Q10: Read the following information and answer the questions given below.

Modern Day Meditation

In a world where bad news has become everyday news, people are turning to an ancient technique to deal with stress:
meditation. At meditation centers, prayer groups and yoga studios around the United States, more and more are
finding peace of mind by being quiet. Some use meditation to help deal with life changes; others, to process the painful
reality of political and social unrest around the world of the type that has been experienced more recently. Stress from
the September 11 terrorist attacks is "probably about 70 percent" of the reason one Chicago man started meditating
and practicing yoga with his new wife. He became so emotionally affected that he realized he needed help in managing
his stress. The yoga classes he takes begin and end with meditation. This "quiet time" helps him feel a lot more relaxed
and gives him more breath control. The fact is, though, that he is not alone.

Across the country, many are turning to more meditative exercise as they seek both psychological and physiological
relief. In addition to helping people work out their stress, these classes bring people together, in the same way that
religious services or other community activities have done in the past. Different schools of meditation teach particular
techniques, but they share a common basis - focusing attention on something your mind can return to if you are
distracted. This may be the rhythm of breathing, a mantra (a word or phrase repeated continually, either silently or
aloud), an object such as a candle flame or religious icon, a positive affirmation, feelings of loving kindness, or a repetitive
movement, as in walking or t'ai chi. Regardless of the specific technique or mode that is followed, meditation has well-
documented benefits. Medical research indicates that it causes a sharp decrease in metabolic activity, reduced muscle
tension, slower breathing, and a shift from faster beta brainwaves to slower alpha, theta and delta waves. It also reduces
high blood pressure. Practitioners are convinced that meditation is good for health because it relaxes the body.

For ages, meditation has been a core practice of many groups meeting in their communal or religious centers.
However, let's not forget that this is the twenty-first century. So, for those people who are too shy or busy to go to the
nearest meditation center, there are Internet sites that offer online guided meditation. One has a variety of
meditations from various religious traditions. At another, Jesuit priests post meditations and readings from the
Scriptures every day, and at still another, Buddhist and Hindu practitioners include music and visuals to accompany
their offerings. These websites allow anyone with a computer access to meditation at any time. The fact is that
whether online, at yoga classes, or at local spiritual centers, more people are turning to the practice of meditation.

1. According to this article meditation is becoming more popular today because


(a) people have to work much harder than in the past
(b) people have to work much harder to find peace of mind
(c) people have to cope with more difficulties in their personal and communal lives

2. The example of one Chicago man shows


(a) the physical and psychological benefits of yoga and meditation
(b) the physical benefits of yoga and meditation
(c) the psychological benefits of yoga and meditation

3. What is the basic underlying approach of all meditative techniques?


(a) People repeat a phrase over and over.
(b) People focus on something that they can return to.
(c) People concentrate on a candle flame or a religious object.

Page 3 of 91
VERBAL ABILITY

HAP_VA_BTECH_SEM_4_1218

4. What two important points are explained in this reading?


(a) An explanation of the various teachers and the different schools of meditation.
(b) A description of the places to go and procedures people can follow to learn meditation.
(c) An explanation of the best practices and various health benefits from meditation.

5. What is one idea that is explained in this reading?


(a) The future growth of religious centers that teach meditation.
(b) The present day appeal of meditation classes compared to the past attraction to religious or community
celebrations.
(c) The increasing popularity of meditation through the availability of on-line resources.

The Second "Battle of New Orleans"


In 1815, the pirate-turned-patriot Jean Lafitte joined the future president Andrew Jackson to protect Louisiana, the
Mississippi River, and America from a British military invasion. Lafitte pulled, pushed, and floated his large guns
through bayous, swamps, and marshes to fight in the Battle of New Orleans. In a shocking victory, Jackson and Lafitte
routed the British and saved New Orleans, Louisiana and the Mississippi River for the United States. Today, due to the
work of modern man and progress, the wetlands that Lafitte crossed to protect his homeland are being swallowed by
the salt water of the Gulf of Mexico. The state of Louisiana and the Mississippi Delta are gradually disappearing. The
coastal wetlands are lost at the rate of 100 yards, the length of a football field, every 15-20 minutes.

Over many thousands of years, nature created Louisiana and the Mississippi Delta. Along the over 2,000-mile course,
the Mississippi River drains soil from much of America's heartland to create the rich land of the Mississippi Delta.
Over time, the Delta region grew abundant in vegetation, fish, ancient trees, and other life forms. But in the last two
centuries, the wetlands created by the Mississippi were drained for a variety of reasons. At first it was to stop diseases
that endangered people who moved there and to grow crops to feed America and the world. Then the valuable
cypress trees were cut down to build homes and to export the wood. Natural canals were turned into large channels to
facilitate transportation. This massive work was called "reclamation," the reclaiming of the land from the water that
the Mississippi River delivered. Later, more land was reclaimed to develop a lucrative and thriving petroleum
industry. Then, more canals were needed to make way for large ships. Levees were built for protection from the
annual spring floods. Ultimately, less fresh water drained into the Mississippi Delta.

Now, the natural habitat that nurtured wildlife, 300 species of birds, thousands of species of fish, flora, and other
ecological systems is rapidly disappearing. The ecology-friendly island barriers that were created by the flow of the
Mississippi and protected the coast have crumbled into the salt water. The salt water, without the island barriers and
the soil drainage of the Mississippi, now comes to meet the great river. The fresh water ecological system is dying and
disappearing. Daily, the salty Gulf waters cover what was once a thriving wetland full of fresh water life.

There are plans to save the coast, re-establish freshwater flooding, and drive back the advancing salt water. One plan
spends $14 billion dollars over the next 20 years to save this fragile, endangered, and disappearing land. Most people
see a need to protect their homeland. Some others disagree with taking immediate steps to protect the wetlands and
environment. It is safe to say that the battle has begun. Whether an Andrew Jackson or a band of pirates led by Jean
Lafitte will come to Louisiana's rescue in this war for the environment is not yet known. We can be sure though, in the
end, nature will have the final say.

6. Andrew Jackson ____________ in 1815.


(a) protected Louisiana from the Mississippi River
(b) was forced to join the British to fight pirates on the Mississippi
(c) attempted to stop a British invasion of America

Page 4 of 91
VERBAL ABILITY

HAP_VA_BTECH_SEM_4_1218

7. In Louisiana, coastal wetlands ______________.


(a) are disappearing daily
(b) enjoy the refreshing salt water from the Gulf
(c) are vital to the expansion of the state of Louisiana
8. The Mississippi River _______________.
(a) has remained the way it has always been for thousands of years
(b) has changed rapidly in the last few years to meet the needs of a growing population
(c) has changed steadily over the last two centuries to meet the needs of the population
9. The island barriers _____________.
(a) have grown as a result of the flow of the Mississippi
(b) have changed the flow of the Mississippi
(c) have disappeared as a result of the flow of the Mississippi

10. There is _______________ to drive back the salt water and save the wetlands.
(a) general agreement that a plan should be made
(b) complete agreement that a plan must be made
(c) little agreement that a plan can be made

READING COMPREHENSION - III


SESSION – 3

Directions for Q1 to Q4: Read the following passage and answer the questions given below.

Rohit brushed quickly past an elderly woman waiting on the platform ahead of him to get onto the metro. He wanted to
be sure to get a seat to read his Economic Times. As the train rolled out of the station, he lifted his head from the
newspaper and stared at the man directly across from him. A tsunami of antipathy came over him. Rohit knew this man,
knew him all too well. Their eyes locked. As the train readied full speed, the ruckus of speeding wheels against the
winding rails and a wildly gyrating subway car filled Rohit's ears. To this frenetic beat Rohit effortlessly listed in his
head all the reasons this man, whose eyes he stared coldly into, was an anathema to him. He had climbed the upper
echelons of his firm using an imperious manner with his subordinates, always making sure everyone knew he was the
boss.

Despite his impoverished upbringing, he had become ostentatious. Flush with cash from the lucrative deals he had
made, he had purchased a yacht and a home in Mumbai. He used neither. But, oh, how he liked to say he had them.
Meanwhile, Rohit knew, this man's parents were on the verge of being evicted from their run-down tenement
apartment in Allahabad.
What bothered Rohit most about this man was that he never even attempted to make amends for his evil ways. Could
this man change? Rohit did not know. He could try though.
The train screeched to Rohit‘s stop. He gave the man one last hard look. ‘See you around," he mumbled to himself. And he
knew he would, because Rohit had been glaring at his own reflection in the glass in the metro. It would take years of hard
work and therapy, but Rohit would one day notice this man again on the train and marvel at what a kinder person he had
become.

1. Why did a tsunami of antipathy come over Rohit?


(a) Because he was angry at himself and unable to stand looking at himself.
(b) Because the man sitting across him was his former boss who treated him badly.
(c) Because he wanted to read his newspaper and not be disturbed, especially by someone he disliked.
(d) Because the guy sitting across him was financially better off than Rohit.

Page 5 of 91
VERBAL ABILITY

HAP_VA_BTECH_SEM_4_1218

2. What was the biggest reason (stated or implied) for Rohit disliking the man in the metro?
(a) The man was known to be extremely rude and domineering especially with his subordinates.
(b) The man was remorseless and had not made any effort to reform himself for the better.
(c) The man did not bother to take care of his parents who were on the verge of being evicted from their
humble dwelling.
(d) The man did not have respect for things or money and while people did not have a place to stay, he had
bought a flat which he did not even use.

3. Which statement makes most sense from what is said in the paragraph?
(a) Rohit has few friends. (b) Rohit knows himself well.
(c) Rohit has had a difficult life. (d) Rohit is incapable of change.

4. What does it mean to have an 'imperious manner with underlings'?


(a) To ignore them. (b) To be stoic around them.
(c) To openly humiliate them. (d) To not be affected by them.
(e) To be domineering towards them.

Directions for Q5 to Q8: Read the following passage and answer the questions given below.

The beneficial aspects of ocean currents have long been known. For countries on the East side of the Atlantic, winters
are a balmy holiday compared with the same latitudes on the West: the frigid coasts of Newfoundland and Labrador.
It’s a remainder that “weather” is not just a matter of the Sun’s heat affecting the Earth’s atmosphere. The world’s
interconnected oceans can store up solar heat in one part of the globe in the season, and invisible rivers in the ocean
can transport the warmth thousands of kilometres to another part of the globe and deliver it in another season.

In the case of the North Atlantic, heat is carried Northward and Eastward by the Gulf Stream. This current warms the
coast evenly through the year, in winter as well as summer. Averaged over a year, the Gulf Stream provides Western
Europe with a third as much warmth as the Sun does.

This ocean warmth is so important to Europe that climatologists are seriously concerned about the stability of the Gulf
Stream. If it switched off, Europe would be plunged into a mini-ice Age. And current studies suggest that the unseen
river in the North Atlantic is dangerously fickle.

The focus of today’s worries is the problem of global warming – the way that human activities are changing the
climate, as the world gets warmer through the build-up of so-called greenhouse gases, such as carbon dioxide.
Climatologists think that global warming may put the brakes on the Gulf Stream. While the rest of the world comes to
swelter in greenhouse conditions, Europe would freeze!

This concern is based on a new understanding of how the great ocean currents are all interconnected. The Gulf Stream
is part of a giant pattern of moving water that stretches right around the globe.

5. Which of the following statements are correct about Gulf Stream?


(a) It is an ocean which spreads warmth evenly across the coast
(b) Solar heat is carried in North-East direction
(c) It is a part of giant pattern of moving water around the world
(d) Both (a) and (c)

6. What are the main concerns of climatologists?


(a) The entire planet will be warmer with increase in global warming
(b) Greenhouse gases can change directions of ocean currents
(c) Interconnection of currents can be harmful for oceanic life
(d) Global warming can stop the flow of Gulf Stream

Page 6 of 91
VERBAL ABILITY

HAP_VA_BTECH_SEM_4_1218

7. What is the possible meaning of ‘fickle’?


(a) Constant flow in one direction (b) Likely to change due to instability
(c) Modify under extreme conditions (d) Fixed at one place

8. What is the role of ocean currents in maintaining the weather conditions?


(a) It helps in maintaining cold temperature in warmer regions
(b) It balances the effect of Sun’s heat
(c) It stores heat and warms distant places
(d) Unpredictable currents can create an imbalance in water temperature

Directions for Q9 to Q12: Read the following passage and answer the questions given below.

My phone rings again. It is futile to ignore it anymore. Maneesha is persistent. She will continue to bedevil me until
I acquiesce.

“Hello”, I answer.

“The circus, Atika?” she says in her sing-song voice. “When are we going ? Only two more days left! ”

I abhor the Circus. The boisterous crowds, the overwhelming smell of animal feces, the insanely long lines with
wailing children and the impossibility of finding a clean restroom all combine to make this an event that I dread.

For Maneesha, my best friend since the angst of middle school, the Circus is a sign that divine powers really do exist.

“Really, Atika, where else can you pet an elephant, see a stuntman ride a horse, laugh till you are ready to cry, see the
world’s smallest person and eat fried potatoes and butter soaked popcorn?” Maneesha asks gleefully.

“Hell?” I guess.

The fried food at the Circus is a gastronomical nightmare on its own. I once tried a fried Cottage Cheese stick at the fair
and was sick to my stomach for hours. And a fried burger with oil soaked potato patty, cheese, multicolored sauces
AND a greasy slice of cottage cheese? How could that not be deleterious to your health?

I have not seen Maneesha for a good month; our schedules are both so hectic. My hatred of the Circus becomes
inconsequential to my desire to hang with Mani.

Alas, I ignore my anti-Circus bias for the umpteenth year. “Pick me up at noon”, I say and hang up the phone.

9. What does it mean to acquiesce?


(a) To give in (b) To speak kindly
(c) To pay attention (d) To answer the phone

10. Why might the author have chosen to capitalize all the letters in the word "and" when writing about the burger
she ate?
(a) To make sure the reader understood it was a list.
(b) To show that a greasy slice of cottage cheese was the last ingredient.
(c) To highlight her dislike of greasy slice of cottage cheese.
(d) To emphasize how many ingredients were in the burger.

11. How does Maneesha seem to feel about the circus?


(a) Ambivalent (b) Condescending (c) Jubilant (d) Nonchalant

12. What does the term "gastronomical" suggest?


(a) Enormous (b) Health risk (c) Culinary issue (d) Resulting in gas

Page 7 of 91
VERBAL ABILITY

HAP_VA_BTECH_SEM_4_1218

Directions for Q13 to Q20: Read the following passage and answer the questions given below.

The impressive recent growth of certain sectors of the Indian economy is a necessary but insufficient condition for the
elimination of extreme poverty. In order to ensure that the poorest benefit from this growth, and also contribute to it,
the expansion and improvement of the microfinance sector should be a national priority.

The Studies suggest that the impact of microfinance on the poorest is greater than on the poor, and yet another that
non-participating members of communities where microfinance operates experience socio-economic gains –
suggesting strong spillover effects. Moreover, well-managed microfinance institutions (MFIs) have shown a capacity
to wean themselves off of subsidies and become sustainable within a few years.

Microfinance is powerful, but it is clearly no panacea. Microfinance does not directly address some structural problems
facing Indian society and the economy, and it is not yet as efficient as it will be when economies of scale are realised and
a more supportive policy environment is created. Loan products are still too inflexible, and savings and insurance
services that the poor also need are not widely available due to regulatory barriers.

Still, microfinance is one of the few market-based, scaleable anti-poverty solutions that is in place in India today, and
the argument to scale it up to meet the overwhelming need is compelling. According to Sa-Dhan, the overall outreach
is 6.5 million families and the sector-wide loan portfolio is Rs.2,500 crore. However, this is meeting only 10% of the
estimated demand. Importantly, new initiatives are expanding this success story to some of the country's poorest
regions, such as eastern and central Uttar Pradesh.

The local and national governments have an important role to play in ensuring the growth and improvement of
microfinance. First and foremost, the market should be left to set interest rates, not the state. Ensuring transparency and
full disclosure of rates including fees is something the government should ensure, and something that new technologies
as well as reporting and data standards are already enabling. Furthermore, government regulators should set clear
criteria for allowing MFIs to mobilise savings for on-lending to the poor; this would allow for a large measure of financial
independence amongst well-managed MFIs — as the Grameen Bank of Bangladesh has achieved in recent years through
an aggressive and highly successful savings initiative. Each Indian state could consider forming a multi-party working
group to meet with microfinance leaders and have a dialogue with them about how the policy environment could be
made more supportive and to clear up misperceptions.

There is an opportunity to make a real dent in hard-core poverty through microfinance. A new paper, "Measuring the
Impact of Microfinance: Taking Stock of What We Know" (which can be downloaded for free from website), shows
that in most countries where quality microfinance programmes have been scaled up, poverty has been reduced,
sometimes dramatically. With one state leading the way, we need to build on a successful model. By unleashing the
entrepreneurial talent of the poor, we will slowly but surely transform India in ways we can only begin to imagine
today.

13. Why according to the author, should microfinance be scaled up in India?


(a) The demand for microfinance is high (b) It is a market-based anti-poverty solution
(c) It is sustainable (d) Both (a) and (b)
(e) (a), (b) & (c)

14. What could be the meaning of the word ‘panacea’ in the passage?
(a) Solution (b) Problem
(c) Solution to all problems (d) Sustainable solution

15. Why are saving products not available?


(a) Due to inflexibility of loan products (b) Due to regulatory restrictions
(c) Since insurance services are not available (d) Saving products are not available

Page 8 of 91
VERBAL ABILITY

HAP_VA_BTECH_SEM_4_1218

16. Why does the author talk about the ‘enterpreneurial talent of poor’ in the concluding paragraph?
(a) Enterpreneurship among poor is encouraged by microfinance
(b) Enterpreneurship among poor is an alternate to microfinance
(c) Enterpreneurship among poor is discouraged by microfinance
(d) None of these

17. Which of the following is not a challenge faced by microfinance in India?


(a) Does not help the poorest (b) Efficient when economy of scale is achieved
(c) Non-conducive policy environment (d) Structural problems of Indian society

18. Which of the following will the author agree to?


(a) Indian economy growth will solve the problem of poverty
(b) Indian economy growth is not enough to solve the problem of poverty
(c) Indian economy growth aggravates the problem of poverty
(d) None of these

19. Which of the following is correct with regard to microfinance?


(a) The supply is more than demand (b) The demand is more than supply
(c) The supply and demand are well balanced (d) None of these can be inferred from the passage

20. What is the author’s view about interest rates?


(a) The government should set them
(b) There should be transparency with regard to them
(c) The market forces should set them
(d) Both (a) and (b)
(e) Both (b) and (c)

Directions for Q21 to Q28: Read the following passage and answer the questions given below.

Since the late 1970s when the technology for sex determination first came into being, sex selective abortion has
unleashed a saga of horror. Experts are calling it ‘Sanitised Barbarism’. Demographic trends indicate the country is
fast heading towards a million foetuses aborted each year.

Although foetal sex determination and sex selection is an offence in India, the practice is rampant. Private clinics with
ultrasound machines are doing brisk business. Everywhere, people are paying to know the sex of the unborn child,
and paying more to abort the female child. The technology has reached even remote places through mobile clinics. Dr.
Puneet Bedi obstetrician and specialist in foetal medicine, says these days he hardly sees a family with two daughters.
People are getting the sex determination even for the first child, he says.
The 1991 census showed that two districts had a child sex ratio (number of girls per thousand boys) less than 850; by
2001 it was 51 districts. Child rights activist Dr. Sabu George says foeticide is the most extreme form of violence
against women. "Today a girl is several times more likely to be eliminated before birth than die of various causes
during the first year. Nature intended the womb to be a safe space. Today doctors have made it the most unsafe space
for the female child," he says. He believes that doctors must be held responsible – “They have aggressively overnight
the misuse of technology and legitimised foeticide.”

Akhila Sivadas, Centre for Advocacy and Research, Delhi, feels that the PCPNDT Act (Pre-Conception and Pre-Natal
Diagnostic Techniques – Regulation and Prevention of Misuse) is very well conceived and easy to use. The need of the
hour is the legal literacy to ensure the law is implemented. “The demand and supply debate has been going on for
some time. Doctors say there is a social demand and they are fulfilling it. They argue that social attitudes must change.

Page 9 of 91
VERBAL ABILITY

HAP_VA_BTECH_SEM_4_1218

However, in this case supply fuels demand. Technology will have to be regulated. Technology in the hands of greedy,
vested interests cannot be neutral. There is a law to prevent misuse and we must be able to use it,” she says.

On the ‘Demand’ side, experts such as Dr. Agnihotri argue that women’s participation in workforce, having disposable
incomes and making a contribution to the larger society will make a difference to how women are seen. Youth icons
and role models such as Sania Mirza are making an impact, he says.

Others feel there needs to be widespread visible contempt and anger in society against this ‘Genocide’- “the kind we
saw against the Nithari killings,” says Dr Bedi. “Today nobody can say that female foeticide is not their problem.” Time
we all did our bit to help save the girl child. Time’s running out.

21. What does the word ‘sanitised’ imply in the first paragraph of the passage?
(a) Unforgivable (b) Legitimate (c) Free from dirt (d) None of these

22. Which “demand” does the author refer to, in paragraph 5?


(a) Demand for principled doctors (b) Demand for high income jobs for women
(c) Demand for youth icons (d) Demand for sex determination and abortion

23. What is the doctor’s explanation for foeticide?


(a) They think it is legitimate
(b) They do it because people demand it
(c) The technology is available and there is no harm using it
(d) None of these

24. Which of the two people mentioned in the passage suggest similar solution to the problem?
(a) Dr.Agnihotri and Dr.George (b) Dr.Bedi and Dr.George
(c) Dr.George and Dr.Bedi (d) Dr.George and Ms.Sivadas

25. Which of the following will Dr. George agree to?


(a) The girl child is as safe in the mother's womb as after birth.
(b) The girl child is safer in the mother's womb in comparison to after birth.
(c) The girl child is safer after birth as compared to the mother's womb.
(d) None of these

26. What is Akhila Sivadas's opinion on the PCPNDT act?


(a) The act is inconsistent (b) The act needs reform
(c) The act encourages demand for foeticide (d) The act is sound, but needs enforcement

27. What is the tone of the passage?


(a) Factual (b) Biased (c) Aggressive (d) Sad

28. What is the solution to the problem of female foeticide as envisioned by Dr. Bedi?
(a) Effective use of law (b) Mass public outrage
(c) Comparison with Nithari killing (d) Contempt towards doctors

Page 10 of 91
VERBAL ABILITY

HAP_VA_BTECH_SEM_4_1218

SENTENCE CORRECTION – I
SESSION – 4

The 4 Steps for Sentence Correction


1. Take a First Glance
2. Read the Sentence
3. Find a Starting Point
4. Eliminate Answers

 Identify what SC concept is being tested.

For example, if the concept tested is parallelism, try to figure out what items should be in parallel. If it is modifiers,
identify the referent of each clause and their correct placement.

 Treat options A, B, C, D, E equally.

Even if you feel sure that a particular answer choice is right, don’t make up your mind till you have really looked at the
other options.

 Look for subject verb mismatches.

Subject Verb Agreement is one of the easiest errors to identify in any SC questions, at least 1 option can be eliminated
in this way. So look for these first.

Either my neighbour or her children is coming for dinner. ans: are

Neither they nor I were mistaken. ans: was

Errors in use of Pronouns:

e.g.

One must not reveal his secrets to all. ans: one’s secrets to all

- ‘one’ as a subject should use one’s.

 Don’t get confused by pronoun ambiguity.

Ambiguous pronouns are a problem only if the meaning of the sentence is affected because of the pronoun. Use this
concept only if all else fails.

Example 1

The common cold is one of our most indiscriminate diseases; it makes no distinction between you and me,
A B C D
millionaires and paupers, or athletes and couch potatoes. No error
E

Page 11 of 91
VERBAL ABILITY

HAP_VA_BTECH_SEM_4_1218

Step 1: Read the whole sentence

Reading this sentence, I don't notice anything jump out, although D sounds kind of weird. Even if that answer seems
tempting we can't pick it until we determine what the error is.

Step 2: Check each answer choice

First let's check B for errors, since it includes both a pronoun and verb. Make sure to confirm that the verb works.
Let's go through the possible errors for choice B one at a time.

Is the pronoun in the correct case? Yes. "It" is acting as subject for the verb "makes," which is perfectly acceptable.

Does the pronoun agree with its antecedent? Yes. "It" is standing in for "the common cold," which is a singular non-
gendered noun.

Is the verb correctly conjugated? Yes. "It" is a singular pronoun and "makes" is a singular verb.

Is the verb in the correct tense? Yes. It's in the present tense, which is consistent with the rest of the sentence .

Let's move on to choice D, "you and me," which also includes pronouns. This section sounds a little weird, but
remember that pronoun order doesn't matter—we only need to check agreement and case, like we did above.

Are the pronouns in the correct case? This question is a bit trickier. It would sound just fine to replace "me" with
"I"—"it makes no distinction between you and I"—but "between" is a preposition, so the object case "me" is correct.

Although D sounds odd, it doesn't contain an error. We can eliminate it and move onto C, which includes a noun and
preposition. We'll check the preposition first since it's more likely to include an error.

Is the preposition idiomatically correct? Yes. "Distinction between" is a common expression.

Does it incorrectly complete a word pair? No. "Between" is correctly followed by two prepositions connected by
"and."

Is the sentence consistent in its use of plural and singular nouns? Yes. There's no issue with agreement.

Since we didn't find any issues with C, we'll move onto checking the last answer, A, "one of our most."

Is the phrase idiomatically correct? Yes. "One of our most indiscriminate diseases" is a clear and acceptable phrase.

Is the superlative/comparative correct? Yes. The common cold is being compared to all of the other diseases, so
"most" is the correct word choice.

We can conclude that there is no error in A, so we've ruled out all the choices, leaving only E, "No error"

Step 3: Confirm what the error in your answer is

This sentence doesn't have an error, so the correct answer is E.

Example 2

The architect’s research shows that even when builders construct houses of stone, they still use the hammer
A B C
more than any tool. No error
D E

Page 12 of 91
VERBAL ABILITY

HAP_VA_BTECH_SEM_4_1218

Step 1: Read the whole sentence

Step 2: Check each answer choice

We'll start with A,

Is the verb correctly conjugated?

Is the verb in the correct tense

Is the preposition idiomatically correct?

Does it incorrectly complete a word pair?

Is the noun part of a faulty comparison?

Is the sentence consistent in its use of plural and singular nouns?

Is the modifier of the correct type? "Still" is describing when the builders use the hammer, so it's correct as an
adverb.

Are the comparative and superlatives [-er and –est] used appropriately?

Is the noun part of a faulty comparison?

That comparison seem fine, but it actually includes an error. A hammer is a tool, so it must be compared with any
other tool, rather than just "any tool." D is the answer.

Step 3: Confirm what the error in your answer is

Choice D is the answer because it creates an illogical comparison. We've also definitely ruled out the other choices
(except no error).

QUESTION TYPES:

I. SPOTTING ERRORS

II. SENTENCE CORRECTION

III. IDENTIFYING CORRECT AND INCORRECT USAGE

Directions for Q1 to Q5: In each sentence, certain words or phrases have been underlined and labelled (a), (b), (c) or
(d). Choose the underlined word or phrase that is incorrect in each sentence.

1. The (a) Washington Monument is the (b) tribute to the first and perhaps the greatest (c) president of the (d)
United States.

2. Muslims abstain from (a) food and drink during (b) Ramadan, the (c) ninth month of its (d) calendar.

3. The tomato is (a) an integral part of many (b) regional cuisines, and it was (c) popular since (d) the 1600’s.

4. The drainage (a) of swamps and other breeding (b) places of (c) mosquitoes have (d) greatly diminished the
incidence of malaria.

5. The shoulder (a) height of the Siberian mammoth, which (b) roamed throughout (c) the Northern Hemisphere,
was about nine foot (d).

Page 13 of 91
VERBAL ABILITY

HAP_VA_BTECH_SEM_4_1218

Directions for Q6 to Q10: Each sentence has been divided into four parts – (a), (b), (c) and (d). Identify which part
has an error.

6. After an outbreak of the so-called Mad Cow's disease, (a) / the United Kingdom was forced (b) / to quarantine
its beef exports (c) / and killing thousands of cows (d).

7. Police departments all over the world (a) / employs computers and other electronic equipment (b) / to capture
the criminals (c) / they pursue (d).

8. The novels of John Irving (a) / depict common everyday life (b) / in New Hampshire (c) / where he grown
up (d).

9. The Chinese, (a) / who are credited with (b) / the inventing of gunpowder, (c) / also created the kite and the
printing press (d).

10. In 1978, the atoll Enewetak was declared uninhabitable (a) / for at least 350 years (b) / because of the high
levels of radiation (c) / resulting U.S. nuclear testing (d).

Directions for Q11 to Q15: In each sentence below, a part has been given in bold. Choose the best answer option to
replace the bold part. If no correction or replacement is necessary, choose option (e).

11. The performance of our players was rather worst than I had expected.
(a) bad as I had expected (b) worse than I had expected (c) worse than expectation
(d) worst than was expected (e) No correction required

12. Why should the candidates be afraid of English Language is not clear.
(a) the candidates should be (b) do the candidates be (c) should be the candidates
(d) are the candidates (e) No correction required

13. Anyone interested in the use of computers can learn much if you have access to a personal computer.
(a) they have access (b) access can be available (c) he or she has access
(d) one of them have access (e) No correction required

14. The train will leave at 8.30 p.m. We have been ready by 7.30 p.m. so that we can reach the station in time.
(a) were (b) must be (c) are
(d) should have (e) No correction required

15. They examined both the samples very carefully but failed to detect any difference in them.
(a) some difference in (b) some difference between (c) any difference between
(d) any difference among (e) No correction required

Directions for Q16 to Q25: In each sentence below, a part has been underlined. Beneath the sentence you will find
five ways of phrasing the underlined part. The first of these repeats the original; the other four are different. If you
think the original is best, choose the first answer; otherwise choose one of the others.

16. George’s personal diary and album formed the basis for his book about the places he had visited.
(a) the basis for his book about the places he had visited.
(b) the basis for his book regarding the places he had visited.
(c) the basis of his book about the places he visited.
(d) the basis of his book about the places which he had visited.
(e) the basis of his book about the places he had visited.

Page 14 of 91
VERBAL ABILITY

HAP_VA_BTECH_SEM_4_1218

17. By next month Ms. Jones will be Mayor of Tallahassee for two years.
(a) will be Mayor of Tallahassee (b) will have been Mayor of Tallahassee
(c) will be mayor of Tallahassee (d) will have been mayor of Tallahassee
(e) could have been mayor of Tallahassee

18. If he would have revised his first draft, he would have received a better grade.
(a) would have revised (b) had revised (c) could of revised
(d) had of revised (e) would revise

19. Anu claims that cats made the best pets.


(a) made the best pets. (b) could be the best pets. (c) are the best pets.
(d) make of the best pets (e) make the best pets.

20. The government requires that these forms should be submitted before the end of the financial year.
(a) that these forms should be submitted (b) that these forms be submitted
(c) for these forms to be submitted (d) these forms submission
(e) these forms should be submitted

21. The reason why Sravanti wrote the letter was because she could not contact him over the phone.
(a) wrote the letter was because (b) wrote the letter was the reason that
(c) wrote the letter was that (d) had written the letter was because
(e) wrote the letter was since

22. After the heavy rains last week, the water in the reservoir raised another two feet.
(a) raised another two feet (b) risen another two feet (c) have risen by two feet
(d) would raise another two feet (e) rose another two feet

23. The drama enacted by the students had many scenes which were so humorous that it was hardly possible to
keeping a straight face.
(a) hardly possible to keeping (b) hardly impossible keeping (c) hardly possible to keep
(d) hardly impossible to keep (e) hardly possible for keeping

24. If you had attended the seminar you could benefit a great deal.
(a) could benefit (b) would have benefited (c) would benefit
(d) might benefit (e) should have benefit

25. Each group have their own ideology which it is bound to abide by.
(a) Each group have their own ideology (b) Every group has its own ideology
(c) Each group has their own ideologies (d) Each group has ideologies
(e) Every group have their own ideology

Directions for Q26 to Q28: Choose the CORRECT sentence from the four options given.

26. (a) India is a country with at least fifty major regional languages, of whom fourteen have official recognition.
(b) India is a country with at least fifty major regional languages, fourteen that have official recognition.
(c) India is a country with at least fifty major regional languages, fourteen of which are officially recognised.
(d) India is a country with at least fifty major regional languages, among whom fourteen have official
recognition.

Page 15 of 91
VERBAL ABILITY

HAP_VA_BTECH_SEM_4_1218

27. (a) Each of us was scheduled to take the test.


(b) Each of us were scheduled to take the test.
(c) Everyone of us were scheduled to take the test.
(d) Each of us are scheduled to take the test.

28. (a) Mukesh is the richest of the two brothers.


(b) You do not look well today.
(c) She spoke quicker than he did.
(d) I have never been more surer of anything in my life.

Directions for Q29 and Q30: Choose the INCORRECT sentence from the four options given.

29. (a) She looked suspicious at the man wearing the trench coat.
(b) She looked suspiciously at the man wearing the trench coat.
(c) The man wearing the trench coat looked suspiciously at her.
(d) The man wearing the trench coat looked suspicious to her.

30. (a) You may look out the window.


(b) Divide the candy between the two of you.
(c) That is something with which I cannot agree.
(d) Take your shoes off of the bed.

SENTENCE CORRECTION – II
SESSION – 5

Directions for Q1 to Q5: In each sentence, certain words or phrases have been underlined and labelled (a), (b), (c) or
(d). Choose the underlined word or phrase that is incorrect in each sentence.
1. In engineering (a), fatigue is the microscopic cracking (b) of materials, after repeating (c) applications of stress
(d).
2. In one of its (a) more useful functions in nature, the common housefly speeds up (b) the decomposing (c) of
dead (d) animals.
3. Magnolia (a) are native (b) to Japan, China, North America and the Himalayas, and they produce (c) large white,
rose or purple flowers (d).
4. The Pied Piper of Hamelin, the legendary figure of Hamelin, Germany, rid the town (a) of their (b) rats and mice
by leading them (c) away with his flute playing (d).
5. The pharynx is located on (a) the back of (b) the throat where (c) the mouth and nose meet (d) the esophagus.

Directions for Q6 to Q10: Each sentence has been divided into four parts – (a), (b), (c) and (d). Identify which part
has an error.
6. Medusa, the more famous (a) / of the three monstrous Gorgons, (b) / offended Athena (c) who changed
Medusa’s hair into snakes (d).
7. With the introduction of the new syllabus, (a) / the number of colleges reporting (b) / high results are
decreasing (c) / year after year (d).
8. The Maginot Line, (a) / a massive system of fortifications, (b) / became obsolete before (c) / their completion in
1934 (d).

Page 16 of 91
VERBAL ABILITY

HAP_VA_BTECH_SEM_4_1218

9. There are lesser buffalos (a) / roaming the Great Plains today (b) / than during the last century (c) / due to the
campaigns of white Americans against Native Americans (d).
10. The handles of the Big and Little Dippers (a) / extend in opposite directions, (b) / and when one was upright
(c) / the other is inverted. (d).

Directions for Q11 to Q15: In each sentence below, a part has been given in bold. Choose the best answer option to
replace the bold part. If no correction or replacement is necessary, choose option (e).

11. Making friends is more rewarding than to make enemies.


(a) to have enemies (b) enmity (c) get enemies
(d) making enemies (e) No correction required

12. There’s not so much work to do this week.


(a) many (b) plenty (c) less
(d) any (e) No correction required

13. Had I realised how close I was to the edge of the road, I would not have raised the speed of the car.
(a) Had I been realised (b) If I would have realized (c) When I realized
(d) Had I had realized (e) No correction required

14. They continued to work in the field despite of the heavy rains.
(a) even though there is heavy rain (b) although heavily rains (c) in spite the heavy rains
(d) even though it rained heavily (e) No correction required

15. Their earnings are such that they find it difficult to make both ends to meet.
(a) to makings both ends meet (b) to make both ends for meeting (c) to make both ends meet
(d) for making both ends to meet (e) No correction required

Directions for Q16 to Q25: In each sentence below, a part has been underlined. Beneath the sentence you will find
five ways of phrasing the underlined part. The first of these repeats the original; the other four are different. If you
think the original is best, choose the first answer; otherwise choose one of the others.

16. A conjunction is used to connect words and sentences together.


(a) words and sentences together. (b) words or sentences together.
(c) words and sentences. (d) words or sentences.
(e) words to sentences.

17. Citing the recent rise in violence against foreigners, the Embassy urged expats to remain vigilant, avoid the
Blackburn district, and refraining from travelling late at night.
(a) refraining from travelling late at night (b) travelling during the day instead of the night
(c) as an imperative, refrain from travel at night (d) as an imperative, refrain from travel at night
(e) refrain from travelling at night

18. The protestors campaigned against whale hunting, didn’t they?


(a) didn’t they (b) weren’t they (c) were they (d) isn’t it (e) wasn’t it

19. The sports writer questioned the skill of basketball players compared to tennis players.
(a) skill of basketball players compared to tennis players
(b) skills of basketball players compared to tennis players
(c) skill of basketball players to tennis players
(d) skill of basketball players compared to that of tennis players
(e) skill of basketball players compared to those of tennis players

Page 17 of 91
VERBAL ABILITY

HAP_VA_BTECH_SEM_4_1218

20. It is a special feature of cell aggregation in the developing nervous system that in most regions of the brain the
cells not only adhere to one another and also adopt some preferential orientation.
(a) to one another and also adopt (b) one to the other, and also they adopt
(c) one to the other, but also adopting (d) to one another but also adopt
(e) to each other, also adopting

21. The red cross organization appealed to the people to come forward to help the drought victims and said that
each may contribute what they can.
(a) each may contribute what they can (b) each might contribute what they can
(c) each should contribute what they could (d) each may contribute what he can
(e) each may contribute what each one can

22. Unlike the brown sparrow, the passenger pigeon was slaughtered indiscriminately and became extinct in 1914.
(a) was slaughtered indiscriminately (b) did not slaughter indiscriminately
(c) had been slaughtered indiscriminately (d) slaughtered indiscriminately
(e) has been slaughtered indiscriminately

23. Because zebras have very striped patterns, photos can be used to identify them when captured.
(a) very striped patterns (b) varying stripe patterns (c) varied striped patterns
(d) various stripe pattern (e) striped pattern in variety

24. Shyamala had not scarcely entered when Rama shouted.


(a) not scarcely entered when (b) scarcely entered than (c) not hardly entered when
(d) scarcely enters (e) scarcely entered when

25. Between these two shirts which is the least costly?


(a) which is the least costly (b) which one is the least costly (c) which is least costly one
(d) which is less costly (e) which one less cost

Directions for Q26 and Q27: Choose the CORRECT sentence from the four options given.

26. (a) As the invitation stated, we'll see you the tenth for our reunion.
(b) Like the invitation stated, we'll see you on the tenth for our reunion.
(c) As the invitation stated, we'll see you on the tenth for our reunion.
(d) Like the invitation stated, we'll see you the tenth for our reunion.

27. (a) I prefer coffee over tea.


(b) Stella McCartney is regarded one of the greatest designers of today.
(c) If you have any more questions, you should refer to a grammar book.
(d) He did well in the test, except the sentence construction questions.

Directions for Q28 to Q30: Choose the INCORRECT sentence from the four options given.

28. (a) I wonder what he could have said to her.


(b) Anita and she have quit the team.
(c) You didn't tell us that they were here first.
(d) My brother and myself constructed the fence.

29. (a) Nora is one of the candidates who are worthy of my vote.
(b) My problem, which is minor in comparison with others, exist because I failed in the test.
(c) If our staff members don't quit picking at each other, we will not meet our goals.
(d) At the end of the story, they were living happily ever after.

Page 18 of 91
VERBAL ABILITY

HAP_VA_BTECH_SEM_4_1218

30. (a) Eighty percent of the class have passed the test.
(b) Either Gary or I am responsible for allocating the funds.
(c) None of the garbage was picked up.
(d) None of the sentences were punctuated correctly.

SPOT THE ERROR – I


SESSION – 6

TIPS FOR ANSWERING ‘SPOT THE ERROR’

1. Number of: With the word ‘number’, we sometimes use a single verb and sometimes a plural.
A number of + plural verb
A number of is used with a plural countable noun and a plural verb.
E.g.: A number of people have complained about the price increase.
The number of + singular verb
E.g.: It’s a popular school and the number of students has risen again this year.
The numbers of + plural verb
E.g.: The numbers of students are rising steadily.

2. Supposing and if cannot be used together in the same sentence.


E.g.: Supposing if he fails, what will he do? 
If he fails, what will he do? 

3. Both and as well as cannot be used together in the same sentence because both convey the same sense.
E.g.: Both Preeti as well as Pooja are beautiful 
Both Preeti and Pooja are beautiful. 

4. Avoid using the double comparative and double superlative.


E.g.: This car is the most costliest in the market. 
This car is the costliest in the market. 

5. It should be members of the family and not family members.


E.g.: The members of his family have gone to Bangalore.

6. Else is always followed by but and never by than.


E.g.: It is nothing else but your pride which makes you such a thing.

7. The phrase type of, sort of, and kind of do not follow a or an.
E.g.: What kind of friend are you?

8. Little, a little, the little


Little has practically negative meaning.
E.g.: There is little chance of getting the money.

Page 19 of 91
VERBAL ABILITY

HAP_VA_BTECH_SEM_4_1218

A little means some, though, not much. It has a positive meaning.


E.g.: There is a little water in the pot.
The little means not much, but all that is there.
E.g.: The little money we had, helped us to spend the day.

9. Few, a few, the few


Few means not many, hardly any. It has a negative meaning.
E.g.: Few people can speak correct English.
A few means ‘some’. It has a positive meaning.
E.g.: A few people can speak correct English.
The few means not many but all that there are.
E.g.: The few books we had were very interesting.

10. Too … to has got a negative meaning.


E.g.: The boy was too young to understand this.
The tea was very hot. I couldn’t drink it.
The tea was too hot for me to drink.

11. So … that
The structure so … that can be used to show cause and effect.
E.g.: She was so tired that she could not walk

12. ‘Many a’ is always followed by the singular verb.


E.g.: Many a man were drowned in the sea. 
Many a man was drowned in the sea. 

13. ‘A great many’ is always followed by a plural noun and plural verb.
E.g.: A great many students have been declared successful.

14. Unless _________ a condition. Used in negative sense. Never use ‘not’ with unless. It means ‘if …. not’
E.g.: If Raja does not work hard, he will not pass.
Unless Raja works hard, he will not pass.

15. The infinitive without ‘to’ is used after ‘had better, had rather, would rather, sooner … than, rather than’.
E.g.: You had better ask permission from him.

16. Some common errors:

He told the truth. 


He spoke the truth. 

Do not speak a lie. 


Do not tell a lie. 

I look forward to meet you. 


I look forward to meeting you. 

Page 20 of 91
VERBAL ABILITY

HAP_VA_BTECH_SEM_4_1218

17. Redundancy: Use of two or more words that mean the same.
Return back my book.  (Return and back mean the same.)
Return my book. 

18. Always use preposition ‘to’ with inferior, superior, junior, senior
E.g.: Sam is junior to Ram.

19. Fewer/less
Fewer refers to number and less refers to quantity and size or amount.
E.g.: No fewer than fifty passengers were killed in the accident.
Women now spend less time in the kitchen.

20. Avoid double negatives. Words like ‘hardly, scarcely etc’ are adverbs that are negative in meaning. No other
negative words should be used with them.
E.g.: The little house in which he lived was hardly bigger than a hut.

21. ‘WITH A VIEW TO’ is always followed by gerund and not an adjective.
E.g.: A group of banks financed the deal, with a view to selling the debt to investors.

Directions for Q1 to Q30: In the following exercise, each sentence is divided into four parts, i.e. (a), (b), (c) and (d).
One of these parts contains an error. Pick out the part that has an error. If the sentence is free from error, then your
answer will be (e).

1. I no longer has to worry (a)/ about rushing out to buy groceries or vegetables (b)/ at the last minute or stand in
a line (c)/ to get the tickets to the latest blockbuster. (d)/ No error (e)

2. The streets (a)/ are so wet (b)/ it should have (c)/ rained last night. (d)/ No error (e)

3. The wounded men (a)/ was being (b)/ helped by (c)/ some boys. (d)/ No error (e)

4. I found this toy (a)/ while digging (b)/ in the garden, but (c)/ I don’t know to whom it belonged to. (d)/ No
error (e)

5. Qasim tells (a)/ the interviewer, (b)/ “I have got my (c)/ MBA degree in 2010”. (d)/ No error (e)

6. He fell from the running train (a)/ and would have died (b)/ if the villagers did not get (c)/ him admitted in the
nearby hospital immediately. (d)/ No error (e)

7. It being a sunny day, (a)/ the girls will decide to (b)/ go on a picnic to the beach and (c)/ indulge in some water
sports. (d)/ No error (e)

8. Wearing extremely fashionable (a)/ and surrounded by photographers (b)/ and press operators (c)/ she swept
up the microphone. (d)/ No error (e)

9. As soon as the telegram (a)/ reached Nanda’s house that her (b)/ family set out to rescue (c)/ her from the
situation. (d)/ No error (e)

10. He obtained good marks (a)/ not only in English essay (b)/ but also in Arithmetic which was (c)/ full of
complicated calculations. (d)/ No error (e)

Page 21 of 91
VERBAL ABILITY

HAP_VA_BTECH_SEM_4_1218

11. The Hindi and Marathi are (a)/ different forms of the Sanskrit language (b)/ which were once spoken (c)/ in
almost every part of India. (d)/ No error (e)

12. As much as I admire him (a)/ for his sterling qualities (b)/ I cannot excuse him (c)/ for being unfair to his
friends. (d)/ No error (e)

13. Several prominent figures (a)/ involved in the scandal (b)/ are required to appear (c)/ to the investigation
committee. (d)/ No error (e)

14. The RBI has proposed to introduce (a)/ polymer notes after taking into considering (b)/ the cost and longevity
(c)/ associated with their manufacturing. (d)/ No error (e)

15. The lecture to be held (a)/ in the auditorium tomorrow (b)/ was open to the public (c)/ and free of charge.
(d)/ No error (e)

16. The speech he made (a)/ on his visit to our college (b)/ concerning our society’s needs (c)/ have bothered me
greatly. (d)/ No error (e)

17. It was an evening of (a)/ fun and frolic as the city’s glitterati (b)/ and high-profile businessmen made sure
(c)/ that they didn’t miss the show. (d)/ No error (e)

18. Human babies whose mothers (a)/ have had the flu while pregnant may (b)/ have a greater risk of developing
(c)/ schizophrenia lately in life. (d)/ No error (e)

19. Political education in real sense (a)/ should develop people (b)/ who will be committed to (c)/ bring social
change. (d)/ No error (e)

20. He managed to board (a)/ the running train (b)/ but all his luggages was (c)/ left on the station. (d)/ No error
(e)

21. Gaze for a thing (a)/ that are not (b)/ available easily (c)/ in the country is a universal phenomenon. (d)/ No
error (e)

22. Taken into consideration (a)/ the advice of his (b)/ colleagues he decided (c)/ to stay in the institute. (d)/ No
error (e)

23. He cannot withdraw (a)/ all his money (b)/ unless he do not (c)/ gives advance notice. (d)/ No error (e)

24. Kumar said in the (a)/ conference that he would (b)/ train the new team (c)/ from tomorrow. (d)/ No error (e)

25. A customized data science degree (a)/ is yet to become (b)/ a standard programme (c)/ to India’s premier
educational institutes. (d)/ No error (e)

26. Tax evaders should (a)/ be heavily fined (b)/ as they are doing (c)/ it intentionally. (d)/ No error (e)

27. My parents (a)/ felt awful (b)/ as they realized (c)/ they had cheated. (d)/ No error (e)

28. Many a good student is (a)/ suffering because (b)/ of addiction (c)/ to drugs. (d)/ No error (e)

29. There cannot be any situation where (a)/ somebody makes money in an asset (b)/ located in India and does
not pay tax (c)/ either to India or to the country of his origin. (d)/ No error (e)

30. My parents backed me down (a)/ when I decided to leave (b)/ college to work (c)/ and support my family. (d)/
No error (e)

Page 22 of 91
VERBAL ABILITY

HAP_VA_BTECH_SEM_4_1218

SPOT THE ERROR – II


SESSION – 7

Directions for Q1 to Q30: In the following exercise, each sentence is divided into four parts, i.e. (a), (b), (c) and (d).
One of these parts contains an error. Pick out the part that has an error. If the sentence is free from error, then your
answer will be (e).

1. Cottage industry, also (a) known as the putting-out system, was the initial phase of industrialization, (b)
although it was soon overshadowed by other systems of production (c) that (d) have become widespread
during the Industrial Revolution of the 19th century. (e) No error

2. The flower, after (a) fertilization (b) with the grains of pollen that (c) a bee inadvertently transports, will slowly
(d) perish and wither. (e) No error

3. Art and culture flourished (a) briefly during the “Prague Spring,” (b) a short period (c) when relaxed censorship
and (d) looser Soviet control. (e) No error

4. (a) Having died young, Raymond Carver’s career as a short-story writer was cut regrettably (b) short, and (c)
we are left with only a fragment of his only (d) attempt at a novel. (e) No error

5. The defendant (a) surprised the jury not only with his candor (b) as well as with (c) what seemed to be genuine
goodwill (d) behind his smile. (e) No error

6. Exotic pet enthusiasts prize male peacocks (a) for their feathers (b) because they are the (c) more colorful of
the two (d) sexes; in contrast, female peacocks are mostly for breeding purposes. (e) No error

7. Seventy-five percent of what we perceive (a) as taste, (b) especially the perception of flavor, is really
attributable (c) to one's sense (d) of smell. (e) No error

8. Development of new drugs (a) often require years (b) of testing and waiting (c) for labeling approval (d) from
federal regulators. (e) No error

9. Although research in the earth and environmental sciences has pieced together narratives of ancient and
historical environmental changes, (a) but there is still (b) much left (c) to learn about the nature and causes of
(d) changing climatic conditions through time. (e) No error

10. (a) During the reign of Elizabeth I, dozens of commissions were sent out by the central government (b) to
investigate the wood shortage around the (c) nation, each one confirmed the (d) serious decline of the forests.
(e) No error

11. (a) While sperm whales (b) are usually seen in offshore waters, they can also be found near the shore, (c)
where the continental or island shelf is narrow (d) and with the water is deep. (e) No error

12. A farmer's field can have more than 50,000 weed seeds (a) per square meter buried beneath the (b) soil surface
seeds buried more than about one centimeter (c) below the soil surface do not receive (d) enough light to
germinate. (e) No error

13. In 1952, when the Federal Communications Commission lifted its ban on new television stations, (a) there were
105 stations and 15 million households with television sets, (b) but by 1956 the number of stations (c) had
grown to almost 500 and the number of households with sets (d) at nearly 35 million. (e) No error

Page 23 of 91
VERBAL ABILITY

HAP_VA_BTECH_SEM_4_1218

14. Halley's comet is a potato-shaped lump (a) about ten miles long (b) with a mass (c) estimated at 10 billion tons,
(d) most of which is water ice. (e) No error

15. The Neuse River in North Carolina is (a) thought to be about two million years old, and there (b) has likely been
human settlements in the basin (c) for (d) at least 14,000 years. (e) No error

16. (a) Shortly after Queen Eleanor arrived in Nottingham in 1257 (b) for a visit , she fled the town (c) because she
could not stand the smell of coal (d) smoke and was afraid and feared for her health. (e) No error

17. The great white shark is the (a) more infamous (b) of all sharks, the (c) star of Peter Benchley's bestselling
novel Jaws, four Hollywood movies, and (d) many television specials. (e) No error

18. The San Andreas Fault in California is (a) just one section (b) to an active circle of earthquake zones and
volcanoes (c) known as the "Ring of Fire" that (d) surrounds the Pacific Ocean. (e) No error

19. Relying on dams for large-scale water storage and (a) for delivering water to places (b) where it does not
naturally occur (c) have long-term effects on the balance (d) between groundwater and surface water and on
the quality of the surrounding soil. (e) No error

20. (a) Despite a protest (b) by 300 angry drivers, the five members of the bridge authority (c) voted unanimously
(d) to raise the passenger-car toll on the bridge from $1.00 to $2.00. (e) No error

21. Marking the fifth consecutive month of softening retail (a)/ inflation measured by CPI turned (b)/ down more
sharper than expected in December (c)/ and reached its lowest reading since November 2014. (d)/ No error (e)

22. It has become the norm for rich and powerful (a)/ to get bail with ease while the commoner (b)/ and the poor
languish in jail, the Law Commission (c)/ of India said in its latest report. (d)/ No error (e)

23. The minister insisted on the need for improve (a)/the railway safety saying India (b)/needs to invest heavily in
railway infrastructure (c)/to meet the decades-old backlog. (d)/ No error (e)

24. Vardhini of RBI very nearly lost the match (a)/ and failed to qualify herself for the main draw (b)/ of the LIC-
South Zone National-ranking (c)/ table tennis tournament at the Indoor Stadium. (d)/ No error (e)

25. The Climate Bond Initiative, in its (a)/ India update, indicated that about (b)/ 62 percent of the green bond
proceeds has been (c)/ allocated to renewable energy projects. (d)/ No error (e)

26. On July 4, North Korea tested (a)/ Hwasong-14, described as (b)/ an intercontinental ballistic missile capable in
(c)/ reaching the U.S. mainland. (d)/ No error (e)

27. Pinning high hopes (a)/ on either of them (b)/ will only lead to (c)/ disappointments in the future. (d)/ No
error (e)

28. More than one (a)/ student have (b)/ taken the interview (c)/ for this company. (d)/ No Error (e)

29. The Manager says that (a)/ there is (b)/ many important details (c)/ to attend before this gets launched. (d)/
No Error (e)

30. The science project (a)/ was equally successful as (b)/ the software project (c)/ they had presented. (d)/ No
Error (e)

Page 24 of 91
VERBAL ABILITY

HAP_VA_BTECH_SEM_4_1218

SENTENCE IMPROVISATION
SESSION – 8

Directions for Q1 to Q30: In the following questions, a part of the sentence is underlined. Below are given
alternatives to the underlined part which may improve the sentence. Choose the correct alternative. In case no
improvement is required, choose ‘No improvement’.

1. A lot of people has left this place.


(a) has been left this place (b) have left this place
(c) will be left this place (d) were left this place
(e) No improvement

2. This opportunity is a lifelong opportunity for me.


(a) is a lifetime opportunity (b) is being a lifelong opportunity
(c) shall be a lifetime opportunity (d) is a life lasting opportunity
(e) No improvement

3. If the books have been catalogued last week, why haven't they been placed on the shelf?
(a) have been catalogued (b) would have been catalogued
(c) was catalogued (d) were catalogued
(e) No improvement

4. We stood in line for three hours.


(a) from three hours (b) since three hours
(c) upto three hours (d) in three hours
(e) No improvement

5. Jessica Mitford wrote The American Way of Death, a best-selling book that led eventually to an official
investigation of the funeral industry.
(a) that led eventually (b) that had led eventually
(c) that eventually led (d) which eventually led
(e) No improvement

6. In order to increase their profits, many spice producing companies have been engaging themselves in
propaganda against commonly used spices and promoting exotic and expensive varieties of spices as more
healthier options.
(a) as healthier options (b) many healthier options
(c) mostly healthy options (d) less healthy options
(e) No improvement

7. The setting of a story effects the story's plot.


(a) effects the story's plot (b) effects the stories plot
(c) affect the story's plot (d) affects the story's plot
(e) No improvement

8. Charlie would have won the series if he would have played strategically.
(a) could play strategically (b) had played strategically
(c) did played strategically (d) have been playing strategically
(e) No improvement

Page 25 of 91
VERBAL ABILITY

HAP_VA_BTECH_SEM_4_1218

9. The use of radar as well as two-way radio paging makes it possible for state police to intercept most speeders.
(a) eliminates the impossibility (b) allows the possibility
(c) makes possible (d) facilitates the possibility
(e) No improvement

10. The Indians hoped to immediately see the result of the new government’s trade policy.
(a) hoped to immediately saw (b) hoped to immediately seen
(c) hoped to see immediately (d) hoped to immediately seeing
(e) No improvement

11. Farmers in our country suffer because of an over dependence on rain and the threat to natural calamities like
floods.
(a) threat of natural calamities (b) threat against natural calamities
(c) threatened to natural calamity (d) natural calamities threatened
(e) No improvement

12. Though it was a race against time, they managed to fill and submit their examination forms.
(a) race for time (b) race in time (c) race before time
(d) race with time (e) No improvement

13. One view of the economy contends that a large drop in oil prices should eventually lead to lowering interest
rates, as well as lowering fears about inflation, a rally in stocks and bonds, and a weakening of the dollar.
(a) lowering interest rates, as well as lowering fears about inflation,
(b) a lowering of interest rates and of fears about inflation,
(c) a lowering of interest rates, along with fears about inflation,
(d) interest rates being lowered, along with fears about inflation,
(e) interest rates and fears about inflation being lowered, with

14. After the Civil War, contemporaries of Harriet Tubman's maintained that she has all of the qualities of a great
leader: coolness in the face of danger, an excellent sense of strategy, and an ability to plan in minute detail.
(a) Tubman's maintained that she has (b) Tubman's maintain that she had
(c) Tubman's have maintained that she had (d) Tubman maintained that she had
(e) Tubman had maintained that she has

15. From 1982 to 1987 sales of new small boats increased between five and ten percent annually.
(a) From 1982 to 1987 sales of new small boats increased between five and ten percent annually.
(b) Five to ten percent is the annual increase in sales of new small boats in the years 1982 to 1987.
(c) Sales of new small boats have increased annually five and ten percent in the years 1982 to 1987.
(d) Annually an increase of five to ten percent has occurred between 1982 and 1987 in the sales of new small
boats.
(e) Occurring from 1982 to 1987 was an annual increase of five and ten percent in the sales of new small boats.

Page 26 of 91
VERBAL ABILITY

HAP_VA_BTECH_SEM_4_1218

16. In recent years cattle breeders have increasingly used crossbreeding, in part that their steers should acquire
certain characteristics and partly because crossbreeding is said to provide hybrid vigor.
(a) in part that their steers should acquire certain characteristics
(b) in part for the acquisition of certain characteristics in their steers
(c) partly because of their steers acquiring certain characteristics
(d) partly because certain characteristics should be acquired by their steers
(e) partly to acquire certain characteristics in their steers

17. The peaks of a mountain range, acting like rocks in a streambed, produce ripples in the air flowing over them;
the resulting flow pattern, with crests and troughs that remain stationary although the air that forms them is
moving rapidly, are known as "standing waves."
(a) crests and troughs that remain stationary although the air that forms them is moving rapidly, are
(b) crests and troughs that remain stationary although they are formed by rapidly moving air, are
(c) crests and troughs that remain stationary although the air that forms them is moving rapidly, is
(d) stationary crests and troughs although the air that forms them is moving rapidly, are
(e) stationary crests and troughs although they are formed by rapidly moving air, is

18. Like Auden. the language of James Merrill is chatty, arch, and conversational--given to complex syntactic flights
as well as to prosaic free-verse strolls.
(a) Like Auden, the language of James Merrill (b) Like Auden, James Merrill's language
(c) Like Auden's, James Merrill's language (d) As with Auden, James Merrill's language
(e) As is Auden's the language of James Merrill

19. In the textbook publishing business, the second quarter is historically weak, because revenues are low and
marketing expenses are high as companies prepare for the coming school year.
(a) low and marketing expenses are high as companies prepare
(b) low and their marketing expenses are high as they prepare
(c) low with higher marketing expenses in preparation
(d) low, while marketing expenses are higher to prepare
(e) low, while their marketing expenses are higher in preparation

20. Teratomas are unusual forms of cancer because they are composed of tissues such as tooth and bone not
normally found in the organ in which the tumor appears.
(a) because they are composed of tissues such as tooth and bone
(b) because they are composed of tissues like tooth and bone that are
(c) because they are composed of tissues, like tooth and bone, tissues
(d) in that their composition, tissues such as tooth and bone, is
(e) in that they are composed of tissues such as tooth and bone, tissues

21. The Senate approved immigration legislation that would grant permanent residency to millions of aliens
currently residing here and if employers hired illegal aliens they would be penalized.
(a) if employers hired illegal aliens they would be penalized
(b) hiring illegal aliens would be a penalty for employers
(c) penalize employers who hire illegal aliens
(d) penalizing employers hiring illegal aliens
(e) employers to be penalized for hiring illegal aliens

Page 27 of 91
VERBAL ABILITY

HAP_VA_BTECH_SEM_4_1218

22. Scientists have recently discovered what could be the largest and oldest living organism on Earth, a giant
fungus that is an interwoven filigree of mushrooms and rootlike tentacles spawned by a single fertilized spore
some 10,000 years ago and extending for more than 30 acres in the soil of a Michigan forest.
(a) extending (b) extends (c) extended
(d) it extended (e) is extending

23. The period when the great painted caves at Lascaux and Altamira were occupied by Upper Paleolithic people
has been established by carbon-14 dating, but what is much more difficult to determine are the reason for their
decoration, the use to which primitive people put the caves, and the meaning of the magnificently depicted
animals.
(a) has been established by carbon-14 dating, but what is much more difficult to determine are
(b) has been established by carbon-14 dating, but what is much more difficult to determine is
(c) have been established by carbon-14 dating, but what is much more difficult to determine is
(d) have been established by carbon-14 dating, but what is much more difficult to determine are
(e) are established by carbon-14 dating, but that which is much more difficult to determine is

24. The Baldrick Manufacturing Company has for several years followed a policy aimed at decreasing operating
costs and improving the efficiency of its distribution system.
(a) aimed at decreasing operating costs and improving
(b) aimed at the decreasing of operating costs and to improve
(c) aiming at the decreasing of operating costs and improving
(d) the aim of which is the decreasing of operating costs and improving
(e) with the aim to decrease operating costs and to improve

25. The Federalist papers, a strong defense of the United States Constitution and important as a body of work in
political science as well represents the handiwork of three different authors.
(a) and important as a body of work in political science as well, represents
(b) as well as an important body of work in political science, represent
(c) and also a body of work of importance in political science is representing
(d) an important body of work in political science and has been representative of
(e) and as political science an important body of work too, represent

26. Although the term "psychopath" is popularly applied to an especially brutal criminal, in psychology it is
someone who is apparently incapable of feeling compassion or the pangs of conscience.
(a) it is someone who is (b) it is a person
(c) they are people who are (d) it refers to someone who is
(e) it is in reference to people

27. Parliament did not accord full refugee benefits to twelve of the recent immigrants because it believed that to do
it rewards them for entering the country illegally.
(a) to do it rewards (b) doing it rewards
(c) to do this would reward (d) doing so would reward
(e) to do it would reward

Page 28 of 91
VERBAL ABILITY

HAP_VA_BTECH_SEM_4_1218

28. Many policy experts say that shifting a portion of health-benefit costs back to the workers helps to control the
employer's costs, but also helps to limit medical spending by making patients more careful consumers.
(a) helps to control the employer's costs, but also helps
(b) helps the control of the employer's costs, and also
(c) not only helps to control the employer's costs, but also helps
(d) helps to control not only the employer's costs, but
(e) not only helps to control the employer's costs, and also helps

29. The plot of The Bostonians centers on the rivalry between Olive Chancellor, an active feminist, with her
charming and cynical cousin. Basil Ransom, when they find themselves drawn to the same radiant young
woman whose talent for public speaking has won her an ardent following.
(a) rivalry between Olive Chancellor, an active feminist, with her charming and cynical cousin, Basil Ransom
(b) rivals Olive Chancellor, an active feminist, against her charming and cynical cousin, Basil Ransom
(c) rivalry that develops between Olive Chancellor, an active feminist, and Basil Ransom, her charming and
cynical cousin
(d) developing rivalry between Olive Chancellor, an active feminist, with Basil Ransom, her charming and
cynical cousin
(e) active feminist. Olive Chancellor, and the rivalry with her charming and cynical cousin Basil Ransom

30. A powder derived from the North American Echinacea flower, which has been effective in preventing colds, is
grown by many small farmers out West.
(a) A powder derived from the North American Echinacea flower, which has been effective in preventing colds,
(b) A derivative, which has been effective in preventing colds, of the North American Echinacea flower
(c) A North American Echinacea flower derivative, which has been effective in preventing colds
(d) The North American Echinacea flower has a derivative which has been effective in preventing colds, that
(e) The North American Echinacea flower, a derivative of which has been effective in preventing colds,

PARA-JUMBLES – I
SESSION – 9

1. What are para jumbles?


Para jumbles are, as the name suggests, jumbled paragraphs. You are expected to rearrange the sentences so that they
make logical sense.

Solving para jumbles involves a strong grasp of context, language, grammar and more importantly, it requires
common sense.

2. Question Types
Most para jumble questions are of two types:
1. Moving jumbles: This is the plain vanilla version where a bunch of sentences are jumbled and given to you. And
the answer is a certain combination of these sentences.
2. Anchor jumbles: In an anchor jumble, the first and the last sentence are given. And the sentences in between
are in random order. The answer is again, a specific combination of these sentences.

Page 29 of 91
VERBAL ABILITY

HAP_VA_BTECH_SEM_4_1218

3. Approach Methodology
The different methodologies that can be adopted for solving para jumbles are:
(a) Acronym Approach - full form vs. short form
(b) Time Sequence Approach – TSA - either dates or time sequence indicating words
(c) Examples Approach – EA - after an hypothesis or theory
(d) Articles - definite and indefinite
(e) Noun, Pronoun, and Demonstrative Adjective - NPDA Approach - limited to not just noun
(f) Opening - Closing Sentence Approach - OCSA - supported or free, general or need previous explanation
(g) Key Words Approach - KWA - words repeated in two consecutive sentences
(h) Look for the Conjunctions that play a very important role in joining two or more sentences. Such as
after, although, as far as, as long as, as soon as, because, before, if, since, so, though, unless, until, when,
whenever, whereas etc.

4. What to see while doing para jumbles


 Even the toughest para jumbles have two or three sentences that form a link. Try to spot a link that has to
go together and then eliminate the options that do not have that link. This is the first round of elimination.
 The coordinating conjunctions are also very important. Coordinating conjunctions are: for, and, nor, but,
or, yet and so.
 Keep an eye on Adverbs like even, also, still, however, nevertheless, notwithstanding etc. Such adverbs
help link two or more sentences.
 Use the pronouns to your advantage. Ideally a pronoun in one sentence should always point to a noun in
the sentence immediately preceding it.
 General information always precedes specific information. That is the thumb rule. Information/idea that is
more universal should come first followed by information/idea that is less universal or more specific.
 Information of an idea/entity comes first followed by general description. Tangible descriptions should
always take precedence over intangible descriptions. If you describe a man’s personality, describe his
physical features and then move to his intangible aspects, same rule applying to other things.
 The conclusion as a rule, always comes at the end. Any sentence that follows a conclusion creates a break
in the flow of ideas, a feeling of this awkwardness should help you in finding the last sentence of the
paragraph.
 Keep an eye on the chronology and the importance of events. An event of lesser significance comes later
but an event of greater significance always comes first.
 Lastly never rush with your answers. Be patient. Mark the answer only after you have confidently evaluated
the other options.

5. Establishing link between two sentences and then examining the options

Suppose you establish the link 'BA'. The given options are:
(a) DABC (b) ACDB (c) CBAD (d) DBAC.

Now you are left with option (c) and (d) to examine. You read the sentences in the order given by these two options
and use your methods again to determine which one is correct.

Is establishing links between two sentences easy?

Page 30 of 91
VERBAL ABILITY

HAP_VA_BTECH_SEM_4_1218

Directions for Q1 to Q20: Each question has a number of sentences which, when properly sequenced, form a
coherent paragraph. Each sentence is labeled with a letter. Choose the most logical order of sentences from among the
given choices to construct a coherent paragraph.

1. A. Tigers have been known to eat up to 60 pounds (27 kilograms) of meat in one night, but more often they
consume about 12 pounds (5 kilograms) during a meal.
B. It may take days for a tiger to finish eating its kill. The cat eats until it's full, and then covers the carcass with
leaves and dirt.
C. When it's hungry again, the tiger comes back to feed some more, until the meat is gone.
(a) ABC (b) BAC (c) BCA (d) None of these

2. A. Reasons include demand for tiger fur as well as for other body parts that many people use in traditional
medicines.
B. Tigers' habitat has also dwindled seriously as humans have developed land for uses such as farming and logging.
C. The whole tiger species is endangered throughout its range.
(a) ABC (b) BAC (c) CAB (d) None of these

3. A. Connie St Louis, who wrote the original account of Sir Tim’s speech which prompted the row that led him to
resign from his posts at University College London and the Royal Society, told The London Evening Standard
she has received “unpleasant” emails and tweets from his supporters.
B. The journalist who accused Nobel Prize-winning scientist Sir Tim Hunt of sexism said she has been the
victim of Twitter trolls and accused the “establishment” of “bullying” and “protecting their own”.
C. Ms St Louis, a lecturer in science journalism at City University, added: “It doesn’t make me change my mind.
I am standing by my story.”
(a) ABC (b) CAB (c) BCA (d) BAC

4. A. Hamlet is an enigma.
B. His challenge to Guildenstern rings true for everyone who seeks to know him: "You would pluck out the
heart of my mystery."
C. None of us ever really does.
D. No matter how many ways critics examine him, no absolute truth emerges.
(a) ABCD (b) ADBC (c) BCAD (d) None of these

5. A. The results, presented on Tuesday at the Alzheimer’s Society annual research conference in Manchester,
have been hailed as “hugely promising” because they involve medicines that are already known to be safe
and well-tolerated in people – potentially cutting years from the timeline for drugs to reach patients.
B. Speaking ahead of her presentation, Giovanna Mallucci, professor of clinical neuroscience at the University
of Cambridge, said: “It’s really exciting. They’re licensed drugs.
C. This means you’d do a straightforward basic clinical trial on a small group of patients because these are not
new compounds, they’re known drugs.”
D. Two licensed drugs have been shown to halt brain degeneration in mice, raising the prospect of a rapid
acceleration in the search for a medicine to beat Alzheimer’s disease.
(a) ACBD (b) BDAC (c) CDAB (d) DABC

6. A. The latest research suggests that the build-up of abnormal amyloid proteins seen in Alzheimer’s,
Parkinson’s and CJD are simply a visible symptom of neuro-degeneration, but not the root cause.
B. Instead, Prof Mallucci argues, the damage really occurs when a natural defence mechanism in the brain
responds to the accumulation of plaques by switching off the production of all new proteins, wrecking the
brain’s ability to carry out essential repairs.

Page 31 of 91
VERBAL ABILITY

HAP_VA_BTECH_SEM_4_1218

C. In mice with prion disease, a neurodegenerative disorder that scientists use as a model for dementia in
mice, both treatments were shown to restore protein production, stop brain cells from dying off and prevent
memory loss.
D. The drugs work by inhibiting an enzyme, called Perk that activates the defence mechanism.
(a) ABCD (b) BACD (c) DCBA (d) CABD

7. A. Now, you can get a keyring-friendly mini-pack, meaning that it can be on hand at all times.
B. It may be derided as a modish condiment, but it is extremely tasty, and sparks the kind of devotion that prompts
fans of the bright red bottle to buy T-shirts with it on the front and say things like: “Oh, I have it with everything.”
C. So, I decided to spend a day having it with everything.
D. Sriracha hot sauce, which originated in eastern Thailand, is a mix of garlic, sugar, chillis, vinegar and salt.
(a) ABCD (b) DBAC (c) CADB (d) BDAC

8. A. And dropped out after the first year.


B. Deep down I knew that what I really wanted to do was to write stories.
C. When I was growing up in Nigeria I was expected, as every student who did well was expected, to become a
doctor.
D. But I did what I was supposed to do and I went into medical school.
(a) CBDA (b) ACBD (c) DCBA (d) BCDA

9. A. This is because at 11 p.m. on Tuesday night Greece failed to make a €1.6bn payment due to the International
Monetary Fund, one of the country’s creditors, thus becoming the first developed nation to go into “arrears”
with the fund.
B. The effective sovereign default has ratcheted up pressure on the ECB to further restrict the funding
available to banks, which have been swapping Greek government bonds for funding from the Frankfurt-
based central bank.
C. Greece’s banks have been shut since 29 June, after Mr Tsipras’ surprise decision to hold a referendum
prompted the European Central Bank (ECB) to limit the emergency lending available.
D. The ECB decided to maintain the current funding limit.
(a) CABD (b) BADC (c) DCBA (d) CBDA

10. A. "My opinion is that we need to invest for something to grow, whether that is a person, a company or a
nation like Greece," Feeney explained to i100.co.uk.
B. His reasoning: there are 503 million people in the EU. We would only need to chip in about €3 each (the
same price as a feta and olive salad) to raise the money.
C. With Greece's government and its European creditors hitting loggerheads over the impending deadline for
its debt repayments, one 29-year-old from Yorkshire has come up with a novel solution.
D. Thom Feeney, who says he is fed up of "dithering" politicians, has set up a crowd funding page on IndieGoGo
where he hopes to raise €1.6bn (£1.14bn) to help bail out the country.
(a) ABCD (b) CDBA (c) ACBD (d) DCBA

11. A. A story not only allows us to understand ourselves and how we live, it can also bring about change.
B. Forced Entertainment’s retelling of the entire Shakespearean canon using household objects, from salt and
pepper cellars to bottles of olive oil, is entirely serious.
C. One is simply the bare bones, but a story is something more; its meanings depend on the storyteller and
how the tale is delivered, which parts are emphasised and which downplayed.
D. As the Mahabharata tells us at its beginning: “If you listen carefully, at the end you’ll be someone else.”
E. Not least because it points up the vast difference between telling a plot and telling a story.
(a) ACDBE (b) BECAD (c) DACBE (d) BEACD

Page 32 of 91
VERBAL ABILITY

HAP_VA_BTECH_SEM_4_1218

12. A. But seen from abroad, the UK looks a haven of stability.


B. Consumers are spending and businesses are investing.
C. In the past, news of a deficit would have alarmed foreign holders of sterling.
D. They would have pulled their money out of the UK, prompting a fall in the value of the pound, which would
have helped boost exports and trim imports, leading to an improvement in the balance of payments.
E. Growth was stronger in the first quarter at 0.4% than previously believed.
(a) CDAEB (b) ABDCE (c) CADBE (d) BADCE

13. A. At the age of twenty-one he wrote a treatise upon the binomial theorem which has had a European vogue.
B. But the man had hereditary tendencies of the most diabolical kind.
C. Moriarty is a man of good birth and excellent education, endowed by nature with a phenomenal
mathematical faculty.
D. A criminal strain ran in his blood, which, instead of being modified, was increased and rendered infinitely
more dangerous by his extraordinary mental powers.
E. On the strength of it, he won the mathematical chair at one of our smaller universities, and had, to all
appearances, a most brilliant career before him.
(a) ACDEB (b) ABCED (c) CAEBD (d) CEBAD

14. A. Since then, intelligence tests have been mostly used to separate dull children in school from average or
bright children, so that special education can be provided to the dull.
B. In other words, intelligence tests give us a norm for each age.
C. Intelligence is expressed as intelligence quotient and tests are developed to indicate what an average child
of a certain age can do-what a 5-year-old can answer, but a 4-year-old cannot, for instance.
D. Binet developed the first set of such tests in the early 1900s to find out which children in school needed
special attention.
E. Intelligence can be measured by tests.
(a) CDABE (b) DECAB (c) EDACB (d) CBADE

15. A. If the market falls to reflect our notions of equity, government intervention may be needed to redistribute
income.
B. Beyond the question of what to produce, we are also concerned about for whom output is to be produced.
C. In general, the market mechanism tends to answer the basic question of for whom to produce by
distributing a larger share of total output to those with the most income.
D. Although this result may be efficient, it is not necessarily equitable.
E. Is the distribution of goods and services generated by the marketplace “fair”?
(a) BEACD (b) DCBEA (c) CBEAD (d) DACBE

16. A. I wanted to be taken seriously.


B. Not about the teaching material, because I was well prepared and I was teaching what I enjoyed.
C. The first time I taught a writing class in graduate school, I was worried.
D. I really wanted to wear my shiny lip gloss and my girly skirt, but I decided not to.
E. Instead I was worried about what to wear.
F. And I was worried that if I looked too feminine, I would not be taken seriously.
(a) ABDCEF (b) CBFDAE (c) CBEADF (d) ABCFED

Page 33 of 91
VERBAL ABILITY

HAP_VA_BTECH_SEM_4_1218

17. A. One was in The Famous Victories of Henry V, created for the RSC’s First Encounters programme, which
cleverly takes a knife to both text and plot in a significantly pared-down version of both parts of Henry IV,
plus Henry V.
B. And not a single one of them was exactly as it appears on the page.
C. The second was near the beginning of Ivo van Hove’s Kings of War, performed in Dutch, which condenses
several of Shakespeare’s plays to explore the nature of kingship and the responsibilities of leaders.
D. The last was on Sunday night, watching some of Forced Entertainment’s table-top Shakespeare live-
streamed on the Guardian’s website.
E. Were any of these performances less Shakespearean than a full, uncut performance of the original text in
English?
F. Over the last couple of weeks I’ve seen three very different versions of the scene in Henry IV Part II, in which
the future Henry V tries on his ailing father’s crown.
G. I’d say a resounding no.
(a) ACBDEFG (b) GFEDACB (c) FBACDEG (d) DEGACBF

18. A. A bowl of hummus weighing 4,090kg.


B. The Lebanese fought back, making 10,452kg worth of hummus.
C. Its response?
D. As a Palestinian citizen of Israel, he takes his food history seriously, as do most residents of Abu Ghosh.
E. That’s why they were so affronted when in 2009 Lebanon unilaterally declared hummus Lebanese and
made a bowl of it weighing 2,056kg to prove it.
F. The chef’s hummus has been voted by the Jewish Journal’s food blog as the best in Israel and his village once
held the Guinness world record for the biggest bowl of hummus.
G. The village of Abu Ghosh wasn’t having it.
(a) GBCDEAF (b) FDEGCAB (c) AFBGDCE (d) ABCDEFG

19. A. Although there are large regional variations, it is not infrequent to find a large number of people sitting here
and there and doing nothing.
B. Once in office, they receive friends and relatives who feel free to call any time without prior appointment.
C. While working, one is struck by the slow and clumsy actions and reactions, indifferent attitudes, procedure
rather than outcome orientation, and the lack of consideration for others.
D. Even those who are employed often come late to the office and leave early unless they are forced to be
punctual.
E. Work is not intrinsically valued in India)
F. Quite often people visit ailing friends and relatives or go out of their way to help them in their personal
matters even during office hours.
(a) ECADBF (b) EADCFB (c) EADBFC (d) ABFCDE (e) None of these

20. A. It wasn’t reviewed by any of the national newspapers or tipped to be a Christmas chart topper.
B. But Chimamanda Ngozi Adichie’s We Should All Be Feminists might just be the most important book you
read all year.
C. Based on a TEDx talk the Nigerian writer gave in 2012, We Should All Be Feminists is a potent tour de force
on the subject of gender equality; one that whispers rather than shouts and confides rather than chides.
D. There is no excuse not to read it – particularly as it is just 52 pages long.
E. A lunch break or commute would provide ample opportunity.
F. Earlier this month, a short essay by a bestselling, prize-winning author was published with little fanfare.
(a) FBDCAE (b) FADCBA (c) FADCBE (d) FABCDE

Page 34 of 91
VERBAL ABILITY

HAP_VA_BTECH_SEM_4_1218

PARA-JUMBLES – II

SESSION – 10

Directions for Q1 to Q13: Sentences given in each question, when properly sequenced, form a coherent paragraph. The
first and last sentences are 1 and 6, and the four in between are labeled A, B, C and D. Choose the most logical order of
these four sentences from among the five given choices to construct a coherent paragraph from sentences 1 to 6.

1. 1. What does the state do in a country where tax morality is very low?
A. It tries to spy upon the taxpayers.
B. It investigates income sources and spending patterns.
C. Exactly what the tax authority tries to do now even if inconsistently.
D. It could also encourage people to denounce to the tax authorities any conspicuously prosperous neighbours
why may be suspected of net paying their taxes properly.
6. The ultimate solution would be an Orwellian System.
(a) BACD (b) DBAC (c) ABCD (d) DCBA

2. 1. The fragile Yugoslav State has uncertain future.


A. Thus there will surely be chaos and uncertainly if the people fail to settle their differences.
B. Sharp ideological differences already exist in the country.
C. Ethnic, regional, linguistic and material disparities are profound.
D. The country will also lose the excellent reputation it enjoyed in the international arena.
6. At worst, it will once more become vulnerable to international conspiracy and intrigue.
(a) BCAD (b) ADCB (c) ACBD (d) DBCA

3. 1. India’s experience of industrialization is characteristic of the difficulties faced by a newlyindependent


developing country.
A. In 1947 India was undoubtedly as underdeveloped country with one of the lowest per capita incomes in the
world.
B. Indian industrialization was the result of a conscious deliberate policy of growth by an indigenous political
elite.
C. Today India ranks fifth in the international community of nations if measured in terms of purchasing power.
D. Even today, however, the benefits of Indian industrialization since independence have not reached the
masses.
6. In India has been a limited success; one more example of growth without development.
(a) CDAB (b) DCBA (c) CABD (d) BACD

4. 1. The New Economic Policy comprises the various policy measures and changes introduced since July 1991.
A. There is a common thread running through all these measures.
B. The objective is simple – to improve the efficiency of the system.
C. The regulator mechanism involving multitude of controls has fragmented the capacity and reduced
competition even in the private sector.
D. The thrust of the new policy is towards creating a more competitive environment as a means to improving
the productivity and efficiency of the economy.
6. This is to be achieved by removing the barriers and restriction on the entry and growth of firms.
(a) DCAB (b) ABCD (c) BDAC (d) CDBA

Page 35 of 91
VERBAL ABILITY

HAP_VA_BTECH_SEM_4_1218

5. 1. Commercial energy consumption shows an increasing trend and poses a major challenge for the future.
A. The demand for petroleum during 1996-97 and 2006-07 is anticipated to 81 million tons and 125 million
tons respectively.
B. According to the projections of the 14th power Survey Committee Report, the electricity generation
requirements from utilities will be about 415 billion units by 1996-97 and 824 billion units by 2006-07.
C. The production of coal should reach 303 million tons by 1996-97 to achieve plan targets and 460 million
tons by 2006-07.
D. The demand for petroleum products has already outstripped indigenous production.
6. Electricity is going to play a major role in the development of infrastructural facilities.
(a) DACB (b) CADB (c) BADC (d) ABCD

6. 1. Count Rumford is perhaps best known for his observations on the nature of heat.
A. He undertook several experiments in order to test the theories of the origin of frictional heat.
B. According to the colorists, the heat was produced by the ‘’ caloric’’ squeezed out of the chips in the process
of separating them from the larger pieces of metal.
C. Lavoisier had introduced the term ‘’caloric’’ for the weightless substance heat, and had included it among
the chemical elements along with carbon, nitrogen and oxygen.
D. In the ammunitions factory in Munich, Rumford noticed that a considerable degree of heat developed in a
brass gun while it was being bored.
6. Rumford could not believe that the amount of heat generated could have come from the small amount of
dust created.
(a) ABCD (b) CBDA (c) ACDB (d) CDAB

7. 1. The death of cinema has been predicted annually.


A. It hasn’t happened.
B. It was said that the television would kill it off-and indeed audiences plummeted, reaching a low in 1984.
C. Film has enjoyed a renaissance, and audiences are not roughly double of what they were a decade ago.
D. Then the home computer became the projected nemesis followed by satellite television.
6. Why? probably because even in the most atomized of societies, we human beings feel the need to share out
fantasies and our excitement.
(a) CADB (b) BDAC (c) ABDC (d) DABC

8. 1. Visual recognition involves storing and retrieving of memories.


A. Psychologists of the Gastalt school maintain that objects are recognized as a whole in a parallel procedure.
B. Neural activity, triggered by the eye, forms an image in the brain’s memory system that constitutes an
internal representation on the viewed object.
C. Controversy surrounds the question of whether recognition is a single one-step procedure or a serial step-
by-step one.
D. When an object is encountered again, it is matched with its internal recognition and thereby recognized.
6. The internal representation is matched with the retinal image in single operation.
(a) DBAC (b) DCAB (c) BDCA (d) CABD

9. 1. The history of mammals dates back at least to Triassic time.


A. Miocene and Pliocene time was marked by culmination of several groups and continued approach towards
modern characters.
B. Development was retarded, however until the sudden acceleration of evolutional change that occurred in
the oldest Paleocene.
C. In the Oligocene Epoch, there was further improvement, with appearance of some new lines and extinction
of theories.

Page 36 of 91
VERBAL ABILITY

HAP_VA_BTECH_SEM_4_1218

D. This led in Eocene time to increase in average size, larger mental capacity, and special adaptations for
different modes of life.
6. The peak of the career of mammals in variety and average large size was attained in this epoch.
(a) BDCA (b) ACDB (c) BCDA (d) ACBD

10. 1. The wind had savage allies.


A. If it had not been for my closely fitted helmet, the explosions might have shattered my eardrums.
B. The first clap of thunder came as a deafening explosion that literally shook my teeth.
C. I did not hear the thunder I actually felt it – an almost unbearable physical experience.
D. I saw lightning all around me in every shape imaginable.
6. It was raining so torrentially that I thought I would drown in mid air.
(a) BCAD (b) CADB (c) CBDA (d) ACDB

11. 1. Managers need to differentiate among those who commit an error once, those who are repetitively errant
but can be corrected, and those who are basically wicked.
A. The persons in this category will resort to sweet-talk and make all sorts of promises on being caught, but, at
the first opportunity will revert to their bad ways.
B. Managers must take ruthless action against the basically wicked and ensure their separation from the
organization at the earliest.
C. The first category needs to be corrected softly and duly counseled; the second category should be dealt with
firmly and duly counseled till they realize the danger of persisting with their errant behavior.
D. It is the last category of whom the managers must be most wary.
6. The punishment must be fair and based on the philosophy of giving all the possible opportunities and help
prior to taking ruthless action.
(a) ADCB (b) CDAB (c) CADB (d) BDAC

12. 1. Many space enthusiasts now warn that only private enterprise will truly drive human expansion into space,
and yet America’s government keeps ignoring them.
A. It may seem surprising, but there are large numbers of people who would spend hundreds of thousands of
dollars on a trip into space.
B. Tourism and entertainment are both possibilities.
C. Two people, so far, have spent $20 m, and another two are on their way.
D. What might cause market forces to take up the mission?
6. Film and television companies would also spend tens of millions if they could.
(a) DCAB (b) BACD (c) DBAC (d) ACBD

13. 1. Jane Austen died and came back as a fantasy writer.


A. The book itself has been called, by a media ever eager to summarise even 800-pages hardcover tomes into a
snappy catchphrase. “Harry Potter for adults”.
B. Unlike her previous avatar, the 21st century Ms Clarke (nee Austen) seems to be enjoying the attention
showered upon her and far from publishing her first book under a pseudonym, has been a central performer
at her own media circus.
C. The book, which she now calls her “debut”, began attracting media attention long before publication and on
release it’s been universally lauded.
D. In her new avatar, she calls herself Susanna Clarke, lives in Cambridge, and has authored a fat historical
fantasy novel set in the year 1860.
6. It’s also been praised by perhaps the best living author of British fantasy novels, Neil Gaiman, as
“unquestionably the finest English novel of the fantastic written in the past seventy years.”
(a) BADC (b) DBCA (c) DCAB (d) DCBA

Page 37 of 91
VERBAL ABILITY

HAP_VA_BTECH_SEM_4_1218

Directions for Q14 to Q20: The first and the last parts of the sentence are marked 1 and 6. The rest of the sentence is
split into four parts and marked-P, Q, R and S. These four parts are not given in their proper order. Read the sentences
and find out which of the five combinations is correct.

14. 1. It depends upon the


P. a new heaven on earth or to destroy
Q. user, whether science will be used to create
R. the word in a
S. outlook and mentality of the
6. common conflagration.
(a) PQRS (b) SPQR (c) QPRS (d) SQPR

15. 1. The English


P. plundered the country
Q. and raw materials,
R. as a result, the Company became rich
S. of precious mineral wealth
6. and the Indian people became poor.
(a) PQSR (b) SQPR (c) SRQP (d) PSQR

16. 1. The power and pride


P. for in the courage
Q. of Sparta was above all
R. discipline and skill of these troops
S. in its army,
6. it found its security and its ideal.
(a) SPRQ (b) PQRS (c) QSPR (d) PRQS

17. 1. It is the
P. the careful observer Q. apparently trivial phenomena
R. which gives even the S. intelligent eye of
6. their value.
(a) SPRQ (b) PQRS (c) QSPR (d) PRQS

18. 1. Nuclear test explosions


P. food as well as
Q. present and future generations
R. directly injuring the
S. take place, contaminating air and water and
6. of mankind.
(a) RQSP (b) PSQR (c) SPRQ (d) PQRS

19. 1. The club is an


P. nearby municipal school, for the children
Q. proper and healthy functioning of a
R. are constantly disturbed by its
S. intolerable nuisance to the
6. bizarre activities.
(a) QSRP (b) SQPR (c) PQRS (d) RPSQ

Page 38 of 91
VERBAL ABILITY

HAP_VA_BTECH_SEM_4_1218

20. 1. What the country needs


P. and change tactics
Q. who would encourage players
R. are coaches and officials
S. to read the game as it progresses
6. accordingly.
(a) RQPS (b) SPRQ (c) QSPR (d) RQSP

SESSION – 11

Slide 1: PREMISES AND CONCLUSIONS

Slide 2:

1. No one under eighteen-years-old can vote.

2. Harish is under eighteen-years-old.

3. Therefore, Harish cannot vote.

• Arguments are composed of one or more premises and a conclusion.

• Premises are statements offered as reasons for accepting another statement.

• A conclusion is a statement supported by reasons.

In this example, statements 1 and 2 are premises, and statement 3 is the conclusion.

In other words, statements 1 and 2 are the reasons leading to the conclusion in Statement 3.

Slide 3:

Here’s a graphical representation of Premise and Conclusion.

Slide 4: Chennai has a population of over 4 million. Mumbai has a population of close to 12 million. Delhi has a
population of almost 10 million.

The three sentences above do not constitute an argument because there is no conclusion, and arguments MUST have a
conclusion.

They are just a series of assertions and make no judgement of any kind.

This is the fundamental difference between ARGUMENTS and FACT SETS.

Page 39 of 91
VERBAL ABILITY

HAP_VA_BTECH_SEM_4_1218

Slide 5: Differentiating premises from conclusions requires both practice and attention to the finer elements of
language.

TIP #1: INDICATOR WORDS

Look for premise indicators--words like because, since, for, and given that--that provide clues when premises are being
offered.

Look for conclusion indicators--words like therefore, thus, hence, and so--that provide clues when conclusion indicators
are being offered.
Premise Indicators Conclusion Indicators
Since May be inferred from Therefore For this reason
Because May be derived from Hence For these reasons
For May be deduced from So It follows that
As Given that Accordingly I conclude that
Follows from In consequence Which shows that
As shown by Consequently Which means that
Inasmuch as Proves that Which entails that
As indicated by As a result Which implies that
The reason is that Thus We may infer

Slide 6 & 7: In a free society people have the right to take risks as long as they do not harm others as a result of taking
the risks. Therefore, it should be each person’s decision whether or not to wear a seat belt.

Identify the premise(s) and conclusion in this argument.

Premise: In a free society, people have the right to take risks as long as they do not harm others as a result of taking
the risks.
Conclusion: It should be each person’s decision whether or not to wear a seat belt.
Notice the word therefore in this passage. This is a premise indicator that gives us a clue.

Slide 8 & 9: Given that price of steel is rising, we will no longer be able to offer discounts on our car parts.
Identify the premise(s) and conclusion in this argument.

Premise: Price of steel is rising.

Conclusion: We will no longer be able to offer discounts on our car parts.


Notice the word given in the beginning. This is a premise indicator that gives us a clue.

Slide 10 & 11: Last year, the city of Indibad increased the size of its police force by fifty officers. This year, there was a
10 percent decrease in the number of violent crimes reported in Indibad. Clearly, a larger police force discourages
criminal activity.
Identify the premise(s) and conclusion in this argument.

Premise: Indibad increased the size of its police force and saw a decrease in reports of violent crime.
Conclusion: A larger police force discourages criminal activity.
Notice the word clearly in this passage. This tips us off that a premise is being offered.

Page 40 of 91
VERBAL ABILITY

HAP_VA_BTECH_SEM_4_1218

Slide 12 & 13: A good society treasures its dissidents and mavericks because it needs the creative thinking that
produces new hypotheses, expanded means, a larger set of alternatives, and, in general, the vigorous conversation
induced by fresh ideas.

Identify the premise(s) and conclusion in this argument.

Premise: A good society needs the creative thinking that produces new hypotheses, expanded means, a larger set of
alternatives, and, in general, the vigorous conversation induced by fresh ideas.

Conclusion: A good society treasures its dissidents and mavericks.

Notice the word because in this passage. This tips us off that a premise is being offered.

Slide 14 & 15: Television has a harmful effect on society. This can be seen from the poor school performance of
children who watch significant amounts of television and from the fact that children who watch more than six hours
of television in a day tend to read less than non-television watching children.

Identify the premise(s) and conclusion in this argument.

Premise 1: This can be seen from the poor school performance of children who watch significant amounts of television.

Premise 2: children who watch more than six hours of television in a day tend to read less than non-television
watching children.

Conclusion: Television has a harmful effect on society.

This argument features the premise indicator this can be seen from.

Slide 16 & 17: Make a will. Otherwise, the law will determine who gets your things.

Identify the premise(s) and conclusion in this argument.

Premise: If you don't make a will, the law will determine who gets your things.

Conclusion: You ought to make a will.

The word otherwise often functions--as it does here--as a premise indicator.

Notice that both the premise and the conclusion have been rephrased slightly. The premise has been rephrased in
order to make it a complete sentence. The conclusion has been restated in order to make clear that it is intended as a
statement rather than as a command.

Slide 18 & 19: Therefore, since higher debt has forced consumers to lower their savings, banks now have less money
to loan.

Identify the premise(s) and conclusion in this argument.


Premise: Higher debt has forced consumers to lower their savings.
Conclusion: Banks now have less money to loan.
Here, it could get confusing because the premise indicator since and the conclusion indicator therefore are placed next
to each other.
But you’ll see that therefore is still introducing the conclusion, but the appearance of the conclusion is interrupted by a
clause that contains the premise.
Clue: Notice the commas before since and at the end of the premise clause!

(Note: Similar confusing forms include “This, because…”, “Hence, due to…” etc.)

Page 41 of 91
VERBAL ABILITY

HAP_VA_BTECH_SEM_4_1218

Slide 20 & 21: Research universities also must aggressively support teaching. After all, a significant percentage of
their students are undergraduates, and such institutions are clearly obligated to provide them a quality education.

Identify the premise(s) and conclusion of this argument.

Premise 1: A significant percentage of research universities' students are undergraduates.

Premise 2: Such institutions are clearly obligated to provide undergraduates with a quality education.

Conclusion: Research universities also must aggressively support teaching.

Notice the phrase after all. This phrase is often used as a premise indicator.

Slide 22 & 23: Every trainer at the SMART teaches exactly one class per semester. SMART’s Professor Haridas,
therefore, is teaching exactly one class this semester. Moreover, I heard Professor Haridas say he was teaching only a
single class.

Identify the premise(s) and conclusion of this argument.

Premise : Every trainer at the SMART teaches exactly one class per semester.

Conclusion: Professor Haridas is teaching exactly one class this semester.

Therefore in the second sentence is a conclusion indicator.

But what about the third sentence?

It begins with the additional premise indicator moreover. The premise in this statement is non-essential to the
argument, but provides additional proof for the conclusion.

(Note: Other additional premise indicators include words like furthermore, besides, in addition, what’s more.)

Slide 24 & 25: Phenylketonurics are people who cannot metabolize the amino acid phenylalanine. There are dangers
associated with phenylketonurics, and products containing phenylalanine must carry a warning label that states,
“Phenylketonurics: contains phenylalanine”. In addition, all children in developed societies receive a phenylketonuria
test at birth. Hence, at the moment, we are doing as much as possible to protect against this condition.
Identify the premise(s) and conclusion of this argument.
Premise 1: Phenylketonurics are people who cannot metabolize the amino acid phenylalanine.
Premise 2: There are dangers associated with phenylketonurics, and products containing phenylalanine must carry a
warning label that states, “Phenylketonurics: contains phenylalanine”.
Additional Premise: In addition, all children in developed societies receive a phenylketonuria test at birth.
Conclusion: Hence, at the moment, we are doing as much as possible to protect against this condition.
Slide 26 & 27: The United States prison population is the world’s largest and consequently we must take steps to
reduce crime in this country. Although other countries have higher rates of incarceration, their statistics have no
bearing on the dilemma we face.
Identify the premise(s) and conclusion of this argument.
Premise: The United States prison population is the world’s largest.
Conclusion: We must take steps to reduce crime in this country.
The first sentence has a clear premise and conclusion. Consequently is a conclusion indicator.
But what about the second sentence?

Page 42 of 91
VERBAL ABILITY

HAP_VA_BTECH_SEM_4_1218

It offers a Counter Premise: Other countries’ higher rates of incarceration have no bearing on the dilemma we face.

The word although in the second sentence indicates a counter premise.

Slide 28: More on Counter Premises

Counter premises bring up points of opposition or comparison that NEGATE the author’s argument.

Common Counter Premise Indicators include

But, Yet, However, On the other hand, Admittedly, In contrast, Although, Even though, Still, Whereas, in spite of, Despite,
After all

While at first glance it may seem as if the author is opposing his own case, by raising the counter-premise and then
directly addressing the complaint, the author actually strengthens his case.

Slide 29 & 30: Wine is made by crushing grapes and eventually separating the juice from the grape skins. However,
the separated juice contains impurities and many wineries do not filter the juice. These wineries claim the unfiltered
juice ultimately provides a more flavorful and intense wine. Since these wine makers are experts, we should trust
their judgement and not shy away from unfiltered wine.

Identify the premise(s) and conclusion of this argument.

Premise 1: Wine is made by crushing grapes and eventually separating the juice from the grape skins.

Counter Premise: However, the separated juice contains impurities and many wineries do not filter the juice.

Premise 2: These wineries claim the unfiltered juice ultimately provides a more flavorful and intense wine.

Premise 3: These wine makers are experts.

Conclusion: We should trust their judgement and not shy away from unfiltered wine.

Slide 31 & 32: During last night’s robbery, the thief was unable to open the safe. Thus, last night’s robbery was
unsuccessful despite the fact that the thief stole several documents. After all, nothing in those documents was as
valuable as the money in the safe.

Identify the premise(s) and conclusion of this argument.

Premise: During last night’s robbery, the thief was unable to open the safe.

Counter Premise: Despite the fact that the thief stole several documents.

Additional Premise: After all, nothing in those documents was as valuable as the money in the safe.

Conclusion: Thus, last night’s robbery was unsuccessful.

Slide 33:

(a) Ask yourself, "What claim is the writer or speaker trying to prove?"

That claim will be the conclusion.

What if the passage has no indicator words?

TIP #2: THE WHY TEST

(b) Try putting the word "therefore" before each of the statements in turn. The statement it fits best will be the
conclusion.

Page 43 of 91
VERBAL ABILITY

HAP_VA_BTECH_SEM_4_1218

The Why Test. Once you have found the conclusion, ask yourself why the author believes the conclusion to be true.
The premises should provide the answer to the question. If you try the Why Test and the answer does not make sense,
you have probably reversed the conclusion and premises.

Slide 34 & 35: Companies have identified that giving workers the option of flexible lunch hours leads to happier
employees. Happier employees are more productive. Company X has flexible lunch hours. Company Z operates on a
strict 1 to 2 pm lunch break. Company X will certainly beat Company Z in worker productivity.

Identify the premise(s) and conclusion in this argument.

Premise 1: Flexible hours lead to happier employees, who will be more productive.

Premise 2: Company A has flexible hours and Company B does not.

Conclusion: Company A will certainly beat Company B in worker productivity.

There are no straightforward premise indicators here. However, it is possible to identify the conclusion by
substituting “therefore” before each sentence and confirming with the “Why Test”.

Slide 36 & 37: The Jews and Arabs have been fighting for centuries and I seriously doubt this will ever be resolved.
The United States should get out of this never-ending fight, or the next terrorist bomb might be in Washington--and it
just might be nuclear. (John G. Ferguson III, Letter to the Editor, USA Today, February 12, 2001)

Identify the premise(s) and conclusion in this argument.

Premise 1: The Jews and Arabs have been fighting for centuries.

Premise 2: There is serious doubt this will ever be resolved.

Premise 3: If the United States does not get out of this never-ending fight, the next terrorist bomb might be in
Washington--and it might be nuclear.

Conclusion: The United States should get out of the never-ending fight between the Jews and the Arabs.

In this passage, there are no indicator words to assist us, however the form of the last sentence ("X should do Y, or else
Z will happen") is a common conclusion-premise pattern.

Slide 38 & 39: The Indian census is not perfect: thousands of Indians probably go uncounted. However, the basic
statistical portrait of the nation painted by the census is accurate. Certainly some of the poor go uncounted,
particularly the homeless; but some of the rich go uncounted as well, because they are often abroad and travelling
between one residence and another.

Page 44 of 91
VERBAL ABILITY

HAP_VA_BTECH_SEM_4_1218

Identify the premise(s) and conclusion in this argument.

Counter Premise: The Indian census is not perfect: thousands of Indians probably go uncounted.

Premise: Some of the poor and some of the rich go uncounted.

Conclusion: The basic statistical portrait of the nation painted by the census is accurate.

(Note: Besides applying the ‘Why Test’, also remember: Whenever a sentence is followed by hard transition words like
however, but, yet, on the other hand, etc it cannot be the conclusion because the transition is a form of negation.)

SESSION – 12

Slide 1: Finding Assumptions


Slide 2: QUICK RECAP
All men are mortal.
Therefore, Haridas is mortal.
• Arguments are composed of one or more premises and a conclusion.
• Premises are statements offered as reasons for accepting another statement.
• A conclusion is a statement supported by reasons.
• Assumptions are the unstated link between premise and conclusion.

In this example, statements 1 is the premise, and statement 2 is the conclusion. However, there is a still a missing link
in the argument… i.e. Haridas is a man.

Slide 3:
Here’s another graphical representation of Assumptions.

(ASSUMPTIONS)

You can think of the conclusion of an argument as the top of a building, supported by the building itself (the premises)
and the unseen underground foundation (the assumptions).

Page 45 of 91
VERBAL ABILITY

HAP_VA_BTECH_SEM_4_1218

Slide 4 & 5

My dog won top prize at the dog show last year. Therefore, he will win again this year.

Identify the premise(s), assumption(s) and conclusion in this argument.

Premise: My dog won top prize at the dog show last year.

Conclusion: Therefore, he will win again this year.

Assumptions:

1. The author's dog will be entered in this year's show.

2. The author's dog will be eligible to win the top prize.

3. The author's dog will still be better than all of the other dogs from last year's show, and it will beat out any new
dogs who may be entered this year.

PPT Notes: Anyone of the above assumptions could serve as the correct answer, because each one is necessary in order
for the author to believe his or her conclusion. In other words, if the assumption in the correct answer were not true,
you could reject the conclusion on that basis alone. For example, if we learned that no dog is allowed to win the top
prize two years in a row, then clearly the author's dog is not going to win this year's prize.

At the same time, NONE of the assumptions listed is sufficient by itself to prove that the author's dog will definitely win
the prize this year. For instance, if the author's dog is eligible to win the prize this year, that does not mean it will
definitely do so. The correct answer does not need to make the conclusion definitely true. Indeed, most of the time, the
correct answer will only make the conclusion somewhat more likely to be true.

Slide 6

From the previous example, we learn that…


An argument can involve multiple assumptions. But what is most crucial, especially from the test perspective, is THE
ASSUMPTION on which the entire argument is built.
To identify the appropriate assumption, you should look for the assumption to:
(1) bridge a gap between any premise and the conclusion, and
(2) support/strengthen/validate the conclusion.
Slide 7 & 8
Identify the premise(s), assumption(s) and conclusion in this argument.
Studying regularly is one factor that has been shown to improve one's performance on the GMAT. Anita took the
GMAT and scored a 500. If she studies several times a week, Anita can expect to improve her score.
Premise 1: Studying regularly is one factor that has been shown to improve one's performance on the GMAT.
Premise 2: Anita took the GMAT and scored a 500.
Conclusion: If she studies several times a week, Anita can expect to improve her score.
Assumption: Studying several times a week qualifies as studying regularly.
Slide 9: ASSUMPTION vs. INFERENCE
An inference is a piece of information that can be logically deducted from the given set of statements.
Example: Everyone who has read this lesson will be more informed about Assumptions. Haridas has read this lesson.
Inference: Haridas is more informed about Assumptions.

Page 46 of 91
VERBAL ABILITY

HAP_VA_BTECH_SEM_4_1218

An assumption on the other hand is an unstated or “hidden” premise.

Example: Everyone who has read this lesson will be more informed about Assumptions. Haridas received this lesson
by email. Hence, Haridas is more informed about Assumptions.

Assumption: Haridas has read every email he received.

Slide 10 & 11: TYPES OF ASSUMPTIONS

Identify the assumption(s) in this argument.

Anil weighs less than 90 kilos. Therefore, he cannot have a successful career as a racecar driver.

In order to make the logical leap from Point A to Point B - that Anil is under 90 kilos and therefore cannot have a
successful career as a race car driver - we must insert an additional premise. This unstated premise is an assumption.

Assumption: In order to have a successful career as a racecar driver, one must weigh at least 90 kilos.

TYPE #1: Assumptions that fill a LOGIC GAP

These assumptions answer the question, "How do we logically get from Point A to Point B?"
In this case, P represents weighs less than 90 kilos and C represents cannot be a successful race car driver. An
assumption - call it A - sits between P and C. This A is the assumption that Anil cannot be a successful race car driver
unless he weighs 90 kilos or more. Note that if that assumption is not true, then the whole argument falls apart.

Slide 12 & 13: TYPES OF ASSUMPTIONS

Identify the assumption(s) in this argument.

Sidney's get-rich-quick scheme is sure to succeed. He will buy undervalued properties in foreclosure. Then he will
resell the properties to a local real estate developer and generate large profits.

Assumptions:

1. Undervalued properties exist in foreclosure and easily can be found and purchased by Sidney.

2. It also assumes that local real estate developers will want to buy such properties from Sidney and will be willing to
pay more than he paid in the first place.

TYPE #2: Assumptions that establish the FEASIBILITY of PREMISES in the argument

Page 47 of 91
VERBAL ABILITY

HAP_VA_BTECH_SEM_4_1218

In this case, P1 represents he will buy undervalued properties in foreclosure. P2 represents he will sell these
properties to a developer for large profits, and C represents Sidney's get-rich-quick scheme is sure to succeed.
Statement A makes explicit the assumption that Sidney actually CAN buy undervalued properties in foreclosure and
sell them for higher prices.
Slide 14 & 15: TYPES OF ASSUMPTIONS
Identify the assumption(s) in this argument.
A magazine published an article proclaiming that one can get a promotion by playing golf with one's boss. Kevin
concludes that the best way for him to get promoted is to take golf lessons and join his boss's country club.
These arguments will often use some type of superlative qualifier in the conclusion. That is, we are told that the given
path is the only way to reach a goal, or the best or worst way. In order for such a claim to be true, there cannot be
another way, or a better or worse way.
Assumption: There are no other better ways to gain a promotion than to play golf with one's boss.
TYPE #3: Assumptions that eliminate alternate paths to reach a given conclusion

In this case, X represents playing golf with one's boss and Z represents getting a promotion. The arrow indicates that X
is the best way to get to Z. Y makes explicit that there is no better way for Kevin to accomplish his goal.
There is one another assumption here… What is that?
Assumption 2: That Kevin’s boss plays golf! Now this is a TYPE #2 assumption.
PPT Notes: Notice that the conclusion uses the phrase the best way. This ignores the many other (probably more reliable)
ways that Kevin could get himself noticed and promoted. In other words, the argument assumes that there is no better
way for him to get promoted than to play golf with his boss.
Slide 16 & 17: TYPES OF ASSUMPTIONS
Identify the assumption(s) in this argument.
Scientists have discovered that people with Elmer's disease have elevated levels of elastomer in their blood. The
scientists concluded that a person without Elmer's disease but with elevated levels of elastomer in the blood is likely
to develop Elmer's disease in the future.
Assumption: Scientists are assuming that elevated levels of elastomer is the cause for Elmer’s disease.
TYPE #4: Assumptions that eliminate alternate causes for a given conclusion

Assuming high levels of elastomer is X and Elmer’s disease is Z, the argument is built on the certain assumption
that only X causes Z, and NOT that Z could cause X.
However, Elmer's disease and elastomer show up together and are correlated. But we do not know whether elevated
levels of elastomer causes Elmer's disease, or whether Elmer's disease causes elevated levels of elastomer. There is
even a third possibility: both Elmer's disease and elevated levels of elastomer could be caused by some third,
unknown factor.

Page 48 of 91
VERBAL ABILITY

HAP_VA_BTECH_SEM_4_1218

In short, correlation is not the same as causation.


PPT Notes: In short, if a premise provides a correlation between X and Z, and the conclusion provides a causality in one
direction (X causes Z), then the assumption you are usually looking for is that Z does NOT cause X. That is, you must rule
out the causality in the other direction.
Slide 18: PRACTICE QUESTION #1
When news periodicals begin forecasting a recession, people tend to spend less money on discretionary purchases.
Therefore, the perceived threat of recession decreases the willingness of people to purchase products that they regard
as optional or luxury goods.

The argument above assumes that ________.


(A) there are more luxury goods available after a recession is forecast.
(B) recently, the threat of recession has been increasingly publicized as news periodicals have grown more pervasive.
(C) most people do not regularly read news periodicals.
(D) people's perception of the threat of recession increases when news periodicals begin forecasting a recession.
(E) the people who spent the most money before a recession was forecast were among those who curtailed their
spending after the recession became apparent.

Answer: D
Type 1 Assumption  Fills a logic gap
Note that the premise states that the decreased spending is due to the fact that news periodicals are forecasting a
recession. Furthermore, the expressions perceived threat of recession (in the passage) and perception of the threat of
recession (in the answer choice) are essentially synonymous. Finally, the answer choice logically fills a gap between a
premise and the conclusion.
Premise: News periodicals publish the forecast of a recession
Assumption: As a result, people's perception of the threat of a recession increases
Premise: People spend less money on discretionary items
Conclusion: Therefore, the perceived threat causes people to spend less money

Slide 19: PRACTICE QUESTION #2

The local university recently hired a new soccer coach. Although she has several years' worth of coaching experience
and is a diligent student of the game, she was never a member of a collegiate soccer team. For this reason, the new
coach will be unable to build a successful program.
The argument above is based on which of the following assumptions?
(A) The local university should have hired a former collegiate soccer player as its new coach.
(B) Coaching experience is one of the most crucial factors for coaching success.
(C) The previous coach at the university was dismissed due to her lack of success.
(D) To build a successful soccer program as a coach, one must be a former collegiate soccer player.
(E) The university does not plan to provide the new coach with the resources necessary to build a successful
program.
Answer: D
Type 1 Assumption  Fills a logic gap

Slide 20: PRACTICE QUESTION #3

Methyltetrachloride (MTC) is a chemical found in some pesticides, glues. and sealants. Exposure to MTC can cause
people to develop asthma. In order to halve the nation's asthma rate, the government has announced that it will ban
all products containing MTC.

Page 49 of 91
VERBAL ABILITY

HAP_VA_BTECH_SEM_4_1218

The government's plan to halve the nation's asthma rate relies on which of the following assumptions?
(A) Exposure to MTC is responsible for no less than half of the nation's asthma cases.
(B) Products containing MTC are not necessary to the prosperity of the American economy.
(C) Asthma has reached epidemic proportions.
(D) Exercise and proper nutrition are helpful in maintaining respiratory health.
(E) Dust mites and pet fur cause asthma.

Answer: A
Type 1 Assumption  Fills a logic gap
The government's proposed ban on MTC cannot halve the nation's asthma rate unless MTC is actually the thing
responsible for at least half the nation's asthma cases. If other things are responsible for half (or more than half) of the
nation's asthma cases, then banning MTC will not have the desired effect of reducing asthma rates by half.

Slide 21: PRACTICE QUESTION #4

In order to achieve a substantial reduction in the extremely high number of car accidents in our country, we should
implement a radical new safe driving plan. Seat belts and air bags would be eliminated from the driver's side of all
automobiles to provide the driver with the strongest possible incentive to drive safely. Further, an electroshock system
would administer shocks to the driver if he or she exceeds the speed limit or engages in other unsafe driving practices.

In declaring that the radical new safe driving plan will help to reduce accident rates, the author assumes which of the
following?
(A) Many car accidents are caused, at least partially, by naturally occurring conditions such as rain and fog.
(B) Accidents in which one or both participants exceed 60 miles per hour account for a majority of all fatal car
accidents.
(C) A significant number of accidents are the result of negligence or other unsafe driving practices on the part of the
driver.
(D) To alleviate safety concerns, citizens should be allowed to reinstall their drivers' side seat belts and air bags at
their own expense.
(E) If not implemented properly, electroshock systems can cause heart attacks and other health problems.
Answer: C
Type 2 Assumption  Establishes Feasibility of Premise
The author assumes that a substantial number of accidents are due to driver error or could be avoided with a change in
driver. behavior. The author also assumes that the electroshock system will not itself cause a substantial number of
accidents, or health problems. that could lead to accidents.
Slide 22: PRACTICE QUESTION #5

Computers that can be identified and traced are of no use to thieves. So Cyber Infotech has instituted a plan to reduce
theft by etching the serial number of each computer both on the frame of the monitor and on the face of the hard drive
where it will be impossible to overlook.

Cyber Infotech’s plans assume which of the following?


(A) Cyber Infotech’s plan assumes that thieves do not steal computers that are of no value to them.
(B) Cyber Infotech’s competitors also plan to etch serial numbers on their computers.
(C) Thieves are more inclined to steal computers than any other type of office equipment.
(D) Etching the serial number on the computer has no effect on the performance of the computer.
(E) Imposing stiffer jail sentences for thieves would not decrease the number of computers stolen.
(F) USEFUL TECHNIQUE: The Reverse Test
One way to check whether an answer choice is a necessary assumption is to reverse or negate, that answer. If the
negation would weaken the argument, that answer is a necessary assumption.
Answer: A

Page 50 of 91
VERBAL ABILITY

HAP_VA_BTECH_SEM_4_1218

Slide 23: PRACTICE QUESTION #6


Editorial: The Intercontinental Bank should reallocate the voting shares of its members in order to more effectively
shape global economic policy. For example, China comprises about 15 percent of the world's gross domestic product
but has only a 3 percent voting share, whereas Belgium, with less than 1 percent of the global economy, has a 2
percent share.
Which of the following is an assumption upon which the author depends?
(A) The United States has a larger voting share of the Intercontinental Bank than does China or Belgium.
(B) The specific allocation of voting shares factors into the Intercontinental Bank's effectiveness in shaping global
economic policy.
(C) Only voting shares that are precisely proportional to each country's contribution to the global economy are
appropriate for the Intercontinental Bank.
(D) The Intercontinental Bank is necessary to the maintenance of a prosperous global economy.
(E) As one of the fastest growing economies, China should have a larger voting share in the Intercontinental Bank.
Answer: B
Type 2 Assumption  Establishes Feasibility of Premise
The argument does not actually present any evidence that the efficacy of the Intercontinental Bank will improve if Voting
shares are reallocated; this must be assumed in order to support the conclusion.
Slide 24: PRACTICE QUESTION #7
Advertisement: According to a recent research study, daily use of Fresh Start, a new toothpaste, reduces the risk of
developing dental cavities by over 20 percent. In addition, as a result of a new formula, the use of Fresh Start results in
whiter, healthier-looking teeth. Clearly, Fresh Start not only gives your teeth a beautiful look but also provides the
most reliable protection against dental cavities.
Which of the following is an assumption in the argument above?
(A) No other toothpaste provides more reliable protection against dental cavities.
(B) Fresh Start's formula does a better job of whitening teeth than do competitor formulas.
(C) People are just as interested in having beautiful teeth as they are in having healthy teeth.
(D) Fresh Start also prevents other dental disorders, such as gingivitis.
(E) Reliable protection against dental cavities, combined with excellent aesthetic properties, is likely to make Fresh
Start a popular toothpaste on the market.
Answer: A
Type 3 Assumption  Eliminates an Alternate Path to the same End
Since the argument singles out Fresh Start as the most reliable protection against dental cavities, the author must
assume that no other toothpaste provides stronger protection against cavities. Otherwise, the conclusion of the argument
is inaccurate.
Slide 25: PRACTICE QUESTION #8

Student Advisor: One of our exchange students faced multiple arguments with her parents over the course of the past
year. Not surprisingly, her grade point average (GPA) over the same period showed a steep decline. This is just one
example of a general truth: problematic family relationships can. cause significant academic difficulties for our students.

Which of the following is an assumption underlying the general truism claimed by the Student Advisor?
(A) Last year, the exchange student reduced the amount of time spent on academic work, resulting in a lower GPA.
(B) The decline in the GPA of the exchange student was not the reason for the student's arguments with her parents.
(C) School GPA is an accurate measure of a student's intellectual ability.
(D) If proper measures are not taken, the decline in the student's academic performance may become irreversible.
(E) Fluctuations in academic performance are typical for many students.

Page 51 of 91
VERBAL ABILITY

HAP_VA_BTECH_SEM_4_1218

Answer: B
Type 3 Assumption  Eliminates an Alternate Path to the same End
This assumption correctly eliminates the alternate model of causation, demonstrating that the decline in the GPA did not
cause the arguments between the student and her parents.

Slide 26: PRACTICE QUESTION #9

An industry analyst asserted in his recent report that the relative scarcity of housing in a particular market leads to
larger than normal increases in price. During the late 1990s, according to the analyst's report, occupancy rates -- a
measure of the percentage of housing occupied at a given time -- in crowded urban markets such as New York and San
Francisco hovered around 99.5%. During the same period, housing prices increased by as much as 100% per year,
compared to more normal past increases in the range of 5% to 15% per year.

Which of the following is an assumption that supports the analyst's assertion?


(A) In the housing market, there generally must be at least five buyers per seller in order to cause larger than normal
increases in price.
(B) Increases in demand often reflect an influx of new buyers into the marketplace or an unusual increase in buying
power on the part of the customer.
(C) The US housing market showed a larger than average increase in the 1990s across the country, not just in
crowded urban areas.
(D) Price increases do not cause people to withhold their houses from the market in the hopes that prices will
increase even further in the future.
(E) A significant rise in housing prices in a specific area may cause some potential buyers to relocate to other, less
pricey areas.

Answer: D
Type 4 Assumption Eliminates an Alternate Cause
The analyst claims that scarcity causes the price increase, but the reverse could also be true: the price increase could
cause the scarcity. Perhaps people wait to sell because they think the market will continue to rise, or perhaps people will
not sell their own property because they would then have to pay inflated prices for a new property. In order to conclude
that scarcity causes prices to rise, the analyst must assume that the "reverse" causation does not occur. Only Option D
denies reverse causation.

Slide 27: PRACTICE QUESTION #10

A group of college students participated in a sleep study where half of them slept for a full eight hours and the other
half were woken up two to three times during the night. The half that slept the full eight hours uninterrupted
performed better, on average, on a logical reasoning exam the next day than the other half whose sleep was
interrupted. Therefore, people have the best brain function when they sleep for a full eight hours, uninterrupted.

What are the three major flaws in the above argument?

This is an open ended question…THINK!

1. Sample of people: The people who were tested were all college students who, on average, are aged 18-22. However,
the conclusion is about people in general, which is a leap on logic.

2. Leap in Logic: A higher score on a logical reasoning test doesn’t necessarily mean a higher brain function. There are
lots of other parameters to consider.

3. The word best: The study discussed in the argument only tested two different sleep scenarios. Just because one is
better than the other, doesn’t necessarily mean that it is the best.

Page 52 of 91
VERBAL ABILITY

HAP_VA_BTECH_SEM_4_1218

CRITICAL REASONING
SESSION – 13

 Critical Reasoning questions involve reading brief arguments (each argument is generally one to three sentences
long) and answering questions relating to those arguments.
 In order to analyze arguments, it is important to understand their basic structure:
 Premises + (Assumptions) = Conclusion
 In words, premises and assumptions lead to a conclusion.
 PREMISES are STATED pieces of information or evidence that generally provide support for the given
conclusion.
 ASSUMPTIONS are UNSTATED parts of the argument that are NECESSARY to reach the given conclusion.
 The main point of the argument is the CONCLUSION, which is logically supported by the assumptions and
premises. Conclusions are in the form of an opinion or a claim.
 Here is an example of Critical Reasoning argument…

 There are two broad things we need to understand in order to answer Critical Reasoning questions effectively
and efficiently.

 We need to understand the specific information given for that question, and also how to conduct the necessary
reasoning to answer a question of this type.

Question Types
The four major question types in Critical Reasoning are as follows:

(1) Find the Assumption

(2) Draw a Conclusion

(3) Strengthen the Conclusion

(4) Weaken the Conclusion

 Notice that three of the four major question types involve finding the conclusion in order to answer the question,
and the fourth requires us to find the conclusion itself among the answer choices. Clearly, the conclusion is the
most important part of each argument!

Page 53 of 91
VERBAL ABILITY

HAP_VA_BTECH_SEM_4_1218

You may also encounter any of seven minor question types:


 Explain an Event or Discrepancy
 Analyze the Argument Structure
 Evaluate the Conclusion
 Fill in the Blanks
 Resolve a Problem
 Provide an Example
 Mimic the Argument
 Classify Statements as True/False based on Content

How to Approach Critical Reasoning Questions?


1. Read the question stem first, and determine what type of question you are facing.
2. Read the passage critically; analyze the basic components of the argument in light of the question.
3. Formulate a correct answer to the question in your head or on scratch paper.
4. Attack the answer choices until only one remains.

Statements and Inferences

Directions for Q1 and Q2: Select the correct alternative from the given choices.

1. Statement:
Country Y has sought the help of country X, to catch the notorious don. Because of this request, country X is
caught between the devil and the deep sea.
Which of the following can be inferred from the above statement?
(a) Country X does not want to antagonize Country Y or the don.
(b) The don is in country X.
(c) Country X can find out the whereabouts of the don.
(d) The don is involved in illegal activities.
2. Statement:
If we throw ethics to the winds and dance with the powerful, we must remember that we may step on the devil’s
tail. So let’s dance with caution.
Which of the following can be inferred from the above statement?
(a) Dancing with the powerful is necessary.
(b) One has to throw ethics to the winds if one wants to dance with the devil.
(c) The powerful are devils.
(d) None of these
Statements and Course of Action
Directions for Q3 and Q4: Select the correct alternative from the given choices.

3. Now-a-days, many sky scrapers are coming up, leaving no place for children to play. The children are forced to
play on the roads, sometimes causing accidents. Some parents do not allow their children to play, leading to lack
of physical exercise.

Page 54 of 91
VERBAL ABILITY

HAP_VA_BTECH_SEM_4_1218

Which of the following would be the most appropriate course of action to solve the problem?
(a) Parents should set up a gym in the house for the physical exercise of the children.
(b) The authorities should not allow building of skyscrapers so that the children will get a playground at
convenient distances.
(c) Children should play in school playgrounds.
(d) None of these
4. Software companies are setting up their offices in the city. Many people from various states are coming to the
city for the job. As the number of people in the city is increasing at a rapid pace, the people are facing problems
in finding residential dwellings. The rentals have doubled in one year in the city, despite having rent control acts.
Very few houses or hostels are available at affordable rents.
Which of the following would be the most appropriate course of action to solve the problem?
(a) Do not allow the software companies to enter the city.
(b) The authorities should enact one more rent control act.
(c) Authorities should build new buildings.
(d) None of these

Statements and Conclusions

Directions for Q5 and Q6: In each of the following questions a statement is given followed by two conclusions
I and II. Mark your answer as:
(a) If only conclusion I follows. (b) If only conclusion II follows.
(c) If neither I nor II follows. (d) If both I and II follow.

5. Statement:
The constitutional amendment carried out in the monsoon session of Parliament prohibits child labour in any
organization.
Conclusions:
I. All employers in India must abide by the new rule.
II. All the victims of child labour will now enroll in schools.

6. Statement:
Company ABC has an envious track record in manufacturing top quality cameras with the latest innovations,
which ensures that the end user gets excellent pictures even in bad weather conditions.
Conclusions:
I. No other company has got as much recognition as ABC in this sector.
II. Even a layman can take great photographs using the cameras made by ABC.

Directions for Q7 and Q8: In each question below is given a statement followed by possible consequences. Find
which of the consequence is most likely.

7. Statement:
Many houses collapsed and many families were left homeless due to a major earthquake that shook Lathur
district, which lies in a seismically sensitive zone.
Possible consequences:
(A) The Government may build a huge multi-storeyed building to give shelter to those who lost their houses.
(B) The government may provide make-shift arrangements for those who lost their houses.
(C) The people may relocate to other districts.
(a) Only A and B (b) Only B (c) Only A (d) Only B and C

Page 55 of 91
VERBAL ABILITY

HAP_VA_BTECH_SEM_4_1218

8. Statement:
The Meteorological Department cautioned the government that heavy rains and floods may occur in the next two
days.
Possible Consequences:
(A) The government may evacuate the people from low lying areas.
(B) The meterological department may make arrangements to supply food to the people in the areas that could
get affected.
(C) The government may collect money from the public to help the people who could get affected by the floods.
(a) All of A, B and C (b) Only A and B (c) Only A and C (d) Only A

Directions for Q9 and Q10: Read the following questions and answer accordingly.
Passage Analysis

9. Prolonged spells of hot, dry weather at the end of the grape-growing season typically reduce a vineyard’s yield,
because the grapes stay relatively small. In years with such weather, wine producers can make only a relatively
small quantity of wine from a given area of vineyards. Nonetheless, in regions where wine producers generally
grow their own grapes, analysts typically expect a long, hot, dry spell late in the growing season to result in
increased revenues for local wine producers.
Which of the following, if true, does most to justify the analysts’ expectation?
(a) The lower a vineyards’s yield, the lesser the labor required to harvest the grapes.
(b) Long, hot, dry spells at the beginning of the grape-growing season are rare, but they can have a devastating
effect on a vineyard’s yield.
(c) Grapes grown for wine production are typically made into wine at or near the vineyard in which they were grown.
(d) Grapes that have matured in hot, dry weather make significantly better wine than ordinary grapes.

10. In the past, most children who went sledding in the winter snow in Verland used wooden sleds with runners and
steering bars. Ten years ago, smooth plastic sleds became popular, they go faster than wooden sleds but are
harder to steer and slow. The concern that plastic sleds are more dangerous is clearly borne out by the fact that
the number of children injured while sledding was much higher last winter than it was 10 years ago.
Which of the following, if true in Verland, most seriously undermines the force of the evidence cited?
(a) A few children still use traditional wooden sleds.
(b) Very few children wear any kind of protective gear, such as helmets, while sledding.
(c) Plastic sleds can be used in a much wider variety of snow conditions than wooden sleds can.
(d) Most sledding injuries occur when a sled collides with a tree, a rock, or another sled.

Strengthening & Weakening An Argument

11. Until now only injectable vaccines against influenza have been available. They have been primarily used by older
adults who are at risk for complications from influenza. A new vaccine administered in a nasal spray form has
proven effective in preventing influenza in children. Since children are significantly more likely than adults to
contract and spread influenza, making the new vaccine widely available for children will greatly reduce the
spread of influenza across the population.

Which of the following, if true, most strengthens the argument?


(a) If a person receives both the nasal spray and the injectable vaccine, they do not interfere with each other.
(b) The new vaccine uses the same mechanism to ward off influenza as injectable vaccines do.
(c) Government subsidies have kept the injectable vaccines affordable for adults.
(d) Many parents would be more inclined to have their children vaccinated against influenza if it did not involve
an injection.

Page 56 of 91
VERBAL ABILITY

HAP_VA_BTECH_SEM_4_1218

Directions for Q12: Read the following questions and answer accordingly.
Passage Analysis
Strengthening & Weakening An Argument

12. Motorists in a certain country frequently complain that traffic congestion is much worse now than it was 20
years ago. No real measure of how much traffic congestion there was 20 years ago exists, but the motorists
complaints are almost certainly unwarranted. The country’s highway capacity has tripled in the last twenty
years, thanks to a vigorous highway construction program, whereas the number of automobiles registered in the
country has increased by only 75 percent.
Which of the following, if true, most seriously weakens the argument?
(a) Most automobile travel is local, and the networks of roads and streets in the country’s settled areas have
changed little over the last 20 years.
(b) Gasoline prices are high, and miles traveled per car per year have not changed much over the last 20 years.
(c) The country’s urban centers have well-developed public transit systems that carry most of the people who
commute into those centers.
(d) The average age of automobiles registered in the country is lower now than it was 20 years ago.

Statements and Assumptions

Directions for Q13: In each question below is given a statement followed by several assumptions. Find which of the
assumption is implicit in the statement.

13. Statement:
Radios are fast getting replaced by transistors.
Assumptions:
I. Transistors are superior to radios.
II. Government has banned manufacturing of radios.
(a) Only (I) is implicit (b) Only (II) is implicit
(c) Either (I) or (II) is implicit (d) Neither (I) nor (II) is implicit

Statements and Assumptions

Directions for Q14: In each question below is given a statement followed by several assumptions. Find which of the
assumption is implicit in the statement.

14. Statement:
“Do not allow any candidate to leave the examination hall until the examination is over, except when an
emergency calls for.” - An instruction to the supervisor.
Assumptions:
I. The supervisor himself is not supposed to leave the examination hall unattended.
II. The supervisor has the authority to determine the extent of emergency.
III. The examination is for less than two hours.
(a) Only (I) is implicit (b) None is implicit
(c) Only (II) is implicit (d) Only (I) & (II) are implicit

Page 57 of 91
VERBAL ABILITY

HAP_VA_BTECH_SEM_4_1218

Theme Detection
Directions for Q15 and Q16: Read each of the short passages given below and choose the option best representing
the theme of the passage.

15. The TRIPS agreement has, as its Preambular objective, a desire to ensure that measures and procedures to
enforce intellectual property rights do not themselves become barriers to legitimate trade. Further, one of its
objectives is to contribute to the transfer and dissemination of technology. It has, among its principles, the
promotion of public interest in sectors of vital importance to the socioeconomic and technological development
of its members. At the same time the agreement recognizes intellectual property rights as private rights. Finally,
the agreement encourages adjustments aimed at higher levels of protection of intellectual property rights.
(a) The TRIPS agreement is more show and less substance.
(b) The benefits of being a signatory to the TRIPS Agreement.
(c) The objectives of TRIPS Agreement.
(d) The deadlines of TRIPS Agreement.
16. During the last two decades, physicians and mental health professionals have begun to discover the limitations
of Western allopathic medicine. The focus is on pathology and disease and not on prevention. The
destructiveness of so many pharmaceutical and surgical remedies, the separation of physical and emotional
problems and the assumption of an asymmetrical, relationship between an all-powerful physician and a
submissive patient have led clinics and researchers to look for answers in other traditions and cultures.
(a) Western allopathic medicine has failed completely.
(b) People have realized the serious limitations of western allopathic medicine.
(c) The narrow focus of western allopathic practices has forced the medical fraternity to search for alternative
systems of health control.
(d) Traditional medicine will come to play an increasing role in the treatment of patients by assuming an
emotional relationship between the doctor and the patient.

Directions for Q17: Read the given statements and answer according to the questions given.

17. Scientists warn of a global warming, a ‘greenhouse effect’ resulting from increased atmospheric pollutants,
including carbon dioxide from the burning of wood, coal and oil. A coal-industry spokesperson says that the
effect need not cause concern in the near future if, as some scientists believe, the Earth faces another ice age
within the next thousand years since each calamity could cancel out the other.
Which of the following, if true, casts the most serious doubt on the conclusion of the spokesperson?
(a) There is a general cyclical pattern in the recurrence of ice ages on Earth.
(b) The disastrous results of the greenhouse effect have begun to occur and will probably intensify within the
next fifty years.
(c) Trees absorb some of the carbon dioxide in the lower atmosphere and produce oxygen, which is not a pollutant.
(d) Much of the carbon dioxide currently being produced comes not from coal but from the burning of trees
cleared from large areas of tropical rain forest.

Critical Reasoning Exercise


18. Insect Infestations in certain cotton growing regions of the world have caused dramatic increases of cotton on
the world market. Knowing that cotton plants mature quickly, many soybean growers in Ortovia plan to cease
growing soybeans, the price of which has long been stable and to begin raising cotton instead, thereby taking
advantage of the high price of cotton to increase their income significantly over the next several years.
Which of the following, if true, most calls into question the reasoning on which the plan is based?

Page 58 of 91
VERBAL ABILITY

HAP_VA_BTECH_SEM_4_1218

(a) The cost of raising soybeans has increased significantly over the past several years and is expected to
continue to climb.
(b) Tests of a newly developed, inexpensive pesticide have shown it to be both environmentally safe and effective
against the insects that have infected the cotton crops.
(c) In the past several years, there has been no sharp increase in the demand for cotton, and for goods made out
of cotton.
(d) Many consumers consider cotton cloth a necessity rather than a luxury and would be willing to pay
significantly higher prices for cotton goods than they are currently paying.
(e) The species of insect that has infested the cotton plants has never been known to infest soybean plants

19. Finding of a survey of Systems magazine subscribers: Thirty percent of all merchandise orders placed by
subscribers in response to advertisements in the magazine last year were placed by subscribers under age
thirty-five.
Finding of a survey of advertisers in Systems magazine: Most of the merchandise orders placed in response to
advertisements in Systems last year were placed by people under age thirty-five.
For both of the findings to be accurate, which of the following must be true?
(a) More subscribers to Systems who have never ordered merchandise in response to advertisements in the
magazine are age thirty-five or over than are under age thirty-five.
(b) Among subscribers to Systems, the proportion who are under age thirty-five was considerably lower last year
than it is now.
(c) Most merchandise orders placed in response to advertisements in Systems last year were placed by Systems
subscribers over age thirty-five.
(d) Last year, the average dollar amount of merchandise orders placed was less for subscribers under age thirty-
five than for those aged thirty-five or over.
(e) Last year many people who placed orders for merchandise in response to advertisements in Systems were
not subscribers to the magazine.

20. Which of the following most logically completes the passage?


Concerned about the well-being of its elderly citizens, the government of Runagia decided two years ago to
increase by 20 percent the government-provided pension paid to all Runagians over 65. Inflation in the
intervening period has been negligible, and the increase has been duly received by all eligible Runagians.
Nevertheless, many of them are no better off financially than they were before the increase, in large part because
______________.
(a) they rely entirely on the government pension for their income.
(b) runagian banks are so inefficient that it can take up to three weeks to cash a pension check.
(c) they buy goods whose prices tend to rise especially fast in times of inflation.
(d) the pension was increased when the number of elderly Runagians below the poverty level reached an all-time
high.
(e) in Runagia children typically supplement the income of elderly parents, but only by enough to provide them
with a comfortable living

Page 59 of 91
VERBAL ABILITY

HAP_VA_BTECH_SEM_4_1218

MIMIC THE PATTERN


SESSION – 14

ANALOGOUS REASONING/PARALLEL REASONING

In this section on “Analogous Reasoning” you will develop the ability to identify the analogy between two arguments.
Analogous arguments are also called Parallel arguments.

You will be given an argument that has a particular method of reasoning. Your task will be to find one argument from
a list of arguments that is analogous (parallel) to the given argument.

After the argument is presented, questions are posed as follows:

 The pattern of reasoning in the argument above is similar to which one of the following arguments?

 Which one of the following arguments is parallel in its structure to the above argument?

The correct answer is the one that has the same type of premises and the same type of conclusion as the given
argument. The two arguments must be analogous in structure but they do not have to use the exact same words. If the
given argument has a flaw in reasoning, the analogous argument also must have a flaw in reasoning. If the given
argument uses conditional reasoning, the correct answer must also have conditional reasoning. Incorrect answers are
those that are not analogous in premises or conclusion or both.

Example: (analogous premises and analogous conclusion)

Argument 1: If he is wise he must not jump from the bridge. He is wise and so he did not jump from the bridge.

Argument 2: If she is nice, she must not lock the door. She is nice and so she did not lock the door.

PARALLEL REASONING

In Parallel Reasoning questions you are given an argument and asked to identify the answer choice that most closely
parallels the argument. This means the two arguments will have the same structure but different concepts and ideas
discussed in them. A very simple example is the following:

Argument: All sharks have fins and all dolphins have fins therefore they are similar.

Answer choices:

(A) Sharks and dolphins are similar, however, they have many differences.

(B) All sharks have teeth, dolphins are similar to sharks therefore they must have teeth.

(C) Bats and eagles must be similar because every bat has wings and so does every eagle.

(D) All dogs have eyes and all cats have eyes.

(E) Some rats have fur and all mammals have fur, therefore, rats are mammals.

Which is the correct answer choice? In order to solve parallel reasoning questions you can ignore the specifics and
look only to the general structure. So ignore the terms like dolphins, fins, sharks, rats and bats. Break it down to a
basic structure and then look for the same in the answer choices.

The structure of the argument “All sharks have fins and all dolphins have fins therefore they are similar.” can be
broken down as follows:
ALL of entity X have attribute Y and ALL of entity Z have attribute Y. Therefore X and Z are similar.

Page 60 of 91
VERBAL ABILITY

HAP_VA_BTECH_SEM_4_1218

You can also break it down into a numbered list of elements that must be included in the parallel argument:

1. ALL of entity X have attribute Y

2. ALL of entity Z have attribute Y

3. Therefore X and Z are similar

If not all of these elements are present in the answer choice then it cannot be right.

Look for this same structure in the answers.

(A) This answer says they two entities are similar but have differences. It does not include the fact that they share
some attribute.

(B) Does not give us the premise that both entities share the same trait. It is instead used as a conclusion based on
the premise that the two entities are similar.

(C) This is the correct answer choice. It used different specifics, bats, eagles and wings instead of dolphins, sharks
and fins, however, it has the same basic structure. You have two entities that share the same trait and therefore
must be similar. Note that they do change the order of the argument, this is okay as long as the required
elements are present.

Element 3: Bats and eagles must be similar because

Element 1: every bat has wings

Element 2: and so does every eagle.

(D) This is missing our third element: Therefore X and Z are similar

(E) There are a number of differences here including that not all the given entities are said to have the given flaw,
only some. To be parallel in reasoning we must have all entities with the given trait.

A good way to approach Parallel Reasoning questions is the quickly write down a list of the required elements as we
did above. Do this in short hand and be quick. Then check each answer choice to see if it has all the required elements.

PRACTICE EXERCISE

1. All the cars, when considered together, use a lot of iron. So, Stanley’s car also uses a lot of iron.
The flawed pattern of reasoning in the argument above is similar to that in which one of the following?
(a) Joan loves democracy. So, the people of her country also love democracy.
(b) All the fruits are healthy to eat. So, the apples are healthy to eat.
(c) All the trucks, when considered together, pollute the air very much. So, Nancy’s truck also pollutes the air
very much.
2. If a person plays the piano, that person must be talented. Tina plays the piano and therefore she must be
talented.
The reasoning in the argument above is parallel in its structure to which one of the following?
(a) If a person drivers a car, that person must be a good driver. Sam does not drive a car and so he must not be
a good driver.
(b) If a person draws well, that person must be creative. Todd draws well and consequently he must be
creative.

3. Ian’s mom told him that if he studies for two hours today, then he can eat an ice-cream. Ian ate an ice-cream
today; so he must have studied for two hours today.
The flawed pattern of reasoning in the argument above is similar to that in which one of the following?

Page 61 of 91
VERBAL ABILITY

HAP_VA_BTECH_SEM_4_1218

(a) If the train maintains its speed, it will arrive at the station in time. The train arrived in time; so it must have
maintained its speed.
(b) If it snows, then the temperature will be cold. So, since it did not snow, the temperature was not cold.

4. Before going to the backyard, Vivian was warned by his dad that he should be careful about fire ants in the
yard. Vivian claimed that since his home was new, the backyard will not have fire ants. so, he ignored the
warning.
The reasoning in the argument above is analogous to which one of the following?
(a) Ronald was alerted by his mom not to get wet in the rain since he could damage his watch. But, Ronald
reasoned that his watch was water proof. Therefore, he dismissed the warning.
(b) Amber’s mom cautioned her to be careful while handling the electric drill. Amber argued that the new drill
was safe. So, she ignored the warning to be cautious.

5. All P are Q.
All Q are R.
Therefore, all P are R.
Which one of the following arguments has a reasoning similar to the argument above?
(a) All M are N.
If there is a N, then it is a O.
Therefore, all O are M.
(b) All A are B.
If there is a B, then it is a C.
Therefore, all A are C.

6. A doctor attending to a patient first noticed that he had a high temperature and after some time a high bold
pressure. So, the doctor concluded that the patient’s high temperature caused the high blood pressure.
The pattern of reasoning in the argument above is similar to that in which one of the following?
(a) The home owner noticed that when he walked up the stairs, the stairs swayed first and then there was a
squeaking noise. So, he concluded that the squeaking noise made the stairs to sway.
(b) When Lucy was driving her car, she first noticed a blue van and then after a minute, she witnessed an
accident. So, she conclude that the blue van caused the accident.

7. P, Q and R are events that happened at different times. P caused Q. The effect of Q is R. Therefore, R was caused
by P.
Which one of the following arguments has a reasoning similar to the argument above?
(a) L, M and N are events that happened at different times. L caused M to happen. M triggered N. Therefore, L
is the effect of N.
(b) I, J and K are events that happened at different times. J is the effect of I and J is the cause of K. Therefore, K is
the effect of I.

8. Of late, incidences of theft of watches have increased. So, people must prove ownership of their watches when
requested.
Which one of the following arguments has a reasoning similar to the argument above?
(a) In recent days, cars are being stolen in record numbers. Consequently, anyone must be able to prove
ownership of their vehicles when asked to do so.
(b) Lots of computers are being stolen these days. Therefore, people must show proof of ownership of their
computers on request.

Page 62 of 91
VERBAL ABILITY

HAP_VA_BTECH_SEM_4_1218

9. Rhonda was sure that facial cream will help remove her pimples and applied a lot of it on her face. But, the
pimples did not go away. So she blamed her pimples for the problem.
The pattern of reasoning in the argument above is similar to which one of the following?
(a) Alex was sure that the ant spray that he used will kill the ants in the kitchen. But even after spraying a lot,
the ants did not die. So, Alex blamed the ants for the problem.
(b) Bruce kept wiping the table hard with a grease remover because he wanted to get rid of the sticky
substance on the table. The sticky substance did not go away. So, Bruce blamed the grease remover for the
problem.

10. Most people like to lead peaceful lives. Hence, radio programs that deal with controversies must not be aired.
The reasoning in the argument above is analogous to which one of the following?
(a) Most children like to eat healthy food. So, food items that are spicy must not be served to them.
(b) Most people like humor. So, television programs that show serious content must not be telecast.

11. Everyone who thinks the Rams would win the championship thought that Jones would receive the award for
Most Valuable Player. But Jones did not receive the award for Most Valuable Player. Therefore, anyone who
believes the Rams will win the championship is wrong.
Which one of the following arguments contains flawed reasoning most similar to that in the argument above?
(a) Anyone who thinks that eating before exercising is a good idea has never taken a health class. But Jim has
never taken a health class and knows that he should not eat before exercising. Therefore, taking a health
class is not necessary for you to know eating before exercising is not a good idea.
(b) If you believe in fairies then you do not believe in vampires. Since Cindy believes in vampires, she cannot
believe in fairies.
(c) Anyone that believes that seagulls migrate based on advanced spatial recognition patterns believes that
most bird species have highly developed frontal cortexes. But is has been conclusively proven that most
bird species do not have highly developed frontal cortexes. Thus, the belief that seagulls migrated based on
advanced spatial recognition patterns is false.
(d) Anyone who believes animals deserve better treatment believes that animals are capable of moral
judgment. You do not believe that animals deserve better treatment so you do not think they are capable of
moral judgment.
(e) Anyone who thinks chickens are ugly thinks ducks are ugly. Since there is no reason to think ducks are ugly
there is no reason to think chickens are ugly.

12. While there are certain dog-training techniques that generally tend to be effective for most dogs, it is important
to keep in mind that each dog will respond particularly well to certain techniques that other dogs might not be
receptive to. Therefore, the best practice is to keep general principles in mind when training dogs, while
tailoring their educations to their particular traits.
Which of the following propositions does this reasoning above most closely conform to?
(a) While a general set of principles exist for accomplishing a goal, it is important to acknowledge the unique
circumstances or qualities that may exist, and to take them into consideration.
(b) Unique individuals and unique circumstances call for specialized training to meet their needs.
(c) Even though general principles are broad enough to encompass most individuals or circumstances, there
will always be outliers that require special attention.
(d) Although dogs are different, they are similar enough that a basic set of training principles can be applied to
them universally, with slight room for variation.
(e) It is dangerous to apply a broad set of principles to a set of unique individuals because they will react to the
same set of circumstances differently.

Page 63 of 91
VERBAL ABILITY

HAP_VA_BTECH_SEM_4_1218

13. Economist: All hamburger joints must offer fries and drinks to maximize their revenue. The Burger Shack is a
hamburger joint. Because it offers fries and drinks, its revenue is clearly being maximized.
The flawed reasoning in which one of the following is most similar to that in the economist’s argument?
(a) Successful campaigning relies on two factors: powerful action committees and sufficient exposure. Sarah
Strong was a congressional candidate in the last election. She must have had a powerful action committee
and sufficient exposure, because she ran a successful campaign.
(b) For a video game to sell well, it must include both a memorable character and lots of action. Arkham’s
Revenge is a video game that has been selling well. Therefore, it must have a memorable character and lots
of action.
(c) Every piano teacher needs to have a clear course of instruction and a patient personality to succeed.
Beverly, a piano teacher, has both of these, so she is undoubtedly successful.
(d) Every traffic jam is caused by two factors: impatient driving and lack of available roads. In the city of San
Calistranus, there are plenty of available roads and few impatient drivers. So, clearly, there are few traffic
jams in San Calistranus.
(e) It will never snow unless the temperature is below 40 degrees and there is sufficient humidity. On
Thursday there was sufficient humidity, but the temperature was 45 degrees, so it didn’t snow.

14. All limes are green. Therefore, any fruit that is green is a lime. If a person sees a fruit that is green, that person
may assume that the fruit is a lime.
Which of the following most closely parallels the flawed reasoning above?
(a) Liberal views are often expressed in immigration courts. Therefore, is a person encounters a liberal view, it
is likely that the view was expressed in an immigration court.
(b) Immigration courts have more liberal views. Therefore, liberal views are only expressed by immigration
courts. Therefore, if a person encounters an immigration court, that person can assume that the court will
have a liberal view.
(c) Liberal views are expressed in immigration courts. Therefore, immigration courts are liberal. If a person
encounters a liberal view, that person can assume that the view was expressed in an immigration court.
(d) Immigration courts tend to adopt more liberal views. Therefore, if a person encounters a conservative view,
it is unlikely that the view was expressed in an immigration court.
(e) Immigration courts adopt more liberal views. Therefore, any court that adopts a more liberal view is an
immigration court. If a person encounters a court that has a more liberal view, that person may assume that
the court is an immigration court.

15. If a wine receives a high score from the National Sommelier Association, it is more likely to be sold in fine-
dining restaurants. A new wine from Oregon just received a high score from the National Sommelier
Association. Therefore, it is more likely to be sold in fine-dining restaurants.
Which of the following choices most closely reflects the reasoning in the argument above?
(a) Using recycled materials is beneficial to the environment. Some newspapers only use recycled paper in
their products. Therefore, a newspaper is likely to be beneficial to the environment.
(b) Famous actors tend to appear in popular movies. Popular movies often feature famous actors. Therefore, it
is unlikely to find a popular movie without famous actors.
(c) Brighter colors attract more attention. The professor attracts a lot of attention. Therefore, he probably
tends to wear more bright colors.
(d) Older coins are more valuable. Anita has a coin believed to have originated from the colonial era, but lacks
any proof. Therefore, it is questionable whether her coin is valuable.
(e) Pearls that are larger than average are more likely to retail for a higher price. A pearl diver found a dozen
pearls that are much larger than average. Therefore, the pearls are more likely to retail for a higher price.

Page 64 of 91
VERBAL ABILITY

HAP_VA_BTECH_SEM_4_1218

16. All English Springer Spaniels have long hair. All Rottweiler’s have short hair. Each of Tina's dogs is a cross
between an English Springer Spaniel and a Rottweiler. Therefore, Tina's dogs have medium-length hair.
Which one of the following uses flawed reasoning that most closely resembles the flawed reasoning used in the
argument above?
(a) All cars made by Chord are very well made. All cars made by Fysler are very poorly made. Half of the cars
on Jim's lot are very well made and the other half are very poorly made. Therefore, half of the cars on Jim's
lot are Chords and half are Fyslers.
(b) All economists know linear algebra. All physicists know relativistic mechanics. Wilma is both an economist
and a physicist. Therefore, Wilma knows both linear algebra and relativistic mechanics.
(c) All typists who practice at least one hour per day can type one hundred words per minute. But some typists
who do not practice can also type one hundred words per minute. Mike, a typist, practices thirty minutes
per day. Therefore, Mike types fifty words per minute.
(d) All halogen gases are toxic to humans. All non-radioactive noble gases are non-toxic to humans. "Nobagen"
gas is a mixture of a halogen gas and a noble gas. Therefore, "nobagen" gas is moderately toxic to humans.
(e) All players on the Wildcats have brown hair. All players on the Razorbacks have red hair. Members of the
Moye family are on both the Wildcats and the Razorbacks. Therefore, some members of the Moye family
have brown hair and others have red hair.

17. A high school football coach has made public comments criticizing the decision by the football coaching staff of
the local university to not play their star quarterback. However, we should not listen to the high school coach’s
criticism. His high school football team has not won a game in several seasons.
The flawed reasoning above most closely resembles which of the following arguments?
(a) We should not heed the weather channel's warnings. They have failed to correctly predict the past twenty
rainy days.
(b) It is likely that the scholar plagiarized this paper because she has been known to plagiarize in the past.
(c) We should not listen to this art critic's negative comments because it is well known that the art critic is a
mediocre artist.
(d) We should not listen to the car salesperson because she has an incentive to ignore negative features of the
car in order to make commission off of a sale.
(e) We should not listen to the neurobiologist’s predictions about the future state of the economy because he
has no formal training in economics.

18. The Forestry Service has issued new warnings about forest fires, which hikers in the National Forest are
encouraged to read thoroughly. The Forestry Service believes that more than two-thirds of forest fires last year
were down to human error. Their hope is to seriously curtail the amount of fires for the coming year.
Which statement uses reasoning parallel to the reasoning used in the above statement?
(a) A non-profit organization providing relief to victims of natural disasters
(b) A school having a class that teaches teenagers safer driving practices in order to lower the number of car
wrecks
(c) A company creating a training video to explain new procedures in their warehouses
(d) A small business owner adding new pieces to his inventory to increase sales
(e) A football coach instituting a new formation to gain an advantage on opponents

Page 65 of 91
VERBAL ABILITY

HAP_VA_BTECH_SEM_4_1218

19. Few people have ever seen the initiation rituals of the secret society, as its leadership believes that if anyone
reports these deep mysteries then the society will cease to have an attraction.
The reasoning used by the secret society’s leadership is most closely paralleled by__________.
(a) a company enforcing a non-disclosure agreement about its new products
(b) a production company not committing itself to any specific future projects
(c) a government agency designating specific documents as classified material
(d) a magician not revealing how tricks are performed in order to keep attracting an audience
(e) a criminal organization threatening any members who reveal information to law enforcement

20. Poet: True sincerity is spoken from the heart, without filters. This form of expression contrasts sharply with
what is (sadly) the more common way of speaking: a manner in which communication is made to create a
comfortable situation, rather than to speak the truth.
Which of the following individuals best embodies the message that the poet is conveying?
(a) A writer, outraged that no publishing house has yet agreed to publish his manuscript, sends an angry email
to his agent in order to release his frustrations. The writer’s email accuses the agent of not advocating on
his behalf with enough zeal.
(b) A piano teacher, aggravated at his pupil’s refusal to practice, vents to a colleague that the pupil’s lack of
motivation has caused him to feel utterly unmotivated as well.
(c) A student, frustrated with the classroom preparation that she was given to tackle a problem set,
respectfully tells her professor that she feels that she was not adequately instructed on the material
necessary for her to complete her assignment.
(d) A mother, worried that her son will not earn good grades if he does not start studying, tells him, “If you
want to get into college, you really need to start doing your homework.”
(e) While a husband is out with his wife, he wants her to feel loved and appreciated. He tells her in complete
honesty that from the moment he saw her, he thought she was stunning and had felt immediately drawn to
her.

Page 66 of 91
VERBAL ABILITY

HAP_VA_BTECH_SEM_4_1218

ANSWER KEY

READING COMPREHENSION - I
SESSION – 1
1. (a) the concern managers have about low productivity on different week-days and how it affects their companies
2. (a) Monday
3. (a) Monday
4. (c) workers would be more productive if the work deadlines were readjusted.
5. (b) gives the reasons why Tuesday is a particularly productive day in the work week.
6. (b) live longer
7. (a) vegetables, fruit, soy, and whole grains
8. (b) low stress levels and spirituality
9. (c) 34.7
10. (b) their behavior

READING COMPREHENSION - II
SESSION – 2
1. (c) people have to cope with more difficulties in their personal and communal lives.
2. (a) the physical and psychological benefits of yoga and meditation.
3. (b) people focus on something that they can return to.
4. (b) A description of the places to go and procedures people can follow to learn meditation.
5. (c) The increasing popularity of meditation through the availability of on-line resources.
6. (c) attempted to stop a British invasion of America
7. (a) are disappearing daily
8. (c) has changed steadily over the last two centuries to meet the needs of the population.
9. (c) have disappeared as a result of the flow of the Mississippi.
10. (a) general agreement that a plan should be made

READING COMPREHENSION – III


SESSION – 3
1. Ans: (c) 2. Ans: (c) 3. Ans: (c) 4. Ans: (e)

5. Ans: (a) 6. Ans: (d) 7. Ans: (b) 8. Ans: (c)

9. Ans: (a) 10. Ans: (c) 11. Ans: (c) 12. Ans: (c)

13. Ans: (d) 14. Ans: (c) 15. Ans: (b) 16. Ans: (a)

17. Ans: (a) 18. Ans: (b) 19. Ans: (b) 20. Ans: (e)

21. Ans: (b) 22. Ans: (d) 23. Ans: (b) 24. Ans: (d)

25. Ans: (c) 26. Ans: (d) 27. Ans: (a) 28. Ans: (b)

Page 67 of 91
VERBAL ABILITY

HAP_VA_BTECH_SEM_4_1218

SENTENCE CORRECTION – I
SESSION – 4

1. Ans: [b]
“the tribute” to be replaced by “a tribute” – Article

2. Ans: [d]
“its” to be replaced by “their” – “Muslims” is subject for which pronoun is “their” – Subject-Pronoun Agreement

3. Ans: [c]
“was” to be replaced by “has been” as the action started in the 1600s and is still happening – Tenses

4. Ans: [d]
“have” to be replaced by “has” as the main subject is “the drainage” which is singular – Subject-Verb Agreement

5. Ans: [d]
“foot” to be replaced by “feet” – Noun Form

6. Ans: [d]
“killing” to be replaced by “kill” – Verb Form or Parallelism

7. Ans: [b]
“employs” to be replaced by “employ” as the main subject “police departments” is plural – Subject-Verb
Agreement

8. Ans: [d]
“grown up” to be replaced by “grew up” – Verb Form

9. Ans: [c]
“inventing” to be replaced by “invention” – Word Form

10. Ans: [d]


it should be “resulting from” and not “resulting” – Idioms and Phrases

11. Ans: [b]


worse than I had expected – Degree of Comparison

12. Ans: [a]


the candidates should be – Sentence Structure

13. Ans: [c]


he or she has access – “Anyone” is singular – Subject-Pronoun Agreement

14. Ans: [b]


must be – future action – Tenses

15. Ans: [c]


any difference between - the word “between” is used as two samples are being compared – Prepositional Use

16. Ans: [c]


the basis of his book about the places he visited - Tenses

Page 68 of 91
VERBAL ABILITY

HAP_VA_BTECH_SEM_4_1218

17. Ans: [d]


will have been mayor of Tallahassee – Tenses

18. Ans: [b]


had revised - Tenses

19. Ans: [e]


make the best pets

20. Ans: [b]


that these forms be submitted

21. Ans: [c]

22. Ans: [e]

23. Ans: [c]

24. Ans: [b]

25. Ans: [b]

26. Ans: [c]


– Subject-Pronoun Agreement – “whom” is only for people while “which” is used for things

27. Ans: [a]


– Subject-Verb Agreement – “each” is always singular

28. Ans: [b]


– All errors relate to use of Adjectives and(or) Adverbs

29. Ans: [a]


– “Suspicious” is an adjective used in the place of adverb

30. Ans: [d]


– Prepositional use – “of” is redundant

SENTENCE CORRECTION – II
SESSION – 5

1. Ans: [c]
“repeating” to be replaced by “repeated” – Verb used as Adjective

2. Ans: [c]
“decomposing” to be replaced with “decomposition” – “decomposing” can be used as verb or adjective and both
are unsuitable because the noun form is required here.

3. Ans: [a]
“Magnolia” to be replaced by plural form i.e. “magnolias” since the rest of the sentence is referring to plural
subject. Clue: use of the word “they”

4. Ans: [b]
Town is singular. Therefore, it should read as “rid the town of its (instead of their) rats”. – Subject-verb
agreement

Page 69 of 91
VERBAL ABILITY

HAP_VA_BTECH_SEM_4_1218

5. Ans: [a]
“on” to be replaced with “at”

6. Ans: [a]
“more famous” to be replaced by superlative “most famous” as there are three Gorgons.

7. Ans: [c]
“are decreasing” to be replaced by “is decreasing”; “the number of colleges” is treated as a singular entity –
Subject Verb Agreement

8. Ans: [d]
“their” to be replaced by “its”; Maginot Line is the main subject and it is singular – Subject-Pronoun Agreement

9. Ans: [a]
Buffaloes are countable; therefore, it should be “fewer” buffaloes.

10. Ans: [c]


Verb form “was” to be replaced with “is” because even now the handles extend in opposite directions.

11. Ans: [d]


making enemies - Parallelism

12. Ans: [e]


– Appropriate Quantifier

13. Ans: [e]

14. Ans: [d]


even though it rained heavily – Conjunction Usage – Joining Sentences

15. Ans: [c]


to make both ends meet - Idioms and Use of Unnecessary Preposition

16. Ans: [d]


words or sentences – Conjunctions either connect words or sentences; not a word and a sentence – “The word
“together” is redundant

17. Ans: [e]


refrain from travelling at night - Parallelism

18. Ans: [a]


didn’t they – Question Tags

19. Ans: [d]


– Logical comparison – Skill of basketball payers can be compared to only skill of tennis players and not the
players themselves.

20. Ans: [d]


to one another but also adopt – Paired conjunction

21. Ans: [d]

22. Ans: [c]

Page 70 of 91
VERBAL ABILITY

HAP_VA_BTECH_SEM_4_1218

23. Ans: [b]

24. Ans: [e]

25. Ans: [d]

26. Ans: [c]


“Like” is a preposition so can’t be followed by a subject and verb – “on” is used with dates

27. Ans: [c]


– Errors pertaining to idioms and phrases must be corrected as follows – Option (a) “prefer…to…” – Option (b)
“regarded as” – Option (d) “except for”

28. Ans: [d]


– Improper use of Reflexive Pronoun

29. Ans: [b]


– Subject-Verb Agreement – Main subject is “problem” which is singular

30. Ans: [a]


– Subject-Verb Agreement - With words that indicate portions like “percent”, “fraction”, “part”, “majority”,
“some”, “all”, “none”, if the object of the preposition is singular, use a singular verb. Else, plural.

SPOT THE ERROR – I


SESSION – 6

1. Ans: [a] 2. Ans: [c]


Have that it should….

3. Ans: [b] 4. Ans: [d]


were delete ‘to’ after belonged

5. Ans: [c] 6. Ans: [c]


delete ‘have’ had not got

7. Ans: [b] 8. Ans: [e]


Decided No error

9. Ans: [b] 10. Ans: [a]


Delete ‘that’ had obtained

11. Ans: [c] 12. Ans: [a]


Was Howsoever I admire

13. Ans: [d] 14. Ans: [b]


Before consideration

15. Ans: [c] 16. Ans: [d]


Is has bothered

17. Ans: [e] 18. Ans: [d]


No error later in life

Page 71 of 91
VERBAL ABILITY

HAP_VA_BTECH_SEM_4_1218

19. Ans: [d] 20. Ans: [c]


Social changes Luggage

21. Ans: [b] 22. Ans: [a]


Which are not Taking into consideration

23. Ans: [c] 24. Ans: [d]


remove ‘do not’ from the next day

25. Ans: [d] 26. Ans: [c]


in India’s they have been

27. Ans: [d] 28. Ans: [e]


had been No error

29. Ans: [b] 30. Ans: [a]


on an asset backed up

SPOT THE ERROR – II


SESSION – 7

1. Ans: [d]
Cottage industry, also (a) known as the putting-out system, was the initial phase of industrialization, (b)
although it was soon overshadowed by other systems of production (c) that (d) have become widespread
during the Industrial Revolution of the 19th century. (e) No error
(a) known: This is in a modifying phrase—an SAT favorite—so you should check that it’s next to the correct
noun. “Cottage industry” is what’s known as the putting-out system, so there’s no problem.
(b) although: This linking word shows a contrast. Should there be one? Yep. Move on.
(c) that: What does “that” refer to? A place or time? Nope—it’s a thing (“systems of production”), so it’s fine.
(d) have become widespread: This is a verb, so you should check both the subject and the tense. The “systems”
are plural, so “have” is correct in number (not “has”), but the time is wrong! It should say “became
widespread.”

2. Ans: [d]
The flower, after (a) fertilization (b) with the grains of pollen that (c) a bee inadvertently transports, will slowly
(d) perish and wither. (e) No error
(a) fertilization: a noun like this could possible be a word choice problem, but there doesn’t seem to be any
incorrect meaning here.
(b) with: A preposition! Check the words that the preposition is combining (especially the one that comes before
it) and make sure that they sound natural together. Although “fertilization with” might sound a little strange, if
you said, “They fertilized the flower with pollen,” that’d be just fine, so it’s probably acceptable here.
(c) a bee inadvertently transports: the most likely problem here is a number issue. “A bee” has to be parallel
with anything that it logically relates to. In this case, it’s related to “the flower.” Both are singular, so it’s
alright. You should also look at the tense of the verb transports, which is just fine, in this case.
(d) perish and wither: This one is a bit hard to see when looking systematically, because it doesn’t have to do
with what part of the sentence is underlined. This is about redundancy; “perish” and “wither” mean
basically the same thing. You don’t need them both.

Page 72 of 91
VERBAL ABILITY

HAP_VA_BTECH_SEM_4_1218

3. Ans: [c]
Art and culture flourished (a) briefly during the Prague Spring, (b) a short period (c) when relaxed censorship
and (d) looser Soviet control. (e) No error
(a) briefly: Notice the “-ly” in the word. Should it be there (should this be an adjective instead)? It’s correct as it
is.
(b) a short period: check that the number of the noun is right, which it is. There was only one Prague Spring.
(c) when: This is a connecting word, so make sure that it makes the right relationship between sentences. Wait
a minute… sentences? The second part doesn’t form a complete thought. There should be a subject after
“when.” Or it could be changed into a preposition like “of,” which doesn’t connect whole thoughts (a.k.a.
clauses) like “when” does.
(d) looser: Check if it should be a comparative (-er) or a superlative (-est). There are only two things being
compared—Soviet control before and during the Prague Spring—so “-er” is correct.

4. Ans: [a]
Having died young, Raymond Carver’s career as a short-story writer was cut regrettably (b) short, and (c) we
are left with only a fragment of his only (d) attempt at a novel. (e) No error
(a) Having died young,: Alarm bells should be ringing. Like choice (a) in question number 1, this is a modifying
phrase. The SAT loves to put these at the beginning of sentences next to subjects that they can’t modify.
Raymond Carver’s career didn’t die young. Carver died young.
(b) short: Check the comma here to see if it’s making a run-on sentence. Since the word “and” comes just after
it, there’s no problem.
(c) we: Is “we” the right number and case here? Should it be “I” or “us”? No problems there, and it does work
alright as a general pronoun, referring to the public rather than any specific people, so move on.
(d) attempt at: This is a prime place to think twice about the preposition. Should it be “attempt on”? How about
“attempt to”? Nope. Sounds pretty good as it is.

5. Ans: [b]
The defendant (a) surprised the jury not only with his candor (b) as well as with (c) what seemed to be genuine
goodwill (d) behind his smile. (e) No error
(a) surprised: Find the subject (defendant), and see that that’s okay. Then check the tense. Doesn’t cause any
problems.
(b) as well as: Does this link correct forms together? Although “with his candor” and “with what seemed to
be…” are parallel, the phrase “not only” that came before should jump out at you. Any time you see “not
only” you should find a “but also,” and here, there isn’t one. No good.
(c) what: Should this be “who,” “when,” or “where”? Nope? Okay.
(d) behind: Match this preposition up with the things it joins. “goodwill behind his smile” sounds fine, because
“behind” and “smile” go well together.

6. Ans: [e]
Exotic pet enthusiasts prize male peacocks (a) for their feathers (b) because the males are the (c) more colorful
of the two (d) sexes; in contrast, female peacocks are mostly for breeding purposes. (e) No error
(a) for: The preposition “for” matches just fine with “prized,” so this is okay.
(b) because: This works to connect the two complete thoughts. There’s no problem in the logic, either, since it’s
a cause-and-effect relationship.

Page 73 of 91
VERBAL ABILITY

HAP_VA_BTECH_SEM_4_1218

(c) more colorful: Although “the more colorful” might sound weird, and you might want to change it to “most,”
that wouldn’t make sense. There are only two things being compared here—males and females—so we
need to keep “more” as it is.
(d) sexes; in contrast,: The semi-colon has to join up two full thoughts (it acts like a period), which it does, here.
“In contrast” is one of many introductory words like “however” which need to be followed by commas, so
that’s also fine.

7. Ans: [c]
Skill: Managing word choice and grammatical relationships between words
Corrected Sentence:
Seventy-five percent of what we perceive as taste, especially the perception of flavor, is really attributable to
our sense of smell.
Explanation for Correct Answer (c): The error in this sentence occurs at (c), where there is an unnecessary shift
in pronoun, from the first-person plural "we" to the indefinite "one."
Explanation for Incorrect Answer (a): There is no error at (a). The preposition "as" appropriately links the verb
"perceive" and the noun "taste."
Explanation for Incorrect Answer (b): There is no error at (b). The adverb "especially" is a necessary part of the
explanatory adverbial phrase, "especially the perception of flavor."
Explanation for Incorrect Answer (d): There is no error at (d). The prepositional phrase "of smell" logically
follows and completes the noun that precedes it, "sense."
Explanation for Incorrect Answer (e): There is an error in the sentence.

8. Ans: [a]
Skill: Managing word choice and grammatical relationships between words
Corrected Sentence:
Development of new drugs often requires years of testing and waiting for labeling approval from federal
regulators.
Explanation for Correct Answer (a): The error in this sentence occurs at (a), where there is subject-verb
disagreement. The plural verb "require" does not agree with its singular subject, "Development."
Explanation for Incorrect Answer (b): There is no error at (b). The preposition "of" is a necessary part of the
prepositional phrase "of testing and waiting."
Explanation for Incorrect Answer (c): There is no error at (c). The preposition "for" is a necessary part of the
prepositional phrase "for labeling approval."
Explanation for Incorrect Answer (d): There is no error at (d). The preposition "from" is a necessary part of the
prepositional phrase "from federal regulators."
Explanation for Incorrect Answer (e): There is an error in the sentence

9. Ans: [a]
Skill: Managing phrases and clauses in a sentence
Corrected Sentence:
Although research in the earth and environmental sciences has pieced together narratives of ancient and
historical environmental changes, there is still much left to learn about the nature and causes of changing
climatic conditions through time.

Page 74 of 91
VERBAL ABILITY

HAP_VA_BTECH_SEM_4_1218

Explanation for Correct Answer (a): The error in this sentence occurs at (a), where there is redundancy. The
opening word of the sentence, "Although," signals a contrast between its clause and the clause to follow.
Therefore, "but, " another contrast word, is not needed because the contrast has already been created by
"Although."
Explanation for Incorrect Answer (b): There is no error at (b). The pronoun "much" appropriately modifies the
noun phrase "left to learn."
Explanation for Incorrect Answer (c): There is no error at (c). The infinitive verb phrase "to learn" is an
appropriate and necessary part of the noun phrase "much left to learn."
Explanation for Incorrect Answer (d): There is no error at (d). The participial adjective "changing" properly
modifies the noun phrase "climatic conditions."
Explanation for Incorrect Answer (e): There is an error in the sentence.

10. Ans: [c]


Skill: Managing phrases and clauses in a sentence
Corrected Sentence: During the reign of Elizabeth I, dozens of commissions were sent out by the central
government to investigate the wood shortage around the nation, and each one confirmed the serious decline of
the forests.
Explanation for Correct Answer (c): The error in this sentence occurs at (c), where there is improper
coordination. Two independent clauses ("dozens of commissions...around the nation" and "each one...forests")
are connected by only a comma, without the conjunction "and" to introduce the second independent clause.
Explanation for Incorrect Answer (a): There is no error at (a). The preposition "During" is an appropriate and
necessary part of the prepositional phrase "During the reign of Elizabeth I."
Explanation for Incorrect Answer (b): There is no error at (b). The infinitive verb phrase "to investigate"
properly completes the verb phrase "were sent out" by explaining why the commissions were sent out.
Explanation for Incorrect Answer (d): There is no error at (d). The adjective "serious" properly modifies the
noun "decline."
Explanation for Incorrect Answer (e): There is an error in the sentence.

11. Ans: [d]


Skill: Managing phrases and clauses in a sentence
Corrected Sentence:
While sperm whales are usually seen in offshore waters, they can also be found near the shore, where the
continental or island shelf is narrow and the water is deep.
Explanation for Correct Answer (d): The error in this sentence occurs at (d), where there is an error in
parallelism. The coordinating conjunction "and" requires that the grammatical structures on either side of it be
parallel. A clause ("the continental or island shelf is narrow") precedes "and," so another clause ("the water is
deep") should follow it.
Explanation for Incorrect Answer (a): There is no error at (a). The subordinating conjunction "While" properly
introduces the clause "sperm whales are usually seen in offshore waters."
Explanation for Incorrect Answer (b): There is no error at (b). The verb phrase "are usually seen" properly
follows the subject "sperm whales."
Explanation for Incorrect Answer (c): There is no error at (c). The relative pronoun "where" properly connects
the prepositional phrase "near the shore" with the clause "the continental or island shelf is narrow."
Explanation for Incorrect Answer (e): There is an error in the sentence.

Page 75 of 91
VERBAL ABILITY

HAP_VA_BTECH_SEM_4_1218

12. Ans: [b]


Skill: Managing phrases and clauses in a sentence
Corrected Sentence:
A farmer's field can have more than 50,000 weed seeds per square meter buried beneath the soil; surface seeds
buried more than about one centimeter below the soil surface do not receive enough light to germinate.
Explanation for Correct Answer (b): The error in this sentence occurs at (b), where there is a lack of
punctuation between two sentences. "Surface seeds...below the soil surface" is a new subject, and "do not
receive" is a new verb, so a new independent clause is present. But no punctuation or conjunction separates
this new clause from the preceding independent clause, resulting in a run-on sentence.
Explanation for Incorrect Answer (a): There is no error at (a). The phrase "per square meter" is appropriate
and necessary to complete the phrase "50,000 weed seeds."
Explanation for Incorrect Answer (c): There is no error at (c). The preposition "below" is an appropriate and
necessary part of the prepositional phrase "below the soil surface."
Explanation for Incorrect Answer (d): There is no error at (d). The determining adjective "enough" is an
appropriate and necessary part of the noun phrase "enough light."
Explanation for Incorrect Answer (e): There is an error in the sentence.

13. Ans: [d]


Skill: Managing phrases and clauses in a sentence
Corrected Sentence:
In 1952, when the Federal Communications Commission lifted its ban on new television station licenses, there
were 105 stations and 15 million households with television sets, but by 1956 the number of stations had
grown to almost 500 and the number of households with sets to nearly 35 million.
Explanation for Correct Answer (d): The error in the sentence occurs at (d), where there is an improper idiom.
The coordinating conjunction "and" requires parallel grammatical structures on either side of it, so the phrase
"at nearly 35 million" should be "to nearly 35 million," to be parallel with the phrase that precedes "and" ("to
almost 500").
Explanation for Incorrect Answer (a): There is no error at (a). The clause "there were" properly introduces the
noun phrase "105 stations and 15 million households with television sets."
Explanation for Incorrect Answer (b): There is no error at (b). The conjunction "but" properly shows a contrast
between the clause that precedes it and the clause that follows it, and the preposition "by" is an appropriate
and necessary part of the prepositional phrase "by 1956."
Explanation for Incorrect Answer (c): There is no error at (c). The past-tense verb phrase "had grown" is
consistent with the tense of the rest of the sentence.
Explanation for Incorrect Answer (e): There is an error in the sentence.

14. Ans: [e]


Skill: Because this sentence is correct as written, none of the four writing skills can be appropriately assigned to
this question.
Corrected Sentence:
Halley's comet is a potato-shaped lump about ten miles long with a mass estimated at 10 billion tons, most of
which is water ice.
Explanation for Correct Answer (e): There is no error in this sentence.
Explanation for Incorrect Answer (a): There is no error at (a). The adjectival phrase "about ten miles long"
properly modifies the noun phrase "a potato-shaped lump."

Page 76 of 91
VERBAL ABILITY

HAP_VA_BTECH_SEM_4_1218

Explanation for Incorrect Answer (b): There is no error at (b). The prepositional phrase "with a mass" properly
modifies the noun phrase "a potato-shaped lump."
Explanation for Incorrect Answer (c): There is no error at (c). The words "estimated at" are an appropriate and
necessary part of the participial verb phrase "estimated at 10 billion tons."
Explanation for Incorrect Answer (d): There is no error at (d). The noun phrase "most of which" properly refers
back to the noun "mass."

15. Ans: [b]


Skill: Managing word choice and grammatical relationships between words
Corrected Sentence: The Neuse River in North Carolina is thought to be about two million years old, and there
have likely been human settlements in the basin for at least 14,000 years.
Explanation for Correct Answer (b): The error in this sentence occurs at (b), where there is a lack of agreement
between subject and verb. The plural noun "human settlements" requires the plural verb "have," not "has."
Explanation for Incorrect Answer (a): There is no error at (a). The words "thought to be" appropriately
complete the verb "is," forming the verb phrase "is thought to be."
Explanation for Incorrect Answer (c): There is no error at (c). The preposition "for" is an appropriate and
necessary part of the prepositional phrase "for at least 14,000 years."
Explanation for Incorrect Answer (d): There is no error at (d). The adjectival prepositional phrase "at least"
properly modifies the noun phrase "14,000 years."
Explanation for Incorrect Answer (e): There is an error in the sentence.

16. Ans: [d]


Skill: Managing word choice and grammatical relationships between words
Corrected Sentence: Shortly after Queen Eleanor arrived in Nottingham in 1257 for a visit, she fled the town
because she could not stand the smell of coal smoke and feared for her health.
Explanation for Correct Answer (d): The error in this sentence occurs at (d). The verb phrase "was afraid" is
redundant with "and feared for." It should be deleted from the sentence.
Explanation for Incorrect Answer (a): There is no error at (a). The adverbial phrase "Shortly after" correctly
modifies the verb "arrived."
Explanation for Incorrect Answer (b): There is no error at (b). The adverbial prepositional phrase "for a visit"
appropriately modifies the verb "arrived" by telling why the Queen arrived.
Explanation for Incorrect Answer (c): There is no error at (c). The subordinating conjunction "because" is an
appropriate and necessary part of the subordinate clause "because she could not stand the smell of coal
smoke."
Explanation for Incorrect Answer (e): There is an error in the sentence.

17. Ans: [a]


Skill: Managing grammatical structures used to modify or compare
Corrected Sentence: The great white shark is the most infamous of all sharks, the star of Peter Benchley's
bestselling novel Jaws, four Hollywood movies, and many television specials.
Explanation for Correct Answer (a): The error in this sentences occurs at (a), where there is an incorrect
adjective form. The prepositional phrase "of all sharks" calls for the superlative form of the adjective ("most")
rather than the comparative form "more," which would be appropriate if only two sharks were being
compared.
Explanation for Incorrect Answer (b): There is no error at (b). The prepositional phrase "of all sharks" logically
follows and completes the subject complement ("the...sharks") that follows the "be" verb.

Page 77 of 91
VERBAL ABILITY

HAP_VA_BTECH_SEM_4_1218

Explanation for Incorrect Answer (c): There is no error at (c). The words "star of" are a necessary part of the
appositive phrase ("the star of...specials").
Explanation for Incorrect Answer (d): There is no error at (d). The adjective "many" properly modifies the noun
phrase "television specials."
Explanation for Incorrect Answer (e): There is an error in the sentence.

18. Ans: [b]


Skill: Managing word choice and grammatical relationships between words
Corrected Sentence: The San Andreas Fault in California is just one section of an active circle of earthquake
zones and volcanoes known as the "Ring of Fire" that surrounds the Pacific Ocean.
Explanation for Correct Answer (b): The error in this sentence occurs at (b), where there is a use of an
improper idiom. The preposition "to" is used to follow the noun "section" where it would be more idiomatic to
use the preposition "of."
Explanation for Incorrect Answer (a): There is no error at (a). The adverb "just" properly modifies the adjective
"one."
Explanation for Incorrect Answer (c): There is no error at (c). The participial phrase "known as" properly
modifies the noun phrase "an active circle of earthquake zones and volcanoes."
Explanation for Incorrect Answer (d): There is no error at (d). The singular verb "surrounds" agrees with its
subject, "circle," and its tense is also consistent with that of the rest of the sentence.
Explanation for Incorrect Answer (e): There is an error in the sentence.

19. Ans: [c]


Skill: Managing word choice and grammatical relationships between words
Corrected Sentence: Relying on dams for large-scale water storage and for delivering water to places where it
does not naturally occur has long-term effects on the balance between groundwater and surface water and on
the quality of the surrounding soil.
Explanation for Correct Answer (c): The error in this sentence occurs at (c), where there is lack of subject-verb
agreement. The subject of the verb is "relying," so a singular verb, "has," is required.
Explanation for Incorrect Answer (a): There is no error at (a). The prepositional phrase "for delivering water"
is appropriately parallel to the preceding prepositional phrase "for large-scale water storage."
Explanation for Incorrect Answer (b): There is no error at (b). The relative pronoun "where" properly links the
noun "places" to the clause "it does not naturally occur."
Explanation for Incorrect Answer (d): There is no error at (d). The preposition "between" is an appropriate and
necessary part of the prepositional phrase "between groundwater and surface water."
Explanation for Incorrect Answer (e): There is an error in the sentence.

20. Ans: [e]


Skill: Because this sentence is correct as written, none of the four writing skills can be appropriately assigned to
this question.
Corrected Sentence: Despite a protest by 300 angry drivers, the five members of the bridge authority voted
unanimously to raise the passenger-car toll on the bridge from $1.00 to $2.00.
Explanation for Correct Answer (e): There is no error in this sentence.
Explanation for Incorrect Answer (a): There is no error at (a). The preposition "Despite" is a necessary part of
the prepositional phrase "Despite a protest."

Page 78 of 91
VERBAL ABILITY

HAP_VA_BTECH_SEM_4_1218

Explanation for Incorrect Answer (b): There is no error at (b). The preposition "by" is an appropriate choice to
follow the noun "protest," and it is a necessary part of the prepositional phrase "by 300 angry drivers."
Explanation for Incorrect Answer (c): There is no error at (c). Nothing in the sentence suggests that the verb
"voted" is in error in tense or in any other aspect. The word "unanimously" also fits the sentence, following
upon previous information ("five members of the bridge authority").
Explanation for Incorrect Answer (d): There is no error at (d). The infinitive phrase "to raise" logically follows
and completes the verb "voted."

21. Ans: [c]


‘More sharper’ is wrong. We need an adverb to modify the phrasal verb ‘turned down’ – correct form is ‘more
sharply’

22. Ans: [e]

23. Ans: [a]


Preposition ‘for’ in part (a) should be followed by a noun or noun substitute. So ‘improving the railway safety’
should be the right noun phrase.

24. Ans: [b]


In part (a) ‘nearly lost’ implies Vardhini won the match. Therefore in part (b) instead of ‘and failed to qualify’
we should have ‘but qualified herself’. This is a logical error.

25. Ans: [c]


(c) Noun proceeds are always used in the plural sense. So we should have 62 percent of the proceeds have been
Note:
Plural verbs are required for nouns that have no singular form, such as proceeds, goods, ashes, remains,
credentials, premises, etc.
The proceeds of the magic show are to be given to the fund for soldier’s welfare.
The goods are being dispatched today by goods train.

26. Ans: [c]


(c) When capable is used for referring to capacity it should be used as ‘capable of’

27. Ans: [e]

28. Ans: [b]


Replace ‘have’ with ‘has’.
‘More than one' is always followed by singular noun and singular verb.
The correct sentence should be: More than one student has taken the interview for this company.
e.g. More than one room is vacant.
Note that, in case of, ‘More + plural noun + than one’, verb is also plural.
e.g. More students than one are late.

29. Ans: [b]


Replace ‘is’ with ‘are’. When subject of the sentence is – ‘many’, verb is always plural.
The correct sentence should be: The Manager says that there are many important details to attend before this
gets launched.
When following words are used as subject in a sentence, verb is always plural.

Page 79 of 91
VERBAL ABILITY

HAP_VA_BTECH_SEM_4_1218

30. Ans: [b]


'Equally' and 'successful as' cannot be used together.
The correct sentence should be: The science project was as successful as the software project they had
presented.
e.g. Ravi and Manish are equally successful.
Ravi is as successful as Manish.

SENTENCE IMPROVISATION
SESSION – 8

1. Ans: [b]
People is plural therefore, ‘have left this place’ is correct.

2. Ans: [a]
Opportunities are termed as ‘lifetime opportunity’.

3. Ans: [d]
‘Last week’ dictates simple past tense ‘were.’

4. Ans: [e]
‘For three hours’ shows the duration of time spent.

5. Ans: [c]
Eventually led.

6. Ans: [a]
Two comparative degrees cannot be used together.

7. Ans: [d]
To affect means to influence.

8. Ans: [b]
The sentence talks about something which has already taken place.
Therefore ‘had played strategically’ is correct.

9. Ans: [e]

10. Ans: [c]


Split infinitive (to immediately see) should be avoided.

11. Ans: [a]


The sentence says natural calamities are threat to farmers.

12. Ans: [e]


It is an idiom.

13. At issue is the need for logical and formal parallelism in a coordinate series. B, the best choice, clearly and
correctly uses parallel noun phrases to list three effects of a drop in oil prices: a lowering of..., a rally in ..., and a
weakening of.... In place of the correct lower before/ears, choice A uses an incorrect participial adjective,

Page 80 of 91
VERBAL ABILITY

HAP_VA_BTECH_SEM_4_1218

lowering, that could cause confusion by seeming at first to function as a verb. A also violates parallelism. In C and
D, the use of along with confuses meaning by making fears about inflation an independent effect, not an object of
lowering. D and E violate parallelism by substituting an awkward gerund clause for the first noun phrase.

14. In choice D, the best answer, the phrase contemporaries of Harriet Tubman presents a complete possessive
without adding an apostrophe (e.g., Tubman's). Choices A, B, and C use a redundant possessive:
contemporaries of Harriet Tubman's. All choices other than D have errors in verb tense. Because the sentence
describes essentially simultaneous actions completed in the past, the simple past tense formsmaintained and
had are required. Thus, the present tense forms has and maintain are incorrect in A, B, and E, as are. the
present perfect have maintained in C and the past perfect had maintained in E.

15. A, the best choice, conveys the relevant information clearly and directly. Because the focus of interest is the
sales of new small boats, that should be the subject of the sentence. Since the period of time covered began
and ended in the past, the verb should be in the simple past tense (increased). The adverb annually fits most
logically after the amount of the increases. B, C, D, and E all distort the focus and disrupt the sensible order of
ideas. In addition, B, C, and D use incorrect verb tenses to refer to the simple past (is, have increased, and has
occurred). In C, the expression five and ten percent makes no sense without the word between. Finally, E is
especially clumsy and confused.

16. Choice E is best; it best indicates purpose for crossbreeding-- partly to acquire. In A, in part that does not
grammatically connect the underlined portion to the first part of the sentence (the independent clause). In both
A and B, in part is not parallel with and partly in the nonunderlined portion. Choice C causes a misreading,
suggesting that the steers' acquisition has caused the crossbreeding. D awkwardly and illogically shifts to the
passive voice: certain characteristics should be acquired by their steers; the steers, however, are not agents
in the acquisition.

17. The main challenge in this sentence is to observe the agreement of subject and verb (the resulting flow
pattern... is known...) despite the distraction of a complex intervening structure containing several plural
elements (with crests and troughs...). Choices A, B, and D can, therefore, be eliminated because they use an
incorrect plural verb form, are. Choice E uses the correct verb form, is, but it incorrectly introduces a
dependent adverbial although clause into a prepositional phrase (with crests ...). Choice D also makes this
error. Such dependent clauses can only occur in the predicates of full clauses. C, the best choice, uses the correct
verb form, is, and correctly puts the although clause inside the predicate of the relative clause (that... rapidly).

18. At issue is a comparison of Auden's language with Merrill's language. Only C, the best choice, uses the elliptical
like Auden's (language being understood), to compare Auden's language with Merrill's language. A, B, and D
compare Auden (the person) with Merrill's language. Choice E is awkward and unidiomatic.

19. A, the best choice, correctly balances the contrasting terms low and high in parallel form (adjectives in the
positive degree). It also makes clear who, exactly, is preparing for the coming school year (companies). B uses
the plural pronouns their and they without an appropriately stated referent. C, D, and E violate the parallelism
needed for the contrasting terms by making the second term an adjective in the comparative degree (higher).
Furthermore, the use of higher without a stated point of comparison makes it unclear what the expenses are
higher than. E also uses the pronoun their without an appropriate referent.

20. Only E, the best choice, clearly states that teratomas consist of tissues such as tooth and bone, and that such
tissues are not normally found in the organ with the teratoma. Clear statement of this fact requires the
repetition of tissues to establish the appositive--tissues normally found.... Without such repetition, A and B
imprecisely state that the tooth and bone, as opposed to the tissues, are not normally found in the affected
organ. Choices B and C alter the meaning with the use of like', that is, they suggest that the tissues are not tooth
and bone, but only like them. The confused syntax of D states that their composition, not the tissues, is found
in the organ....

Page 81 of 91
VERBAL ABILITY

HAP_VA_BTECH_SEM_4_1218

21. The sentence contains a relative clause (that...) indicating, in its compound predicate, two effects of the
immigration legislation: (it) would grant x and (would) penalize y. The auxiliary would may be omitted
before penalize, but the main verbs must remain parallel. Only C, the best choice, observes these conditions. A
and B produce incoherent, fused sentences in which the two main clauses are not parallel. Furthermore, in A
the referent of they is unclear, and in B the statement hiring illegal aliens would be a penalty makes no sense.
D violates parallel structure by substituting a present participle (penalizing) for the second main verb. E
introduces an incoherent passive infinitive construction that violates sense and parallel structure.

22. Choice A, the best answer, preserves grammatical parallelism while allowing for logical expression of temporal
relationships; A employs the parallel participial phrases spawned... and extending ... to modify filigree.Other
choices present different grammatical constructions that are not participial modifiers and thus not parallel to
spawned: extends in B is a present-tense verb; it extended in D begins a new clause; and is extending in E
ungrammatically introduces a new predicate. In C, extended is nonparallel if it is assumed to be a past tense
verb form; if it is assumed to be a past participle, it illogically states, as does D, that the filigree extended only in
the past.

23. Two instances of subject-verb agreement must be observed in this sentence: The period ... has been
established and what is much more difficult to determine ... is. Both clauses have singular subjects and must
have singular verbs. Only B, the best choice, observes these requirements. A incorrectly uses the plural form
are in the second clause. Choices C and D incorrectly use the plural form have in the first clause, and D
incorrectly uses are in the second clause as well. E incorrectly uses the plural form are in the first clause.
Furthermore, because the date of the period in question was established before the writing of the sentence, the
verb of that clause must be in the present perfect form (has been established).

24. The best choice, A, offers an adjective phrase unequivocally modifying policy and exhibiting grammatical
parallelism (decreasing ... and improving). In choice B, the gerund the decreasing is not grammatically
parallel with the infinitive to improve. Likewise, in C and D, the decreasing of... costs is not parallel with
improving the efficiency. In E, the infinitives to decrease and to improve, while parallel, are less idiomatic
than the prepositional phrase of decreasing ... and improving in modifying the noun aim. Also, with the aim...
improve can easily be construed as referring to the Baldrick Manufacturing Company and so does not refer
unequivocally to policy.

25. Choices A, C, and D contain singular verbs that do not agree in number with the plural subject, papers.
Furthermore, A violates parallelism by aligning the adjective important with the noun defense; C, employing
the present progressive tense, wrongly suggests that the triple authorship of The Federalist papers is a
developing situation rather than an accomplished fact; and D, employing the present perfect tense, suggests
that the situation of triple authorship is no longer the case. D is also garbled syntactically because the
conjunction and has been misplaced. In E, the wording is awkward. Choice B is best.

26. In choices A and B, the pronoun it simultaneously refers forward to someone (or a person) and backward to
the term "psychopath" As a result, the sentence asserts illogically that the term is actually a kind of person
rather than a word referring to a kind of person. Choice C repeats this fault and adds an error in agreement:
they (plural) does not agree in number with the term (singular). E omits a main verb, such as applied, that, in
grammatical context here, is required after is. Also, the word people incorrectly shifts number from singular to
plural. In choice D, the best answer, the verb refers is correctly used after it, and the alignment of pronouns and
antecedents is both logical and grammatical.

Page 82 of 91
VERBAL ABILITY

HAP_VA_BTECH_SEM_4_1218

27. Choice D, the best answer, appropriately uses the adverb so to refer back to the verb accord. The other choices
inappropriately use pronouns (it or this) to refer back to the verb. Also, A and B use the indicative verb
rewards, whereas the logic of the sentence demands the conditional would reward (what Parliament believes
to be the undue rewarding of illegal immigrants has not actually taken place but is considered only as an
outcome of a hypothetical action).

28. To convey the idea that shifting a portion of health-benefit costs back to workers has two complementary
effects, the correct sentence must link grammatically parallel statements of these effects with and also or with
not only ... but also. In choice A, helps ... but also undermines the and also paradigm, wrongly suggesting a
contrast in the effects. In choice E, the unidiomatic not only ... and violates the not only ... but also paradigm.
Choices B and D are not parallel. Also, the phrase helps the control in B is vague and unidiomatic. Choice C, the
best answer, develops the parallel not only helps to... but also helps to.

29. The enumeration of the rivals requires the conjunction and'. either the rivalry between x and y or the rivals x
and y. Choices A and D wrongly substitute with for and in the first paradigm; choice B wrongly substitutes
against for and in the second. Choice E does not clearly state that Chancellor is party to the rivalry. E also
awkwardly pairs Chancellor and rivalry, not Chancellor and Ransom, as antecedents of they. Choice C, the
best answer, correctly uses the between x and y paradigm and clearly and unequivocally identifies both parties
in the rivalry.

30. To answer this question, you first have to realize that the basic structure of this sentence is fairly simple. The
main verb of the sentence is “grown”. What is grown? A flower is grown, not its derivative. Choices A, B, C and D
all imply that it is a derivative or a powder which is grown out West and not the flower. This is a clear flaw in
logic. The only answer choice that has the correct subject is Choice E.

PARA-JUMBLES – I
SESSION – 9

1. Ans: [a] 2. Ans: [c] 3. Ans: [d] 4. Ans: [b] 5. Ans: [d]

6. Ans: [a] 7. Ans: [b] 8. Ans: [a] 9. Ans: [a] 10. Ans: [b]

11. Ans: [d] 12. Ans: [a] 13. Ans: [c] 14. Ans: [c] 15. Ans: [a]

16. Ans: [c] 17. Ans: [c] 18. Ans: [b] 19. Ans: [c] 20. Ans: [d]

PARA-JUMBLES – II
SESSION – 10

1. Ans: [a] 2. Ans: [a] 3. Ans: [d] 4. Ans: [b] 5. Ans: [a]

6. Ans: [c] 7. Ans: [c] 8. Ans: [d] 9. Ans: [a] 10. Ans: [a]

11. Ans: [b] 12. Ans: [c] 13. Ans: [d] 14. Ans: [d] 15. Ans: [d]

16. Ans: [c] 17. Ans: [a] 18. Ans: [c] 19. Ans: [b] 20. Ans: [d]

Page 83 of 91
VERBAL ABILITY

HAP_VA_BTECH_SEM_4_1218

CRITICAL REASONING
SESSION – 13

1. Ans: [d]
Choice (a): From the phrase “caught between the devil and the deep sea”, it can be concluded that country X
does not want to antagonize either party.
Choice (b): The statement does not give any clue, regarding the whereabouts of the don.
Hence, (a) is a conclusion.
Choice (c): As FBI has sought the help of country X, it is the assumption of FBI that country X can find out the
whereabouts of the don.
Hence, (c) is an assumption.
Choice (d): As it is stated that the don is notorious, it can be inferred that he is involved in illegal activities.
Hence, (d) is an inference.
2. Ans: [d]
Choice (a): As it is asked, to dance with caution, it can be concluded that the situation is such that dancing with
the powerful is necessary. Hence, (a) is a conclusion.
Choice (b): The last sentence ‘dance with caution’ indicates, that it is not necessary to throw ethics to the wind,
if one wants to dance with the devil.
Choice (c): The last sentence ‘dance with the powerful’ and ‘stepping on the devil’s tail’ is understood that the
author assumes that the powerful are devils. Hence, (c) is an assumption.
Choice (d) is the answer.
3. Ans: [d]
Here the problem is the lack of playground. (a) is not a solution for the given problem. (b) is a negative course of
action. Children cannot remain in school for long. Hence, (c) is not feasible.
Choice (d) is the answer.
4. Ans: [d]
Here the problem is that the infrastructure has not developed to match the inflow of people. So (d) is a proper
course of action. (a) is a negative course of action while (b) assumes the existing rules are not sufficient and (c) is
not a practical solution.
5. Ans: [a]
The statement tells us that as a result of an amendment in the constitution children cannot be employed by any
organization. Since it is a constitutional amendment, it follow that all organizations should abide by it. However,
from the statement we cannot deduce what these children would do once they are out of their jobs, hence
conclusion II does not follow. However, I follows, therefore (a) is the answer.
6. Ans: [c]
The statement does not indicate that ABC is the only company that has an envious track record. Hence,
conclusion I does not follow. Also the statement does not describe the expertise required to handle such a
camera. Hence, conclusion II does not follow. Therefore, the answer is choice (c).
7. Ans: [b]
The place is located in a seismically sensitive area. In such places, a multi storeyed building is fraught with risk.
Hence (a) is not a possible consequence.
The government takes immediate relief measures as a first step. One among such relief measures is a private
make step. One among such relief measures is a private make shift arrangements. Hence (b), is a possible
consequence. Relocating the people to other district is not a practical solution. As it leads to other problems like
lose of income source etc. Hence (c), is not a possible consequence.
Choice (b) is the answer.

Page 84 of 91
VERBAL ABILITY

HAP_VA_BTECH_SEM_4_1218

8. Ans: [d]
As a precaution, the government may evacuate the people from low-lying areas. Hence, (a) is a possible
consequence. It is not the duty of the Meteorological Department to take up rescue operations. Hence, (b) is not a
possible consequence.
The Government maintains a separate fund to deal with natural calamities. Moreover, no damage had taken
place so far. Hence (c) is not a possible consequence.
Choice (d) is the answer.

9. Ans: [d]
Choice (a): Lower labor costs mean less expenditure for the winemakers; this does not explain how revenues
would increase.
Choice (b): This statement about low yields does not explain an increase in revenues.
Choice (c): The proximity of production to the vineyard is irrelevant to the question of how hot the dry weather
can be responsible for decreased yield and increased revenues.
Choice (d): Correct. This statement properly provides the explanation that the weather conditions will lead to
better wines. With better wines typically commanding higher prices, the winemakers will gain the increased
revenues that the analysts anticipate. The correct answer is option (d).

10. Ans: [c]


Choice (a): The limited use of some wooden sleds does not weaken the argument.
Choice (b): The absence of protective gear would affect accidents with both kinds of sleds.
Choice (c): This statement weakens the argument by providing an alternate explanation for the increased
accidents.
Choice (d): This statement is true of accidents with both the kinds of sleds. The correct answer is option (c).
11. Ans: [d]

12. Ans: [a]


Choice (a): Correct. This statement properly identifies a weakness in the argument: the response to the road
complaint addresses a different subject, highway capacity, not the issue of traffic congestion encountered by
most motorists.
Choice (b): If high gas prices actually prevented motorists from driving and if motorists’ driving habits were the
same as they were twenty years ago, then these points should strengthen the argument that there is no basis for
their complaints.
Choice (c): The number of commuters who use public transit does not affect the argument that the motorists’
complaints have no basis.
Choice (d): The age of registered cars is irrelevant to the argument. The correct answer is option (a).

13. Ans: [a]


Assumption (I) is implicit since superiority of transistors over radios must be the decisive factor for the
replacement of radios by transistors. Assumption (II) is not implicit since the statement does not, at any stage
imply the banning on the manufacturing of radios by the government.

14. Ans: [d]


Assumption (I) is implicit since it is the supervisor who is going to be in-charge at the examination hall.
Assumption (II) is implicit since the extent of emergency requirement would naturally have to be decided on the
spot (and cannot be predefined). Assumption (III) is not implicit since nothing can be implied about the duration
of the examination.

Page 85 of 91
VERBAL ABILITY

HAP_VA_BTECH_SEM_4_1218

15. Ans: [c]


The passage enumerates the various objectives of TRIPs Agreement as: ‘ensuring that measures and procedures
to enforce intellectual property rights do not themselves become barriers.’ ‘Contribution to transfer and
dissemination of technology,’ ‘promotion of public interest in sectors of vital importance,’ ‘encouraging
adjustments aimed at higher levels of protection of intellectual property rights’. Option (c) is correct as it best
suits the theme of the passage. Option (a) is incorrect as it is not mentioned anywhere in the passage. Option (b)
is incorrect as it partially represents the theme of the passage. Option (d) is incorrect as there has been no
mention of deadlines in the passage.

16. Ans: [c]


The passage itself talks of the limitations of Western allopathic medicine,’ ‘focus on pathology and disease rather
than on prevention,’ and ‘the separation of physical and emotional problems.’ Option (c) is correct as it best suits
the theme of the passage. Option (a) is incorrect as this has not been mentioned anywhere in the passage. Option
(b) is incorrect as it is partially correct in the given context. Option (d) is incorrect as assuming an emotional
relationship between the doctor and the patient will not be enough.

17. Ans: [b]


As per the spokesperson’s conclusion, the earth faces another ice age within the next thousand years and this
could cancel out the effect of global warming. The ice age is expected within the next thousand years but we
cannot wait for the next thousand years as the green house affect may have begun. What if it causes damage
within next 30 or 50 years? This argument would weaken the spokeperson’s conclusion. Option (b) highlights
this and is the correct answer.

18. Ans: [b]


Tests of a newly developed, inexpensive pesticide have shown it to be both environmentally safe and effective
against the insects that have infected the cotton crops.

19. Ans: [e]


Last year many people who placed orders for merchandise in response to advertisements in Systems were not
subscribers to the magazine.

20. Ans: [e]


In Runagia children typically supplement the income of elderly parents, but only by enough to provide them with
a comfortable living

MIMIC THE PATTERN


SESSION – 14

PRACTICE EXERCISE

1. Ans: [c]
Premise: All the cars, when considered together, use a lot of iron.
Conclusion: So, Stanley’s car also uses a lot of iron.
The premise is about cars as a whole, but the flowed conclusion is about one particular car. We need to find an
answer choice that has this same type of flaw in its reasoning. Note the question gives a hint that the argument
is flawed.
(a) – incorrect – This argument is opposite in its reasoning – it takes one person’s desire and assumes that her
entire country desires the same.
(b) – incorrect – The conclusion of this argument is not about one apple, but it is about several apples. This is
not analogous to the original argument.
(c) – correct – This argument has the same type of premise and conclusion as the given argument.

Page 86 of 91
VERBAL ABILITY

HAP_VA_BTECH_SEM_4_1218

2. Ans: [b]
Note the conditional premise in the argument.
Person plays the piano  person must the talented
Tina plays the piano.
Therefore, she must be talented.
(a) – incorrect
Person drivers a car  person must a good driver
Sam does not drive a car. (not analogous)
So, he must not be a good driver. (not analogous)
(b) – correct – The premise and conclusion of this argument is analogous to the original argument.
Person draws well  person must be creative
Todd draws well.
Consequently, he must be creative.

3. Ans: [a]
Ian studies for two hours  he can eat an ice-cream
Ian ate an ice-cream.
Therefore, he studied for two hours.
The flaw in this argument is that it interprets the condition incorrectly. Just because he ate an ice-cream today
does not imply that he studied for two hours. The conclusion is the converse of a conditional.
(a) – correct – This argument has a parallel conditional premise and an incorrect converse conclusion similar to
the main argument.
Train maintains its speed  will arrive in time
Train arrived in time.
Therefore it maintained its speed. (converse of the conditional)
(b) – incorrect – This argument has a different structure than the original. The premise is conditional but the
conclusion is the inverse of the conditional/
it snows  temperature will be cold
it did not snow.
So, the temperature was not cold.

4. Ans: [a]
Vivian was asked to be careful about fire ants.
Vivian claimed that his home was new and will not have fire ants.
So, he ignored the warning.
(a) – incorrect – Ronald argued that his watch was water proof but did not say that his watch was new. This
premise is not analogous.
(b) – correct – This choice is analogous to the original argument
Amber was cautioned to be careful while using the drill.
Amber said that the drill was new and safe.
So, she ignored the warning.

Page 87 of 91
VERBAL ABILITY

HAP_VA_BTECH_SEM_4_1218

5. Ans: [b]
All P are Q.
All Q are R.
Therefore, all P are R.
(a) – incorrect – The conclusion in this argument is not analogous.
All M are N.
All N are O (same as If there is a N then it is a O)
Therefore, All O are M. (this should have been all M are O to be analogous)
(b) – correct – The premises and conclusion are analogous to the original.
All A are B.
All B are C. (same as If there is a B, then it is a C)
Therefore, all A are C.

6. Ans: [b]
Note the causality in the argument.
High temperature noticed first, then blood pressure
Therefore, high temperature c  blood pressure
(a) – incorrect – The conclusion in this argument has its cause and effect reversed.
Stairs swayed first and then squeaking noise
Therefore, squeaking noise c  swaying stairs
(b) – correct – The premise and conclusion are analogous.
Lucy noticed blue van first and then an accident
Therefore, blue van c  accident
They symbol c  refers to the causal relationship. Consult the book titled Critical thinking & Logical reasoning
Primer for a detailed discussion.

7. Ans: [b]
The given argument tests your ability to understand causal relationships. Drawing a causal diagram will help
map the analogies.
PcQ
QcR
Therefore, P c  R
(a) – incorrect – The conclusion is not analogous to the original conclusion.
LcM
McN
N c  L (this conclusion should have been reversed to be analogous)
(b) – correct – The premises and conclusion are analogous.
IcJ
JcK
Therefore, I c  K

Page 88 of 91
VERBAL ABILITY

HAP_VA_BTECH_SEM_4_1218

8. Ans: [b]
Theft of watches is increasing.
So, people must prove ownership of watches when requested.
Note carefully that this argument concludes that ownership of “watches” must be proved because “watches” are
being stolen.
(a) – incorrect
Cars are being stolen in record numbers
So, prove ownership of vehicles when requested.
The conclusion does not refer to “cars” but instead refers to all vehicles.
This is not the same type of reasoning found in the original argument.
(b) – correct – This argument is analogous to the original argument.
Lots of computers are being stolen.
So, People must prove ownership of their computers on request.

9. Ans: [a]
Facial cream was applied to remove pimples
But pimples did not go away
So, pimples were blamed for the problem
This argument blames the pimples instead of the facial cream.
(a) – correct – This argument is analogous to the original argument since it blames the ants instead of the ant
spray.
Spray was used to kill the ants in the kitchen
But the ants did not die
So, the ants were blamed for the problem
(b) – incorrect – If this argument is to be analogous to the original argument, then the sticky substance must be
blamed.
The grease remover was used to remove the sticky substance
Sticky substance did not go away
So, the grease remover was blamed

10. Ans: [a]


Most people like to lead peaceful lives.
So, radio programs dealing with controversies must not be aired.
The reasoning in this argument is that peaceful lives and controversies do not go together. We need to look for
this analogy.
(a) – incorrect
Most children like healthy food.
So, spicy food items must not be served.
Healthy food and spicy food could go together. So, this argument is not analogous to the original argument.
(b) – correct
Most people like humor.
So, television programs that show serious content must not be aired.
Humor and serious content do not go together. This argument is analogous to the original argument.

Page 89 of 91
VERBAL ABILITY

HAP_VA_BTECH_SEM_4_1218

11. Ans: [c]


The correct answer parallels the flawed reasoning of the stimulus. Both give a situation where the first
argument is dismissed on the basis that the second argument presented is false.

12. Ans: [a]


The correct answer most closely conforms with the excerpt.
The incorrect answers are wrong for the following reasons:
Saying that dogs should be treated the same with "slight room for variation" emphasizes continuity over
individualization, and therefore is wrong.
The statement "It is dangerous to apply a broad set of principles.is not stated in the excerpt.
"Unique individuals and unique circumstances call for specialized training to meet their needs" exaggerates the
need for individualization.
The concept of "outliers" is not present in the excerpt, a red flag that this is an incorrect answer choice.

13. Ans: [c]


The correct answer choice is the only one which commits the fallacy of affirming the consequent. In other
words, both it and the original argument improperly assume the converse of their conditional statements. We
know that the Burger Shack needs to offer fries and drinks to maximize its revenue, but we do not know that its
revenue is necessarily maximized if it offers them. Likewise, we do not know whether Beverly is a successful
piano teacher simply because she has a clear course of instruction and a patient personality.

14. Ans: [e]


The flawed reasoning in the text is as follows:
X has trait Y. Therefore, anything with trait Y must be X. If a person encounters something with trait Y, then it
must be X.
The correct answer properly reflects this reasoning. All the other answer choices do not properly follow this
pattern: they may leave out a link of the causal reasoning or misconstruct it.

15. Ans: [e]


The flow of reasoning in the argument is that if X has Y quality, then Z is likely to happen.
In the text,
X = wine
Y = receives a high score from the National Sommelier Association
Z = likely to be sold at fine dining restaurants
The correct answer follows this reasoning:
Pearls that are larger than average are more likely to retail for a higher price. A pearl diver found a dozen
pearls that are much larger than average. Therefore, the pearls are more likely to retail for a higher price.
X = pearls
Y = larger than average
Z = more likely to retail at a higher price

16. Ans: [d]


The flawed reasoning used in the passage is that a combination of two "parent" items with different attributes
necessarily yields a "child" product having attributes that are averages of its parents' attributes. The correct
answer uses parallel reasoning inasmuch as the argument uses the fact that halogen and noble gases differ with
respect to toxicity to conclude that a combination of such gases would yield a gas having toxicity that is the
average of the toxicity of its "parent" gases.

Page 90 of 91
VERBAL ABILITY

HAP_VA_BTECH_SEM_4_1218

17. Ans: [c]


The stimulus holds that an opinion is false on the basis that the person with that opinion has made past
mistakes in the same area. The correct answer is similar because it follows the same pattern of critiquing a
position based on the person with the opinion not the opinion itself.

18. Ans: [b]


The Forestry Service's main goal is to reduce the number of forest fires from year to year. Their reasoning is
that educating people on how fires are started is the most effective way to achieve this goal. Similarly, the
school wishes to reduce car wrecks among teenage drivers by educating teenagers about safe driving practices.

19. Ans: [d]


The secret society keeps its rituals secret, as do all of the other examples given in the answer choices; however,
the secret society fosters this secrecy in the hope that it will attract more people to its ranks. This is most
similar to a magician keeping secrets in order to attract an audience.

20. Ans: [b]


The piano teacher speaks with the most sincerity because he does not speak with any filters, and has no reason
to edit what he says.
The other answer choices are incorrect for the following reasons:
There is no evidence in this answer choice that the writer said anything that was sincere in his email; all that
we know is that he was releasing his frustration.
The student does not channel completely unfiltered sincerity given the fact that she is making an effort to
“respectfully” communicate her frustrations to her professor. She therefore is using filters in her
communication with him.
The mother is trying to pressure her son to study; she is not expressing her true feelings that she is worried he
will not earn good grades.
Although the husband is speaking in part from the heart, he is also speaking with the motivation to make his
wife happy. This therefore does not comport with the poet’s philosophy.

Page 91 of 91

You might also like